Dies ist ein Archiv der Seite Wikipedia:Auskunft. Es enthält alle Abschnitte, die in der Kalender-Woche 02 im Jahr 2023 begonnen wurden.

Möchtest du in einer neuen Diskussion auf dieses Archiv verweisen? Nutze folgenden Link und ersetze ________ durch den Namen des Abschnittes. Die Nummer im Inhaltsverzeichnis gehört nicht dazu:

[[Wikipedia:Auskunft/Archiv/2023/Woche 02#________]]


← vorherige Woche Gesamtarchiv nächste Woche →

Lesehilfe benötigt

 
@Kreuzschnabel: Bibelkolporteure gab es um diese Zeit (1896) sogar mit Pferd und Wagen - hier in Nordholland!

Moin! Ich habe hier die Geburtsurkunde des Nazi-Staatssekretärs Georg Ahrens (1896-1974) gefunden. Ich kann alles lesen, nur in der Zeile wohnhaft zu die Straßenangebe nicht. Wer hilft mir auf die Sprünge? MfG, GregorHelms (Diskussion) 10:31, 9. Jan. 2023 (CET)

Da müsste ich mich mit einem Testaccount registrieren! --TheRunnerUp 10:37, 9. Jan. 2023 (CET)
Vielleicht könnte Benutzer:GregorHelms einen Screenshot machen und auf Commons hochladen. Das spart allen Nichtbenutzern von ancestry.de das Probeabo. --Rôtkæppchen₆₈ 10:53, 9. Jan. 2023 (CET)
Habe in diesem Zusammenhang schon einen Versuch gestartet ... muss aber wiederholt werden! MfG, GregorHelms (Diskussion) 10:57, 9. Jan. 2023 (CET)
Foto temorär hochladen geht am einfachsten mit https://de.imgbb.com/ (Ich hoffe mal, dass die Seite seriös und unproblematisch ist) --Doc Schneyder Disk. 11:00, 9. Jan. 2023 (CET)
Versuch 2: [1] MfG, GregorHelms (Diskussion) 11:06, 9. Jan. 2023 (CET)
Delmenhorst, Hasbergerstraße No. 28. Das ist ja eine witzige Schriftenmischung. --Kreuzschnabel 11:20, 9. Jan. 2023 (CET) Bibel-Colporteur ist andererseits auch ein interessanter Beruf :D --Kreuzschnabel 11:22, 9. Jan. 2023 (CET)
Bei Kirchenbucheintragungen und standesamtlichen Urkunden ist es ziemlich weit verbreitet, dass der „Fließtext“ in deutscher Kurrentschrift und die Eigennamen in lateinischer Schrift geschrieben sind. --Jossi (Diskussion) 12:12, 9. Jan. 2023 (CET)
Die Straße gibt es noch; wird heute Hasberger Straße geschrieben. --Proofreader (Diskussion) 12:55, 9. Jan. 2023 (CET)
Ja, aber inwiefern ist der Straßenname eine andere Sorte Name als der Ortsname, so dass „Delmenhorst“ lateinisch und die Straße in Kurrent geschrieben ist? --Kreuzschnabel 13:31, 9. Jan. 2023 (CET)
An dieser Stelle ein herzliches Dankeschön! MfG, GregorHelms (Diskussion) 16:27, 9. Jan. 2023 (CET)

Fehlt dort nicht eine Angabe dazu, wie viele Wähler es insgesamt gibt? Zudem fehlt eine Erläuterung, woher die Zahl 1,96 kommt … --Leyo 14:09, 9. Jan. 2023 (CET)

Die 1,96 sind ein Fraktilwert der Normalverteilung: "95 % aller Messwerte haben eine Abweichung von höchstens 1,960 σ vom Erwartungswert". Wenn man sonst nichts besseres weiß oder sich nicht intensiv mit statistischen Verteilungen beschäftigen will, nimmt man einfach die Irrtumswahrscheinlichkeit 5 % und diesen Fraktilwert. Die Anzahl der Wähler insgesamt ist nicht wichtig, wenn man als Modell das Urnenmodell mit Zurücklegen verwendet oder die Anzahl der Wähler viel größer als die Stichprobe ist. 91.54.32.105 15:11, 9. Jan. 2023 (CET)
Gesetz der großen Zahl - die Werte gelten für eine theoretisch unendlich große Grundgesamtheit. Sie sind konservativ, da sie für endliche Grundgesamtheiten generell noch besser sind (PS: Da der Anteil der bekannten gegenüber der unbekannten Information sich erhöht), was aber nur dann wirklich spürbar ist, wenn die Grundgesamtheit nicht ein starkes Vielfaches der Stichprobe ist. Bei kleinen Grundgesamtheiten müssen natürlich andere Berechnungsansätze gewählt werden. -- 79.91.113.116 15:39, 9. Jan. 2023 (CET)
Besten Dank für eure Antworten! Mögt ihr die Essenz davon in den Artikel einarbeiten? --Leyo 19:42, 9. Jan. 2023 (CET)

Citius, Altius, Fortius: Muss das bei Entkalkern auch sein?

Hallo liebe Wikipedianer! Früher, da stieg ich in die Dusche. Schnappte mir ein Schwämmchen, sprühte Entkalker in die schlimmste Ecke (habe eine Spezialdusche die mit orangener Farbe anzeigt wo sich derzeit der meiste Kalk befindet) und reinigte diese Stelle. Dies mit der blanken Hand. Irgendwann aber, ging das nicht mehr. Der Entkalker hat böse meiner Haut auf der Hand zugesetzt. Klaro! Stolz berichtet ja jeder Entkalker: Jetzt noch stärker! Neue Formel: Noch intensiver! Tja, das schnelle kurz mal reinigen ist nicht mehr. Eigentlich sind jetzt jedes Mal Gummihandschuhe notwendig. Für die Umwelt eigentlich gar nicht so doll. Die Frage an das geballte Wissen hier: Gibt es Alternativen? Oder gibt es noch absichtlich schwache Reiniger? Quasi ein unverändertes Produkt wie vor zehn Jahren? Fortschritt erscheint mir hier eher Rückschritt zu sein. Rolz Reus (Diskussion) 09:11, 9. Jan. 2023 (CET)

Die Industrie müllt uns und die Umwelt ja zu mit allerlei Spezialprodukten, wo eigentlich wenige einfache Mittel genügen. Spülmittel, Neutralreiniger, evtl. noch Spiritus, Essig (ggf. zusammen mit Natron) oder Zitronensäure erfüllen eigentlich ihren Zweck. Essig(reiniger) oder Zitronensäure könnten dir helfen. Du kannst dir ja eine Spülbürste in die Dusche hängen, dann musst du nicht mit Händen oder Handschuhen ran. -- MonsieurRoi (Diskussion) 09:26, 9. Jan. 2023 (CET)
Bzgl. "Für die Umwelt..." wirbt Frosch immer sehr (und für mich plausibel). Das Sicherheitsdatenblatt deren Entkalkers nennt nur Zitronensäure als gefährlichen Inhaltsstoff. Ist im Endeffekt was MonsieurRoi sagt nur mit Geruch ;o) ...Sicherlich Post 09:30, 9. Jan. 2023 (CET)
Erg.: Natürlich ist das auch nicht unbedingt hautschonend, aber mit Feenstaub wird es vermutlich nicht klappen. -- MonsieurRoi (Diskussion) 09:32, 9. Jan. 2023 (CET)
In unserer Tabelle mit Säurekonstanten ist zwischen Kohlensäure (pKS-Wert 6,52) und Essigsäure (4,75) eine Lücke. So gesehen ist Essigsäure die gesuchte Säure, auch wenn sie fast zwei Größenordnungen zu stark für das eigentliche Problem ist. --Rôtkæppchen₆₈ 10:00, 9. Jan. 2023 (CET)
zwei Grössenordnungen zu stark? Das würde dann heissen, dass man auch mit Kohlensäure Kalk auflösen könnte?? Was ja gar nicht stimmt.
Als Entkalker benutze ich i.d.R. Zitronensäure, die man günstig im Drogeriemarkt oder im Versand erhält. Einfach nur warm verwenden, nicht heiss und nicht bei Aluminium. -- Peter LX (Diskussion) 11:17, 9. Jan. 2023 (CET)
Im Prinzip könnte man auch Kohlensäure verwenden. Wenn man sie genügend lange einwirken lässt, löst sich der Kalk unter Bildung von Calciumhydrogencarbonat im kohlensäurehaltigen Wasser auf. Sie ist aber wegen ihrer geringen Wasserlöslichkeit unpraktisch. Siehe Wasserhärte#Kalk-Kohlensäure-Gleichgewicht. Citronensäure (pKS1 = 3,13) ist noch ein bisschen stärker als Essigsäure, also eher nicht das, was Benutzer:Rolz Reus sucht. --Rôtkæppchen₆₈ 11:30, 9. Jan. 2023 (CET)
Theoretisch hast du ja recht, aber in der Praxis, bei bestehenden Kalkrändern, kommt man halt mit Sprudelwasser nicht weit. -- Peter LX (Diskussion) 14:28, 9. Jan. 2023 (CET)
Grundsätzlich gibt es auch wiederverwendbare Gummihandschuhe. Sicher nicht die optimale Lösung, aber vielleicht geeignet, bis das aktuell vorhandene Produkt aufgebraucht ist. --Erastophanes (Diskussion) 14:11, 9. Jan. 2023 (CET)
Bin mittlerweile auf Natron für fast alles umgestiegen. Hilft auch bei Kalk. Mit Wasser zu einer Paste anrühren, auf die Stellen auftragen und einwirken lassen. Bei ganz hartnäckigen Stellen Natron plus Zitronensäure. Wenn du kein Natron da hast, kannst du es auch mal mit Backpulver versuchen (auf Dauer aber zu teuer im Vergleich zu Natron). --194.56.48.110 16:55, 9. Jan. 2023 (CET)
Bei festem Kalk wirkt Natron nur scheuernd, aber nicht lösend. Dann kann man genausogut Viss o.ä. nehmen. Anders ist es bei hartem Wasser, wo Natronzugabe ähnlich wie Schnellentcarbonisierung wirkt und den gelösten Kalk ausfällt. --Rôtkæppchen₆₈ 20:31, 9. Jan. 2023 (CET)
Einfach aufsprühen und dann mit blanker Hand wischen ist doch ohnehin schon Murks. Lass es einfach über Nacht einwirken und wiederhole das gegebenfalls. Haushaltspapier sorgt dafür, dass es länger an der Wand bleibt und nicht gleich alles runterläuft. Was an wiederverwendbaren Gummihandschuhen nun so umweltschädlich sein soll, weiß ich gerade nicht (mag aber sein). Lies auch ruhig die Anleitung, wo du schon die Gesetze, die du ausführen sollst, nicht liest (das warst doch du, oder?). Aber nee, hast Du mit den Reinigern ja auch immer schon so gemacht … --77.1.151.72 21:56, 9. Jan. 2023 (CET)
Wahrscheinlich würde schon eine Bürste (Spülbürste o.ä.) reichen, um den aggressiven Entkalker aufzutragen. --Rôtkæppchen₆₈ 01:47, 10. Jan. 2023 (CET)
(BK) Ein paar Blatt mit Frosch o. ä. durchfeuchtete Haushaltsrolle hält sich gut über Nacht an der Wand und gewährleistet eine längere Einwirkung. Was so speziell an seiner Dusche sein soll, konnte ich nicht herausfinden; der orangene Belag ist der Kalk, siehe zB hier. --95.112.1.180 10:16, 10. Jan. 2023 (CET)
Schonendste Alternative: Gleich trocken wischen nach jedem Duschen--2003:DF:7F10:7100:DCB4:2216:E2E5:1459 09:47, 10. Jan. 2023 (CET)

Wie groß muss ein Mond sein, um als "Mond" zu gelten?

Siehe oben und [[2]]

--2003:D5:FF4C:5500:BCCC:C7E3:5D7A:C1CD 13:32, 9. Jan. 2023 (CET)

Keine Mindestgröße vorgegeben; wirf einen Kiesel in eine geeignete Umlaufbahn und wir haben einen neuen Erdmond. In diesem Fall stellt sich die Frage allerdings nicht, da 2022 YG ein Quasisatellit ist und kein Mond. Quasisatellit heißt: Das Ding umläuft die Sonne wie alle anderen Asteroiden auch, aber in Resonanz mit der Erdbahn, so dass es von der Erde aus gesehen diese im Laufe eines Jahres einmal rückwärts zu umrunden scheint. Das ist aber keine gravitative Bindung, eher eine optische Täuschung. --Kreuzschnabel 13:48, 9. Jan. 2023 (CET)
Ein netter Fall ist J002E3. Den hatte man zeitweise für einen natürlichen Asteroiden gehalten bis sich bei näherer Beobachtung herausstellte, dass es offenbar die 3. Stufe der Saturn-V-Rakete der Apollo-12-Mission war. Sie näherte sich Ende 2002 der Erde aus einem heliozentrischen Orbit, umkreiste die Erde 6 Mal, um dann wieder in einen heliozentrischen Orbit überzugehen. Wo wir bei Saturn waren, das wäre dann ein Saturn-Satellit der besonderen Art. Den umgekehrten Fall gab es auch, dass vermeintlicher Weltraumschrott sich dann als natürlicher Himmelkörper herausstgestellt hat: 2006 RH120. --Proofreader (Diskussion) 18:24, 9. Jan. 2023 (CET)

Der kleinste in dieser Liste ist Dimorphos, ein Mond des Asteroiden Didymos, und hat einen Radius von etwa 85 Metern, aber Kreuzschnabel hat recht, eine Untergrenze gibt es nicht; "Mini-Monde" bezeichnen unsere Kollegen in der en WP auch als en:Moonlets; die zählen aber auch zu den Monden. Auch die Körper, aus denen z.B. die Saturn-Ringe bestehen, sind in dem Sinne Monde. --Proofreader (Diskussion) 16:56, 9. Jan. 2023 (CET)

So ganz einig scheint man sich da aber nicht zu sein. Der Artikel Satellit (Astronomie) etwa (auf den man von Mond (Begriffsklärung) als "Mond, Trabant anderer Himmelskörper" verwiesen wird) zählt in seinem Kapitel "Monde von Planeten" nur 205 Monde in unserem Sonnensystem, davon 82 bei Saturn, die Ringe werden nicht erwähnt. Der Artikel Saturn (Planet) selbst listet in der Infobox unter "Monde" "82 + Ringsystem" (zählt die Ringe also irgendwie mit), im Kapitel "Monde" sind es aber wieder nur die 82 (von denen sich einige innerhalb der Ringe bewegen) und ziemlich ausdrücklich nicht die ca. 10 Mio. "Möndchen" aus denen die Ringe bestehen. In der Liste der Saturnmonde werden die Ringe gleich nur noch unter "Siehe auch" erwähnt. --Karotte Zwo (Diskussion) 17:05, 9. Jan. 2023 (CET)
„man“ liegt aber auch nicht direkt im Streit darüber, ob die Brocken der Saturnringe Monde sind oder nicht. Technisch sind es natürlich welche, aber inwiefern man das anwendet, hängt vom Kontext ab. Sie in eine Liste der Monde des Sonnensystems einzubauen wäre beispielsweise einigermaßen hoffnungslos, dann erwähnt man sie halt einmal pauschal oder vernachlässigt sie. Wenn das in der WP nicht ganz einheitlich gehandhabt wird, will ich da nicht gleich Uneinigkeit seitens der Autoren sehen. Es sind halt unterschiedliche Bewertungen. Innerhalb eines Artikels kannst du das gern angleichen oder im Abschnitt „Monde“ einen Hinweis darauf setzen, dass die Ringmöndchen vernachlässigt werden, das schadet natürlich nicht. --Kreuzschnabel 18:56, 9. Jan. 2023 (CET)
Nun, mal so betrachtet: Im Moment ist die Unterscheidung zwischen individuell die Planeten umkreisenden Gesteinskörpern und in der Masse aufgehenden Kleinteilchen noch relativ einfach zu handhaben. Wenn man irgendwann noch stärker mit der Umgebung einzelner Planeten vertraut wird, können die Grenzen irgendwann verschwinden und eine formale Definition von Monden, z.B. anhand der Größe und individuellen Bahncharakteristiken, könnte notwendig werden - ähnlich wie es bei den Planeten war, nachdem etliche Himmelskörper in Sonnenumlauf gefunden wurden, die größer als Pluto waren. -- 79.91.113.116 11:01, 10. Jan. 2023 (CET)

SynchPop Band "Accus"

aktiv in den 1990er Jahren, u.a. mit den Titeln "seven", "ClickClick", "Hunted", "Peace Museum". Sprache: Englisch. Im I-Net gar nichts gefunden (war damals noch Luxus, daher gab es wenig Input). Wo kamen die her? --2A01:598:88BA:B994:3CB8:B0BF:505B:FBD0 13:28, 10. Jan. 2023 (CET)

Meinst Du eigentlich Synth-Pop? Ist der Name der Kapelle ggf. auch falsch geschrieben? --2.174.201.8 15:06, 10. Jan. 2023 (CET)
Hier wäre eine umfangreiche Liste von Synth-Pop-Bands: https://en.wikipedia.org/wiki/List_of_synth-pop_artists --Doc Schneyder Disk. 16:09, 10. Jan. 2023 (CET)

Rätselaufgabe(?) Briefumschlag auswählen

Einschub zur Klarstellung: In der derzeit aktuellen Fassung des Artikels "Umtauschparadoxon" lautet die Beschreibung der Spielsituation:
Herr Lemke möchte Herrn Schmidt beschenken und gibt ihm zwei Briefumschläge mit den Worten „Ich schenke Ihnen einen dieser Umschläge. In beiden befindet sich ein Geldbetrag, im einen doppelt so viel wie im anderen. Sie dürfen einen Umschlag öffnen und dann entscheiden, welchen der beiden Umschläge Sie nehmen.“
--95.119.143.151 11:29, 9. Jan. 2023 (CET)

Es gibt so eine paradoxe Denkaufgabe: jemand soll einen von zwei Briefumschlägen wählen. Ihm wurde mitgeteilt, daß in dem einen Umschlag doppelt soviel Geld wie in dem anderen ist, aber natürlich nicht, wieviel und in welchem. (Realistisch wären beispielsweise die Paarungen 50/100, 100/200, 200/400 oder 500/1000 Euro, eher nicht solche wie 5/10 oder 10000/20000 Euro.) Der Kandidat will natürlich soviel Geld wie möglich bekommen, sinnt also über eine Strategie nach, möglichst den Umschlag mit dem höheren Betrag zu erhalten. Er überlegt sich folgendes: "Eigentlich habe ich jetzt [o. B. d. A.] den Umschlag A gewählt." (Darin sind z. B. - oder vielleicht darf er sogar nachschauen, dann also sicher - 100 Euro, aber über den Inhalt von B bekommt er keine Informationen. B liegt aber sichtbar auf dem Tisch, daran kann während des Auswahlprozesses nichts mehr manipuliert werden.) "Im Umschlag B sind dann also entweder 50 oder 200 Euro. Der Erwartungswert bei einem Wechsel beträgt also 1/2*50+1/2*200=125 Euro, und das ist mehr als die 100 Euro im Umschlag A. Ich sollte lieber Umschlag B nehmen." (Was natürlich paradox ist, denn hätte er zuerst B gewählt, dann müßte er mit der gleichen Argumentation zu A wechseln.) - In welchem Artikel wird diese Aufgabenstellung abgehandelt? --95.112.136.171 22:12, 6. Jan. 2023 (CET)

Das hört sich für mich nach einer Abwandlung des Ziegenproblems an. --2001:9E8:49CC:5300:EDDD:F37:ED93:6A82 22:15, 6. Jan. 2023 (CET)
Umtauschparadoxon --91.141.51.231 22:19, 6. Jan. 2023 (CET)
Der Erwartungswert in der dargestellten einfachen Bedeutung ist im Übrigen ein Teil des frequentistischen Wahrscheinlichkeitsbegriffs. Das Experiment ist jedoch für den frequentistischen Wahrscheinlichkeitsbegriff untauglich: Wenn das Experiment mehrfach durchgeführt werden soll, müsste eine entsprechende Anzahl an Alternativumschlägen bereitgelegt werden, in der in der Hälfte der halbe Betrag und in der anderen Hälfte der doppelte Betrag, also im Mittel mehr als der Betrag des gewählten Umschlags enthalten sind. Dann ist es nicht mehr paradox, dass ein Wechsel auf den anderen Umschlag vorteilhaft ist. --BlackEyedLion (Diskussion) 00:19, 7. Jan. 2023 (CET)
Man muss überhaupt überlegen, was eine Wiederholung des Zufallsexperiments bedeutet: Das Paradoxon tritt auf, wenn man einmal einen Umschlag wählt und n-mal wechselt. Richtig ist aber, n-mal einen Umschlag zu wählen und zu wechseln. --BlackEyedLion (Diskussion) 09:37, 7. Jan. 2023 (CET)
Man muß überhaupt keine mathematischen Grundlagenüberlegungen anstellen: In der Aufgabenbeschreibung liegen da erst einmal zwei Briefumschläge auf dem Tisch. Dazu kommt dann noch unausgesprochenes "Weltwissen", daß man für die möglichen Beträge darin grob eine obere und eine untere Grenze annehmen kann. (Wenn der Betrag in dem Umschlag, in den man reingeschaut hat, nahe einer dieser vermuteten Grenzen liegt, ist es "wahrscheinlich", daß der in dem anderen in der entgegengesetzten Richtung liegt. Beispiel: die Kandidat nimmt an, daß - was aus der Aufgabenstellung nicht hervorgeht - in den Umschlägen ziemlich gleichwahrscheinlich Beträge aus dem Intervall [5, 1000] enthalten sein können, aber keine höheren oder niedrigeren. Findet er nun in "seinem" Umschlag a) 10 oder b) 1000, dann ist es im Fall a vernünftig, zu wechseln (obwohl 20 in seiner Vorstellung nicht wahrscheinlicher als 5 ist) und im Fall b unvernünftig, weil er 2000 ausgeschlossen hat. (Fall a wäre diskutabel: zwar hält der Kandidat an sich beide Möglichkeiten für gleich wahrscheinlich, zieht aber in Betracht, daß er sich über die konkreten Werte der vermuteten Intervallgrenzen geirrt haben könnte - tatsächlich wäre der "Moderator" gar nicht auf die Idee gekommen, nur 5 in einen Umschlag zu stecken, sondern das Minimum wäre 10, und diese Irrtumsmöglichkeit hinsichtlich der "unbekannten und unausgesprochenen Spielregeln" läßt den Wechsel als günstiger erscheinen. - Die "Spielregel" könnte aber auch sein "der Moderator hat in den einen Umschlag 50 und den anderen 100 gelegt, fertig", aber dann wäre Fall a auch gar nicht aufgetreten, und die Vermutungen über die Intervallgrenzen waren auch Humbug.) Diese Vermutung der Existenz von "Randlagen" widerlegt aber die Annahme, daß es ohne nachzuschauen immer richtig ist, zu wechseln: Findet er hingegen 100 vor, gibt es keinen Grund, 50 oder 200 für wahrscheinlicher zu halten. Beim Wechseln ist dann der mögliche Gewinn größer als der mögliche Verlust, also wechseln. (Bzw. je nach Risikopräferenz: Was man hat, das hat man - 100 sind doch schon ein ganz netter Betrag, 50 verlieren wäre doof, und vielleicht 100 gewinnen ist die "Taube auf dem Dach", da nimmt man dann doch vielleicht lieber die sichere Blinde, äh, den "Spatz in der Hand".) Die Situation ist vergleichbar mit "fremde Stadt, Hunger, zwei Imbißbuden in Reichweite, ähnliches Angebot, ähnliche Preise: Wo jetzt futtern?" Da kommt dann ein sehr vertrauenswürdiger Fremder vorbei und sagt: "Gehen Sie zu ****, die ist doppelt so gut wie die andere", aber **** hat man im Verkehrslärm leider nicht verstanden, und dann ist der Tippgeber auch schon in den Bus gestiegen und leider weg. (Und hier gibt es jetzt keinen plausiblen oder Anscheinsgrund, eine Wahl zu ändern.) Paradox wird die an sich vernünftige Wechselentscheidung dadurch, daß in dem anderen Umschlag nun mit Sicherheit 50 oder 200 sein müssen, aber wenn man den gewählt oder gezeigt bekommen hätte, mit der gleichen Argumentation der Wechsel zum 100er-Umschlag (von dem man den Inhalt natürlich nicht kennt, man hat also gar keinen Anhaltspunkt dafür, daß es sich dabei um einen 100er-Umschlag handelt) für vernünftig halten müßte - beides gleichzeitig kann aber nicht stimmen. Mit der Idee der Gleichwahrscheinlichkeit muß also irgendwas faul sein, aber was? Im Rahmen der angenommenen Vorstellungswelt sind 50 oder 200 in dem anderen Umschlag nun einmal genauso wahrscheinlich wie die "richtige" oder die "falsche" Imbißbude, und Häufigkeiten sind hier schlicht nicht anwendbar, die Wahrscheinlichkeiten bleiben aber trotzdem. Die Situation ist eher mit Buridans Esel vergleichbar, aber da man nun nicht verhungern will, bestellt man sein Fast food halt bei der einen Bude, bzw. wechselt trotz zweifelhafter Argumentation den Umschlag (bzw. tut das aufgrund der Risikoaversität, die aber mit Erwartungswerten gar nichts zu tun hat, gerade nicht) - wenn's schon nichts nützt, dann wird es wohl (wahrscheinlich(?!?)) nicht schaden... --77.3.177.76 14:24, 7. Jan. 2023 (CET)

Die Frage ist falsch gestellt. Denn in Wirklichkeit geht es um die Entscheidung, 100 € in bar zu nehmen oder einen Umschlag, in dem 50 € oder 200 € sind. Und da ist die Entscheidung für den Umschlag nicht mehr paradox. Erst recht nicht wenn in dem Umschlag 0 € sind oder 5000 €. --2001:9E8:A522:AE00:3467:22E7:4C87:BF90 14:24, 8. Jan. 2023 (CET)

Bahnhof? Welche Frage denn? --77.1.125.152 19:27, 8. Jan. 2023 (CET)
Die Frage lautet „In welchem Artikel wird diese Aufgabenstellung abgehandelt?“ Sie wurde von 91.141.51.231 um 22:19 am 6. Jan. 2023 (CET) umfassend beantwortet. Die Lösung des vermeintlichen Problems ist: Nimm unbesehen den anderen Umschlag, denn er enthält 1,25 mal soviel Geld. Der Rest ist Spekulation darüber, wieviel Geld die Spielleitung in die Umschläge stecken könnte. --Rôtkæppchen₆₈ 19:56, 8. Jan. 2023 (CET)
Nicht "umfassend", sondern konzise, aber zutreffend. - Und ob die Wechselempfehlung wirklich so das Gelbe vom Ei ist, sollte eigentlich der genannte Artikel beantworten, was er aber nicht wirklich tut. (Tagelanges Lesen des jahrzehnte- und kilometerlangen wirren Artikel-Diskarchives trägt auch nicht wirklich zur Klarstellung bei.) Und so ganz ohne Spekulationen über das Verhalten der Spielleitung geht es auch nicht: Ob das Indifferenzprinzip, daß es nämlich keinen Grund gibt, eine der beiden möglichen Spielsituationen für wahrscheinlicher zu halten, gerechtfertigt ist, erfordert nämlich genau diese Spekulationen. (Übrigens ist nie das 1,25-fache im anderen Umschlag, sondern immer nur entweder die Hälfte oder das Doppelte.) - Aber noch kurz einmal die Beschreibung der Spielsituation: Es gibt zwei Umschläge, in dem einen befindet sich der (unbekannte) Betrag x und in dem anderen 2*x. Der Kandidat schaut in einen Umschlag und findet den Betrag A vor. Nun gibt es genau zwei Möglichkeiten: Entweder befinden sich in den beiden Umschlägen die Beträge A/2 und A, also x1=A/2, und der Kandidat hat den Betrag 2*x1=A gesehen. Oder in den Umschlägen befinden sich die Beträge A und 2*A, also x2=A, und der Kandidat hat x2 gesehen. (Zwangsläufig bzw. gemäß Voraussetzung ist rein rechnerisch x1=A/2 und x2=A, also x2=2*x1, aber natürlich können in den Umschlägen nur die Paare P1=(x1,2*x1) oder alternativ P2=(x2,2*x2) vorliegen, aber niemals können x1 und x2 gemeinsam im Spiel sein.) Das Entscheidungsproblem des Kandidaten ist nun, darüber, ob oder in welchem Maße das Vorliegen von P1 wahrscheinlicher oder unwahrscheinlicher als das von P2 sein könnte, einen Schluß zu ziehen, maW muß er raten, ob das, was da unsichtbar im Gebüsch raschelt, ein Tiger oder ein Löwe ist, um mal den den saudümmsten Vergleich, der mir gerade einfällt, zu strapazieren. Wenn er denkt, daß das Indifferenzprinzip anwendbar ist und er hinreichend risikoaffin ist, den Verlust von A/2 gegenüber den "sicheren" A, der sich einstellt, wenn es nicht, wie erhofft, P2, sondern leider nur P1 ist, dann sollte er tauschen, denn die Gewinnerwartung des Wechsels übersteigt die Verlusterwartung deutlich. ("Gewinn" bedeutet die beim Wechseln auftretende Differenz gegenüber den "sicheren" A, die er bekommt bzw. behalten darf, wenn er nicht wechselt, also (-A/2) im Fall P1 und +A im Fall P2. Wenn er nicht vom Indifferenzprinzip ausgeht, sondern dem Vorliegen von P1 oder P2 Wahrscheinlichkeiten p(P1)=p1 und p(P2)=1-p1 zuordnet - wofür es in der Aufgabenstellung keinen zwingenden Grund gibt - dann ist der Gewinnerwartungswert für einen Wechsel E=((1-p1)-p1/2)*A, und der ist für p1<2/3 positiv.) Das Ergebnis ist anscheinend oder tatsächlich paradox, weil es nicht von A abhängt. Wenn der Kandidat nämlich einen der beiden Umschläge rein zufällig ausgewählt und geöffnet hat, kommt er ggf. für jedes mögliche darin vorgefundene A zu dem Schluß, daß ein Wechsel den Gewinnerwartungswert erhöht, d. h. er braucht überhaupt nicht nachzusehen, um die Wechselstrategie für potentiell lukrativer zu halten. Das erscheint aber aus Symmetriegründen widersinnig zu sein. ("Scheint"; das bedeutet nicht zwangsläufig, daß es das auch wirklich ist! (Wobei sich wiederum die Frage stellt, was die Aussage "ist paradox" eigentlich bedeutet.)) Es kann aber sein, daß die Annahme des Kandidaten über die Größe von p1 mehr oder weniger scharf von A abhängt, und dann kommt es, u. a. auch je nach der - in der Aufgabenstellung ebenfalls nicht thematisierten - Risikopräferenz des Kandidaten sehr wohl auf die Höhe von A an, und die Empfehlung "immer wechseln, unbesehen" kann dann falsch sein. Zusammengefaßt scheint es sich nach meinem Eindruck weniger um ein Paradoxon, als vielmehr um eine nicht gut gestellte Aufgabe zu handeln, die die beiden relevanten Gesichtspunkte "Risikopräferenz" und "Annahmen über die Eintrittswahrscheinlichkeiten der möglichen Ereignisse P1 oder P2" unzulässig ausklammert und deswegen keine mathematisch exakte Lösung hat; der Artikel scheint das nicht adäquat zu behandeln. (Wenn man nach der Belegsituation schaut: die zitierten Autoren anscheinend auch nicht. - Und jetzt? Publikumsbeschimpfung? Wikipedia-Bashing? WP:TF? - Wohl eher ein Fall fürs Mathematik-Qualitätssicherungsportal.) --77.1.125.152 02:06, 9. Jan. 2023 (CET)
Guter Punkt. Man muss den Umschlag gar nicht öffnen, man muss nur an A denken und dann B wählen, um die 1,25-fache Summe zu erhalten? Da stimmt natürlich irgendwas nicht. Lösung: Man berechnet gar nicht den Inhalt von Umschlag B, sondern nur das Verhältnis von B relativ zu A, wobei A jeweils einen unterschiedlichen Wert hat. Wenn man mal, wie oben, von 100 als Basis und dem Halben (50) und dem Doppelten (200) ausgeht, dann erhält man vier Paare für A/B: 50/100, 100/50, 100/200, 200/100. Ist jedes Paar A/B nun z.B. in einem größeren Umschlag verpackt und gleichverteilt, dann zieht der Kandidat ein bestimmtes Paar mit der Wahrscheinlichkeit 1/4.
A B nxA
1 50 100 2,0xA
2 100 50 0,5xA
3 100 200 2,0xA
4 200 100 0,5xA
Summe 450 450 5,0xA
Erwartungswert(Summe/4) 112,5 112,5 1,25xA
nxA erhält man, wenn man zu B wechselt. Der Erwartungswert für den Inhalt jedes Umschlags ist gleich: 112,5 erhält man bei A und bei B im Durchschnitt, wenn man das Spiel ganz oft wiederholt. Der Erwartungswert von B relativ zu A ist 1,25xA, weil man ja nur 50% verlieren, aber 100% gewinnen kann. Daran ändert sich auch nichts, wenn man ein zusätzliches Paar hinzufügt, z.B. 61/122 und entsprechend 122/61. (Der Kandidat muss übrigens so doof sein, dass er nicht merkt, dass nur diese drei Werte vorkommen, sonst würde er im Fall 4 natürlich nicht mehr wechseln.) --Optimum (Diskussion) 11:42, 9. Jan. 2023 (CET)
Das sind alles Argumente, die in der einen oder anderen Form schon in der Artikel-Disk vorkamen und weder überzeugend noch wirklich hilfreich sind. Es gibt keine vier Umschlagpaare, sondern nur genau eines, das auf dem Tisch liegt; "Herr Schmidt" aus der Aufgabenstellung weiß bloß nicht, ob es sich um das Paar, das aus x=x1, oder das, das aus x=x2 entsteht, handelt, und muß diesbezüglich mehr oder weniger intelligent raten, wobei ihm die Kenntnis von "A" hilfreich sein kann - reingucken ist also auf jeden Fall nicht schlechter als nicht reingucken. (Und im übrigen wäre es hilfreich, für er einen eigenen Beitrag keine neue Nomenklatur zu erfinden, die mit der im beantworten Beitrag nicht kompatibel ist.) Gegen die frequentistische Wahrscheinlichkeitsinterpretation spricht, daß das Spiel gar nicht wiederholbar ist: soll - und täglich grüßt das Murmeltier - Herr Schmidt immer wieder die exakt gleiche Spielsituation vorgesetzt bekommen, und wie ist dann sichergestellt, daß er "zufällig" immer den gleichen der beiden ununterscheidbaren Umschläge zum Nachgucken auswählt und also "rein zufällig" darin immer 100 findet? (Die nicht der Aufgabenstellung entsprechende Doppelschmidtvariante wäre, daß Schmidt1 und sein Klon Schmidt2 am Tisch sitzen und Schmidt1 sich "zufällig" einen Umschlag greift und reinschaut und dann Schmidt2 fragt, ob der vielleicht "zufällig" in den anderen Umschlag hineinschauen mag, und dann beschließen aufgrund ihrer gewonnenen jeweiligen Kenntnis beide, daß sie nun lieber tauschen möchten... Also, das käme mir wirklich paradox vor.) Im übrigen ist bereits Deine o. a. gleichmäßige Verteilung von je 1/4 auf welche Fälle auch immer nicht begründet - wer sagt denn, daß die beiden möglichen Umschlagkombinationen (vor dem Reinschauen hab es nur die eine, nämlich (x,2*x) mit unbekanntem x, nach dem Reinschauen ist die Anzahl auf P1 und P2 angestiegen) zwangsläufig gleichwahrscheinlich sind? Die Anwendbarkeit des Indifferenzprinzips müßte schon plausibel begründet werden; aus der Aufgabenstellung ist sie jedenfalls nicht stringent ableitbar. --95.119.143.151 01:40, 10. Jan. 2023 (CET)
Ohne frequentistische Wahrscheinlichkeitsinterpretation gibt es doch gar keinen Erwartungswert und die darauf aufbauende Tauschempfehlung. Ein Einzelereignis lässt sich nicht mit statistischen Mitteln untersuchen. Dann lässt sich eigentlich gar nichts über den zweiten Umschlag aussagen. Man kann sich aber vorstellen, dass das Ereignis mehrfach stattfindet, von mir aus auch nur mit zwei Umschlägen. Lemke zeigt Schmidt zwei Umschläge. Schmidt weiß nur, dass einer der Umschläge doppelt so viele Euro wie der andere enthält. Er wählt einen Umschlag aus und nennt ihn A, öffnet ihn und findet darin einen Eurobetrag. Anschließend tauscht er ihn dann gegen B. Dann werden die Umschläge wieder verschlossen und gemischt, Schmidt wird geblitzdingst und der Versuch startet neu. A kann entweder der Umschlag mit der kleineren Summe X sein, dann enthält B 2X, oder er ist der Umschlag mit der höheren Summe 2X, dann sind in B X Euro. Wenn die Umschläge gleich sind, müsste Schmidt mit gleicher Wahrscheinlichkeit den einen oder den anderen wählen.
A B nxA
1 X 2X 2,0xA
2 2X X 0,5xA
Summe 3X 3X 2,5xA
Erwartungswert 1,5X 1,5X 1,25xA
Wenn Schmidt den Inhalt von Umschlag B jeweils behalten könnte, würde er im Durchschnitt 1,5X einstreichen, ebenso bei A. Der Erwartungswert beim Tausch ist wieder 1,25 x A.
Solche Gedankenexperimente zur Lösung von Problemen sind ja nichts ungewöhnliches. Man denke nur an Einsteins Züge oder Fahrstühle, die mit halber Lichtgeschwindigkeit fahren usw. --Optimum (Diskussion) 04:01, 10. Jan. 2023 (CET)
Hier steckt der Fehler u. a. darin, daß das Auffinden von x und 2*x - bei unbekanntem x, versteht sich - beim Reinschauen in einen zufällig gewählten Umschlag eben nicht zwangsläufig gleich wahrscheinlich sind: wenn Schmidt z. B. weiß (oder begründet vermutet), daß Lemke höchstens 1000 Euro zu verschenken hat oder verschenken würde, dann kann er sich bei jedem gefundenen Betrag über 500 Euro hinaus sicher sein, daß das 2*x sind und sich in dem anderen Umschlag deshalb nur die Hälfte des gefundenen Betrags befinden kann - wechseln wäre dann dämlich. Und ganz allgemein gibt es auch keinen denknotwendigen Grund, anzunehmen, daß Lemke jedes mögliche x aus seinem "Spendabilitätsspektrum" mit gleicher Wahrscheinlichkeit wählen würde. Vielleicht nimmt er immer nur "x=100" (was das Spiel schon theoretisch unwiederholbar machen würde, denn irgendwann hätte auch der dümmstmögliche Schmidt kapiert, was Sache ist, wenn er beim Reinschauen immer nur entweder 100 oder 200 findet), vielleicht ist sein gewähltes x die Realisierung einer diskreten Zufallsvariablen, deren Verteilungsfunktion sich von Mal zu Mal verändert, vielleicht geht er die Geburtstage oder das Alter seiner Angehörigen durch, wer weiß... Und bei einmaligen Ereignissen ist es auch logisch schwierig, mit theoretischen Häufigkeiten zu argumentieren - auch das Indifferenzprinzip ("weil ich über die tatsächlichen Verhältnisse nichts weiß, muß ich davon ausgehen, daß jedes mögliche Ereignis gleich wahrscheinlich ist") steht auf wackligen Füßen; genauso "natürlich" wie eine Gleichverteilung der möglichen x wäre nämlich beispielsweise eine Benford-Verteilung. --77.1.168.51 07:46, 10. Jan. 2023 (CET)
Hier steckt der Fehler u.a. darin, dass immer neue Randbedingungen hinzu erfunden werden, die in der ursprünglichen Frage gar nicht vorkamen. Vielleicht hat Lemke ein Limit von 1000 Euro. Könnte sein. Vielleicht ist er ein Lügner, und in beiden Umschlägen ist der gleiche Betrag? Oder ein Geizhals und in beiden Umschlägen ist nichts, zweimal nichts ist ebenfalls nichts. Wenn man annimmt, dass es unbekannte Randbedingungen gibt, ist die Frage, ob man wechseln soll, nicht lösbar. Allerdings gibt es dann auch kein Paradoxon. --Optimum (Diskussion) 14:32, 10. Jan. 2023 (CET)
Daß es solche Randbedingungen geben muß und ihre Berücksichtigung für die "richtige" Auswahlentscheidung erforderlich ist, ist keine "Erfindung", sondern denknotwendig. Richtig ist, daß die in der Aufgabenstellung aber nicht konkret spezifiziert und daher unbekannt sind. Was aber kann man aus deren Unkenntnis schließen? Eigentlich nichts, auch das Indifferenzprinzip liefert keinen gültigen Schluß dahingehend, daß die nach Kenntnisnahme eines Umschlaginhalts allein noch möglichen Umschlagpaare P1 und P2 gleichwahrscheinlich sind. Bedeuten die fehlenden Informationen in der Aufgabenstellung nun, daß die Aufgabe deswegen unvollständig, schlecht gestellt und deswegen unlösbar ist? Oder ist die "richtige Lösung" gerade, diesen Sachverhalt zu analysieren und die Defektheit der Aufgabenstellung zu erkennen und zu benennen? Ein Paradoxon entsteht anscheinend dadurch, daß man unter ziemlich natürlichen, aus Nichtwissen ableitbaren Vermutungen p1<2/3 für plausibel halten kann, weswegen Wechseln mutmaßlich lukrativer als Behalten wäre. Dann wäre der möglicherweise paradoxe Fehlschluß nämlich: "Wahrscheinlich sollte ich wechseln, und diese Entscheidung hängt eigentlich gar nicht vom Wissen über den Inhalt des Umschlag ab, denn daß es sich dabei mit weniger als einer Wahrscheinlichkeit von 2/3 um den kleineren Betrag handelt, ist vermutlich so, unabhängig davon, ob ich diesen Betrag nun konkret kenne oder nicht. Und deswegen brauche ich in den gewählten Umschlag eigentlich auch gar nicht hineinzuschauen, sondern kann gleich den anderen nehmen." Solange die Umschläge aber noch beide verschlossen und deswegen zunächst für Schmidt noch ununterscheidbar sind (objektiv vielleicht nicht: wenn man sich zwei Briefumschläge genau ansieht, wird man immer "mikroskopische" Unterschiede - Farbton, Faserverlauf, Anhaftungen, Maßabweichungen - erkennen, die die beiden eindeutig identifizieren. Wenn Lemke sich die gemerkt oder die Umschläge vielleicht auch noch heimlich und für Schmidt unerkennbar markiert hat, kann Lemke sie unterscheiden und wissen, ob Schmidt x oder 2*x gewählt hat, Schmidt aber, ohne hineinzuschauen, nicht, auch dann nicht, wenn er auf die für ihn ununterscheidbaren Umschläge "a" und "b" draufschreiben und sie dadurch für ihn bequem unterscheidbar machen würde (was sie ohnehin sind, weil z. B. a auf b oder links davon liegt). Die äußere Unterscheidbarkeit verhilft ihm ja noch lange nicht zu der bei Lemke vielleicht vorhandenen Kenntnis des Betrags A im Umschlag a, dafür müßte Schmidt schon hineinschauen), ist die Situation symmetrisch, weswegen Wechseln gegenüber Nichtwechseln keinen Unterschied machen sollte. Also ist entweder der Schluß "man sollte immer wechseln" oder "es macht keinen Unterschied" falsch, obwohl es für beide Sichtweisen gute Gründe gibt. Das kann man meiner Ansicht nach schon als paradox bezeichnen. --77.10.59.113 17:35, 11. Jan. 2023 (CET)

Warenumschlag am Flughafen Madrid

In Staffel 3, Folge 13 der Serie Border Control heißt es am Ende von Minute 1, am Flughafen Madrid würden jährlich 520 Mio. Tonnen Waren umgeschlagen. Dies entspräche im Schnitt über 1,4 Mio. Tonnen pro Tag. Aber ist diese Kennzahl tatsächlich realistisch? (Immerhin reden wir hier ja nicht von einem Containerhafen, sondern von Luftfracht …)--2A0A:A540:A7FD:0:405:A5B:5F53:8792 20:18, 10. Jan. 2023 (CET)

Einfach in der Infobox in Flughafen Madrid-Barajas selbst nachlesen. --BlackEyedLion (Diskussion) 20:43, 10. Jan. 2023 (CET)
Die Quelle[3] gibt nur eine Maßzahl, aber keine Einheit her. Da hat wohl jemand Kilogramm vermutet und jemand anders Tonnen. Oder sind da Frachtstücke gemeint? --Rôtkæppchen₆₈ 21:29, 10. Jan. 2023 (CET) Hier ist eine Seite mit Einheitsangabe: 2022 waren es 566.373 Tonnen Fracht. --Rôtkæppchen₆₈ 22:36, 10. Jan. 2023 (CET)
@Rotkaeppchen68: Vielen Dank für die Mühen! Also wie ich vermutet habe: Da haben die Redakteure besagter Serie aber mal so richtig danebengehauen ... Mein lieber Scholli! --2A0A:A540:A7FD:0:C8BF:1F99:3685:4500 01:15, 11. Jan. 2023 (CET)
Ich würde da eher die Fehlerursache beim spanischen Flughafenbetreiber suchen. Der hätte auch hinschreiben können, dass Kilogramm gemeint sind. --Rôtkæppchen₆₈ 07:26, 11. Jan. 2023 (CET)
@Rotkaeppchen68: Na ja, aber schon ein grober Überschlag sollte den aufmerksamen Redakteur doch stutzig werden lassen (wie ja bereits bei mir als Laie geschehen). --2A0A:A540:A7FD:0:DBE:723F:7CE7:5F8C 20:58, 11. Jan. 2023 (CET)
Wenn ich das richtig verstehe, ist die Folge im Original spanisch. Es sollte also auch geprüft werden, ob es sich um einen Übersetzungsfehler handelt. Die Folge gibt es kostenpflichtig hier: https://www.discoveryplus.com/es/show/control-de-fronteras-espana?season=3. --BlackEyedLion (Diskussion) 22:28, 11. Jan. 2023 (CET)

Wellenkammer

Was ist der Zweck der Wellenkammer in Bremerhaven ? Siehe dazu auch hier ein Bild mit Text, aus dem ich leider nicht schlau werde. -- Juergen 217.61.201.10 00:27, 11. Jan. 2023 (CET)

http://www.bremerhaven-today.de/Blog%20Posts/container-terminal-bremerhaven.html: "Während einer Sturmflut schlagen die Wellen in diese Kammer, so dass ihre Wucht gebrochen wird." --BlackEyedLion (Diskussion) 00:55, 11. Jan. 2023 (CET)
Auch der nichtschlaumachende Text schreibt doch Die Wellenkammer ist der Ort, an dem die großen Wogen sterben --Kreuzschnabel 20:17, 11. Jan. 2023 (CET)

Droht Ärger wenn das Kind bei der Einschulung erst die erste MMR-Impfung hat?

Impfgegnereltern können ja die seid März 2020 geltende Masernimpfpflicht in Deutschland hinauszögern in dem das Kind nicht in die Kita geschickt wird. Wenn das Kind mit 6 eingeschult wird, und die Eltern haben die erste MMR-Impfung erst 1 - 2 Wochen vor der Einschulung durchführen lassen, muss die Schule trotzdem das Gesundheitsamt informieren wenn der Masernimpfschutz nicht komplett ist?, aber die 2.500 € Bußgeld werden dann aber bestimmt nicht verhängt wenn die zweite Impfung 2 - 3 Wochen später erfolgt, oder?

--2A00:20:4045:1E46:70B2:3DC3:D7B4:E3C6 10:31, 11. Jan. 2023 (CET)

Das Jugendamt sollte prüfen, ob die Eltern geeignet sind, ihr Kind großzuziehen. Ralf Roletschek (Diskussion) 11:27, 11. Jan. 2023 (CET)
Wie sinnvoll der Hinweis über deiner Nichtantwort ist, offenbart sich spätestens jetzt jedem. Du hast nicht einmal die Frage verstanden. --95.116.66.241 12:09, 11. Jan. 2023 (CET)
Ich denke, er hat eher den Hintergrund der Frage kommentiert. --CC (Diskussion) 12:12, 11. Jan. 2023 (CET)
Dann hat er das Intro oben nicht verstanden. --95.116.66.241
Das lässt sich auch anders interpretieren. Übrigens gilt auch auf dieser Seite das Grundprinzip "Keine persönlichen Angriffe". Bitte halte dich daran. --CC (Diskussion) 12:26, 11. Jan. 2023 (CET)
Wo siehst du einen PA meinerseits? Hier ist nicht die Verbreitung von Meinungen das Ziel. Lies dazu noch oben den Abschnitt "Fragen beantworten". Inwieweit besteht da Interpretationsbedarf? Sein Kommentar ist nicht einmal ein Hinweis auf die Antwort zur Frage, sondern nur seine Meinung zur Notwendigkeit des Impfens an sich (die ich übrigens teile). Dem TO ist die neue gesetzliche Regelung dem Grunde nach klar, es geht hier um ein Detail der rechtlichen Folgen. --95.116.66.241 13:02, 11. Jan. 2023 (CET)
Persönliche Abwertung einer konkreten Person, und das wiederholt. Zu Rechtsfragen sind die hiesigen Mitarbeiter nicht sinnvoll zu befragen, siehe den bereits eingefügten Baustein. Befragen sie bitte zu diesem Thema eine aussagefähige Institution, also nicht die de:WP. EoD meinerseits. --CC (Diskussion) 13:06, 11. Jan. 2023 (CET)
Ich stimme zu, die erste Abwertung einer Person hat jedoch bereits Benutzer:Ralf Roletschek begangen, auf die dann nur reagiert wurde. --BlackEyedLion (Diskussion) 13:10, 11. Jan. 2023 (CET)
Das ist im wesentlichen in § 20 Abs. 8, 9 und 12 des Infektionsschutzgesetzes geregelt. Erst muß man der Schule ausreichenden Impfschutz gegen Masern oder ab der Vollendung des ersten Lebensjahres eine Immunität gegen Masern nachweisen. Macht man das nicht, wird das Gesundheitsamt benachrichtigt. Das fordert dann nochmal den Nachweis. Und wenn der nicht in angemessener Frist erbracht wird, folgen weitere Schritte. 91.54.32.105 13:09, 11. Jan. 2023 (CET)
[…] muss die Schule trotzdem das Gesundheitsamt informieren wenn der Masernimpfschutz nicht komplett ist? Ja, dazu ist die Schule gesetzlich verpflichtet.
[…] aber die 2.500 € Bußgeld werden dann aber bestimmt nicht verhängt wenn die zweite Impfung 2 - 3 Wochen später erfolgt, oder? Das ist eine Ermessensentscheidung des zuständigen Gesundheitsamtes.
S.a. FAQ des BMG zum Masernschutzgesetz, FAQ des BMG für Eltern und Erziehungsberechtigte, FAQ des Deutschen Schulportals. --178.4.177.49 20:51, 11. Jan. 2023 (CET)

Bitte um Hilfe bei Internetsuche bzw. bei Suche im archive.org

Hallo zusammen, mit diesem Direktlink auf das Internetarchiv kommt eine Karte zweifelhafter Qualität, die mal auf den Seiten der Stadtverwaltung von Venedig zu finden war. Auf ihr sind località im damaligen "Quartier 1" abgegrenzt und namentlich in blauer Schrift dargestellt. Die Karte ist in gelber Schrift betitelt mit B. Quartiere Q.1 - San Marco, Castello, S.Elena, Cannaregio. Ich such die entsprechende Karte für das frühere "Quartier 2", die müsste betitelt sein mit B. Quartiere Q.2 - Dorsoduro, San Polo, Santa Croce, Giudecca. Gibt es hier eine Spürnase, die diese karte ausfindig macht?--Ratzer (Diskussion) 12:45, 11. Jan. 2023 (CET)

[4] --95.222.51.213 12:57, 11. Jan. 2023 (CET)
Gibt es auch noch aktuell: https://www.comune.venezia.it/it/content/suddivisioni-amministrative-0 --BlackEyedLion (Diskussion) 13:08, 11. Jan. 2023 (CET)

Besten Dank!--Ratzer (Diskussion) 17:17, 11. Jan. 2023 (CET)

Maskenpflicht

Gibt es eine Statistik die belegt dass die Maskenpflicht geholfen hat Die Grippe zu bekämpfen dann wäre es ja ein Argument für die Beibehaltung der Maskenpflicht (nicht signierter Beitrag von 2001:9E8:3F4E:CC00:701F:301E:5354:D83E (Diskussion) 14:50, 11. Jan. 2023 (CET))

Die Suchmaschine meiner Wahl bietet mir diverse Statistiken bei Eingabe von "statistik nutzen Maske" an, unter anderem diese. --95.116.66.241 15:01, 11. Jan. 2023 (CET)
Medizinische Masken (man achte auf den Namen) werden schon seit langem im medizinischen Bereich eingesetzt um, Überraschung!, die Übertragung von Keimen von Mensch A auf Mensch B zu behindern oder im Idealfall zu unterbinden. Es wurden schon vor 50 und mehr Jahren solche Masken von Personal auf Infektionsstationen getragen und von operierendem Personal, um sich nicht zu infizieren oder um den Patienten nicht zu infizieren. Das alles hat man aber nur getan, weil es so schönes Voodoo ist. Oder als ich auf meinem Gelände eine mit Kalkmörtel gemauerte Wand eingerissen habe, habe ich nur deshalb eine FFP-Maske getragen, weil man darunter so schön geschwitzt hat, man gönnt sich ja sonst nichts. Die FFP-Masken kommen ursprünglich aus dem Arbeitsschutz und man kann beispielsweise hier lesen, welche Masken wofür im Arbeitsschutz genutzt werden. Dort sind diese Masken auch seit Jahrzehnten etabliert. --Elrond (Diskussion) 15:50, 11. Jan. 2023 (CET)
 
AD 1918 in Seattle oder AD 2023 in Stuttgart
In der Covid-19-Pandemie hat man sich auf Maßnahmen besonnen, die schon 1918 während der H1N1-Pandemie geholfen haben, nicht weil sie Schlangenöl waren, sondern weil sie schon damals geholfen haben. --Rôtkæppchen₆₈ 16:00, 11. Jan. 2023 (CET)
Das kommt ein wenig so rüber, als vermutetet ihr hinter dem TO einen Maskengegner. Lt. Frage scheint er aber eher Argumente für eine Maskenpflicht zu suchen. Im übrigen sucht er nach Statistiken. Da ist der Hinweis auf bewährtes Maskentragen zwar sicher sehr hilfreich, aber noch nicht die Frage beantwortet. Kalkpartikel zum Beispiel dürften um ein Vielfaches größer sein, als ein Virus. --2A02:3030:814:9FBA:DF8F:B578:A14F:4646 16:39, 11. Jan. 2023 (CET)
Ich halte den TO für den bekannten Gesellen, der hier immer wieder mal nach Maskenpflicht fragt. --Magnus (Diskussion) 16:52, 11. Jan. 2023 (CET)
Ah, okay; davon weiß ich nichts. --2A02:3030:814:9FBA:DF8F:B578:A14F:4646 16:55, 11. Jan. 2023 (CET)
Nun, ich finde die Expertise von Prof. Dr. Karl Lauterbach sollte hier gehört werden, ist er doch dass Mitglied der Regierung mit dem höchsten akademischen Titel, ferner ist es als Mediziner vom Fach. Herr Prof. dr. Karl Lauterbach ist klar pro Maskenpflicht, und forderte im Herbst auch eine Pflicht in Innenräumen. Leider hat er sich politisch nicht durchsetzen können, wissenschaftlich ist er jedoch klar pro Maskenpflicht
q.e.d. --Magnus (Diskussion) 17:00, 11. Jan. 2023 (CET)
Ironie bitte kennzeichnen. --2003:C1:9709:2D00:ECF3:FEC:4DFC:3882 17:09, 11. Jan. 2023 (CET)
Geht’s ein klein wenig differenzierter als „er ist pro Maskenpflicht“? Wie lange in Kraft, für wen unter welchen Umständen gültig? Es gibt doch noch genug Maskenpflicht in Schland. Herr Lauterbach kann kaum wollen, dass wir alle jahrelang nur noch mit Maske rumlaufen, um die Grippe kleinzuhalten. „Es hat einen Nutzen“ allein ist noch kein Argument für die Einführung einer Verpflichtung, weil du mit dieser Verpflichtung immer andererseits Freiheiten beschneidest, das muss im Verhältnis stehen. Während der Corona-Wellen ging es darum, das Gesundheitssystem nicht an den Rand der Triage zu bringen – ein schwerwiegendes Argument zum befristeten Einschränken einer Freiheit. Welches vergleichbare Risiko siehst du von der zu bekämpfenden Grippe ausgehen? Und immer noch gilt: Niemand verbietet dir, den Rest deiner Tage mit Maske rumzulaufen. Aber wenn du es für alle zur Pflicht machen willst – mit all den nicht so offensichtlichen Nachteilen wie: Körpersprache wird durch eingeschränkte Mimik schwer lesbar; von den Lippen ablesen können hörgeschädigte Menschen gar nicht mehr; kleine Kinder lernen das Lesen von Mimik nur unzureichend, wenn sie nur Maskierten begegnen u.v.a.m. –, dann musst du dafür schon einen richtig guten Grund haben. „Weniger Grippe“ ist keiner, der die Nachteile aufwiegt. --Kreuzschnabel 17:55, 11. Jan. 2023 (CET)
Das Stichwort bei der Maskierung lautet Tröpfcheninfektion.
Der Unterschied ist hier umfassend dokumentiert: https://www.nejm.org/doi/full/10.1056/NEJMc2007800
Das darin enthaltene Video zeigt deutlich den Nutzen einer Maske, bzw. wie infektiöse Körperflüssigkeiten weiträumig verteilt werden wenn keine Maske getragen wird. Inwieweit der Schutz des Lebens bei aggressiven Erregern und in Epidemien/Pandemien gegen Petitessen wie „eingeschränkt lesbare Mimik” aufgewogen werden kann, muß die Politik entscheiden. Das ist eine gesellschaftliche und keine medizinische Fragestellung. Denn einerseits mag es sein, daß kleine Kinder das Lesen von Mimik weniger gut erlernen, wenn sie nur Maskierten begegnen. Aber andererseits sterben eine bestimmte Anzahl von Menschen, wenn sie nur Unmaskierten begegnen. Welches Interesse ist höher zu bewerten? --92.73.145.117 18:25, 11. Jan. 2023 (CET)
Das Problem ist aber, dass dein Immunsystem abschwächt, wenn du ständig Masken trägst. Dann hast du irgendwann mal keine Maske an und liegst danach wegen einer Erkältung 'nen Monat flach. Außerdem gibt es kein Maskenverbot. Nur bei manchen Veranstaltungen ein Vermummungsverbot. Du kannst stets eine Maske tragen. Auch jetzt, wenn du hier in der Wikipedia schreibst. --Christian140 (Diskussion) 18:31, 11. Jan. 2023 (CET)
Dazu hast Du ja sicher einen validen Nachweis, oder wurde das irgendwo gehört, wo das irgendwer irgendwie gesagt hat, --Elrond (Diskussion) 19:52, 11. Jan. 2023 (CET)
Du kannst stets eine Maske tragen. Auch jetzt, wenn du hier in der Wikipedia schreibst.” Bitte erkläre den Sinn dieser Feststellung. Vermutest Du etwa einen Infektionsweg über das Internet? Und was hat das Vermummungsverbot mit der zugrundeliegenden gesellschaftlichen Fragestellung zu tun? Hier geht es eindeutig auch nicht um ein Maskenverbot. Was möchtest Du mit Deiner Antwort hier also konkret aussagen? --92.73.145.117 18:55, 11. Jan. 2023 (CET) P.S. Von einem "ständigen" Tragen von Masken war nicht die Rede, sondern von einem Tragen von Masken in Situationen, in denen eine Ansteckung durch eine hohe Erregerbelastung aufgrund der Anwesenheit vieler Personen auf engem Raum sehr wahrscheinlich ist. Da hilft auch ein noch so trainiertes Immunsystem nicht mehr weiter. Und wenn Du Dir die Bilder ansiehst, verstehst Du, daß es um die vielen Personen geht, die ihre Körperflüssigkeiten im Raum verteilen, nicht um einen einzelnen, dem es freigestellt ist, eine Maske zu tragen. Die Maske dient in erster Linie nicht dem Eigenschutz, sondern dem Schutz der anderen Personen im Raum, ist also nur wirksam, wenn alle sie in solchen Situationen tragen. --92.73.145.117 18:55, 11. Jan. 2023 (CET)
"FFP-Masken fallen unter die Verordnung (EU) 2016/425 über Persönliche Schutzausrüstungen (PSA) und sind gemäß der harmonisierten Norm DIN EN 1491 für den Eigenschutz vor Partikeln (darunter fallen auch Aerosole) ausgelegt. Medizinische Masken fallen unter die Medizinprodukteverordnung (EU) 2017/745 und sind gemäß der harmonisierten Norm DIN EN 146832 mit Fokus auf den Fremdschutz vor Infektionen konstruiert", siehe hier. --95.116.66.241 21:56, 11. Jan. 2023 (CET)
Man darf schon noch ein wenig weiterdenken. Das unbewusste Lesen von Körpersprache ist eine extrem wichtige soziale Kompetenz, die man nicht einfach als Petitesse abtun sollte, nur weil einem gar nicht bewusst ist, welche Perfektion man selbst darin entwickelt hat. Wenn das Kind später auf einen Betrüger/Verführer reinfällt, weil es dessen Mimik nicht richtig deuten konnte, und deshalb Schaden nimmt und schlimmstenfalls stirbt, wie sieht dann deine Balance aus? Übrigens stirbt keiner daran, zu vielen Unmaskierten begegnet zu sein, sondern höchstens an einer Infektion und an fehlenden Intensivpflegeplätzen hinterher, die sich darum kümmern können. Mein hypothetisches Kind ist direkt betroffen (schon der erste Maskierte trägt zu dessen Benachteiligung bei, wie auch jeder weitere), deine hypothetische Anzahl von Menschen nur indirekt (der erste trägt sehr wahrscheinlich noch keinen Nachteil, der zweite auch nicht, nur das statistische Risiko steigt mit der Anzahl). --Kreuzschnabel 20:25, 11. Jan. 2023 (CET)
Wenn das Kind später auf einen Betrüger/Verführer reinfällt, weil es dessen Mimik nicht richtig deuten konnte, und deshalb Schaden nimmt und schlimmstenfalls stirbt, wie sieht dann deine Balance aus?” Ich sehe sehr viel „wenn” und sehr viel „vielleicht” in Deiner Argumentation. Willst Du ernsthaft andeuten, daß Dein hypothetisches Kind zeitlebens vom Tode bedroht ist, weil es die Mimik von Betrügern und Verführern mit Pokerface prinzipiell nicht lesen kann, ganz unabhängig davon, ob es zuvor mal einigen Maskenträgern begegnet ist? Ich sehe es etwas anders: Das frühkindliche Lernen die menschliche Mimik zu lesen und korrekt zu interpretieren, findet schon im engsten Kreis der Familie statt. Dort wird üblicherweise keine Maske getragen, weil und sofern es keinen Grund dafür gibt. Die spätere Begegnung mit Menschen, die in öffentlichen Bereichen Masken tragen, kann diese erworbene Fähigkeit nicht mehr beeinträchtigen.
Mein hypothetisches Kind ist direkt betroffen (schon der erste Maskierte trägt zu dessen Benachteiligung bei, wie auch jeder weitere)” Diese angebliche direkte Benachteiligung müßtest Du schon mit etwas mehr Substanz untermauern. Wenn es zuträfe, bräuchten wir tatsächlich ein Maskenverbot.
deine hypothetische Anzahl von Menschen nur indirekt (der erste trägt sehr wahrscheinlich noch keinen Nachteil, der zweite auch nicht, nur das statistische Risiko steigt mit der Anzahl” Solange Du es nur auf Statistik reduzierst, trifft Deine Bewertung zu. Tatsächlich beinhaltet jedoch jede Infektion mit bestimmten Erregern für jeden Einzelnen die ganz konkrete Möglichkeit, daran zu versterben, selbst bei optimaler intensivmedizinischer Betreuung. Es kann auch schon der erste und zweite sein, der nicht überlebt. Wen es trifft, darüber sagt eine Statistik nichts. Das betrachte ich durchaus als „direkt”, nicht als „indirekt”.
Übrigens stirbt keiner daran, zu vielen Unmaskierten begegnet zu sein, sondern höchstens an einer Infektion
Die Wahrscheinlichkeit, mit solch aggressiven Erregern infiziert zu werden, steht in direkter Relation zur Anzahl von Kontakten mit unmaskierten Trägern dieser Erreger. Also ist es durchaus richtig zu sagen, daß Menschen daran sterben, wenn sie zu vielen Unmaskierten begegnen, von denen niemand sagen kann, ob sie Überträger sind oder nicht. --92.73.145.117 03:15, 12. Jan. 2023 (CET)
Was uns der Bundesgesundheitsminister Prof. Dr. Karl Lauterbach versucht zu vermitteln ist der konsequente Schutz vulnerabler Gruppen. Eine Maske stört im Alltag wenig schützt aber die Kliniken vor der Überlastung, gerade im Winter. Ich finde wir sollten mehr auf seine wissenschaftliche Expertise achten, immerhin ist er der einzige Professor im Kabinett. (nicht signierter Beitrag von 87.190.10.211 (Diskussion) 20:42, 11. Jan. 2023 (CET))
Wie gesagt, Ironie bitte kennzeichnen. Lauterbach hat nicht als Arzt gearbeitet. Sein Professorentitel suggeriert Expertise (in Medizin) - die hat er nicht. Ob die Regierungen anderer Länder - zB Schweden - sooo falsch liegen? --2003:C1:9709:2D00:ECF3:FEC:4DFC:3882 21:40, 11. Jan. 2023 (CET)
Er ist Epidemiologe und hat mithin genau die richtige Expertise (und hat natürlich in seinem Leben zumindest während des und nach dem Studium als Arzt gearbeitet - hältst Du Medizinprofessoren etwa für doof/weltfremd/unbedarft?), und daß die schwedische und andere Regierungen eine zynische menschenverachtende Politik mit Vorrang für ökonomische Interessen des Kapitals vor dem Gesundheits- und Lebensschutz praktiziert haben, liegt durchaus im Bereich des Vorstellbaren. Aufschlußreich ist doch schon die verräterische Aussage, die Schutzmaßnahmen, speziell die Maskenpflicht, wären deswegen nicht "verhältnismäßig" (im Verhältnis zu was?), weil die "kritische Infrastruktur" nicht gefährdet, sprich die IS aufgrund der Impfungen nicht überlastet wären - es ging den wirtschaftshörigen Politikern also von Anfang an nie darum, den Einzelnen wirksam vor Infektionen zu schützen - die Impfungen tun das bekanntlich nicht - oder gar, was vernünftig und möglich gewesen wäre, das Virus im Menschen auszurotten, sondern die Infizierten sollten nur bitteschön nicht die Krankenhäuser verstopfen. Es war also von Anfang an auf eine Durchseuchungsstrategie angelegt, nur bitte nicht alle auf einmal - flatten the curve - sondern schön hinten anstellen, und jeder nur ein Kreuz... Dumm nur, daß das mit der Herdenimmunität nach der Durchgang der "Welle" nicht hinhaut - weder eine durchgemachte Infektion noch die Impfungen bewirken eine bleibende Immunität, und auch ohne volle Friedhöfe akkumulieren sich die Fälle von Long COVID mit langfristig bleibenden Schäden bis hin zur Invalidität. Ich fürchte, der Zug ist auch abgefahren, nachdem nun auch noch China die Weiße Fahne rausgehängt und Freund Hein freie Bahn gemacht hat. --77.3.83.252 11:16, 12. Jan. 2023 (CET)
Hm, siehe allerdings auch: Unstatistik des Monats: "Mega" - Studie zum Maskentragen hat viele Mängel, wobei der Nutzen der Masken allerdings wohl nicht bestritten wird, sondern die Methodik der Studie. Sowas verwirrt Menschen möglicherweise. --2A02:3030:814:9FBA:DF8F:B578:A14F:4646 20:26, 11. Jan. 2023 (CET)
Um diese Frage zu untersuchen bräuchte man ein großes Lager mit 1000 zufällig ausgesuchten Personen, von denn 500 immer eine Maske tragen und 500 keine Maske tragen. Dann würde man etwas über den Eigenschutz herausfinden. In einem zweiten Lager bräuchte man noch einmal 500 Leute ohne Maske, die getrennt von den anderen leben. Dadurch bekäme man eine Information zum Fremdschutz von Masken. Alle anderen Statistiken sind Scharlatanerie. --2001:9E8:A504:1900:2B3D:E4A8:7A89:5E44 08:54, 12. Jan. 2023 (CET)

für Schulden des Partners haften ?

Mein Freund möchte sich selbstständig machen, er hat die Möglichkeit eine Bubble tea bar zu übernehmen, ihm fehlt aber das Eigenkapital weil er auch vorbestraft ist. Er hat mich gebeten für die Schulden zu bürgen, ich würde es gerne machen aber muss ich dann auch wenn die Bar pleite geht zahlen ? Es geht um 50.000,00 €, 15.000,00 € könnte ich sofort finanzieren wenn ich meine private Rentenversicherung auflöse. (nicht signierter Beitrag von 2001:9E8:3F5F:5100:6469:C5C0:D436:8703 (Diskussion) 09:38, 14. Jan. 2023 (CET))

Ist auf keinen Fall zu empfehlen. Als Bürge haftest du quasi für alles, denn das ist ja der Sinn der Bürgschaft. Und soweit ich weiß, darf man mit einer Vorstrafe auch keinen lebensmittelverabeitenden Betrieb betrieben. --91.114.184.118 09:45, 14. Jan. 2023 (CET)
Bitte nicht antworten. Entweder Troll(in) oder hoffnungsloser Fall. --Magnus (Diskussion) 10:01, 14. Jan. 2023 (CET)
Oh, ist er mal wieder draußen und will sich jetzt aber wirklich ganz bestimmt total ändern? Nun, „für die Schulden bürgen“ besteht mehr oder weniger darin, dass man sagt „wenn der zahlungsunfähig ist, dann zahle ich alles“, daher wirst du das wohl müssen, ja. Dein Freund könnte also die Bude an die Wand fahren und dann mit dem Finger auf dich zeigen, und du hast die A-Karte. Bürgen heißt, dass du deinen Kopf im Ernstfall für ihn in die Schlinge steckst, deshalb überlegt man es sich sehr sehr sehr sehr sehr gut, für wen man bürgt. Lesetipp: Die Bürgschaft. --Kreuzschnabel 10:10, 14. Jan. 2023 (CET)

Die Frage ist mit "Ja" imho ausreichend beantwortet. --88.72.111.240 11:55, 14. Jan. 2023 (CET)

Archivierung dieses Abschnittes wurde gewünscht von: 88.72.111.240 11:55, 14. Jan. 2023 (CET)

Gendern wenn das Stammwort bereits weiblich ist.

Wie geändert man korrekt,wenn das Stammwort bereist weiblich ist?

Bsp: Witwe, Hexe, Hebamme,

--2003:6:210F:1C45:A0B6:C257:E860:1325 11:26, 9. Jan. 2023 (CET)

Witwer, Hexer, männliche Hebamme... --CC (Diskussion) 11:28, 9. Jan. 2023 (CET)
Letztere heißen offiziell Entbindungspfleger. – Es gäbe da noch generische Feminina wie Waise oder Geisel, die hab ich noch nicht gegendert gesehen. --Kreuzschnabel 11:30, 9. Jan. 2023 (CET)
Das mag dereinst Mal so gewesen sein, heute heiß es für Frauen wie für Männer Hebamme Gesetz über das Studium und den Beruf von Hebammen (Hebammengesetz - HebG) § 3 Berufsbezeichnung [...] (2) Die Berufsbezeichnung „Hebamme“ gilt für alle Berufsangehörigen. --Elrond (Diskussion) 15:16, 10. Jan. 2023 (CET)

Ja wie ist es aber wenn man beide Geschlechter ansprechen will. Gibt es unter euch eine*n Witwe*r? Lässt man dann eine Pause beim Sprechen? --2003:6:210F:1C45:A0B6:C257:E860:1325 11:43, 9. Jan. 2023 (CET)

Es gibt jedenfalls Leitfäden, die das Beispiel sehr geehrte*r enthalten (ohne Aussprachehinweis). Hebamme bezeichnet nach meinem Verständnis gleichermaßen Menschen jeden Geschlechts, wie zum Beispiel Person ja auch. --BlackEyedLion (Diskussion) 11:45, 9. Jan. 2023 (CET)
Wichtig ist der*die Pause*R beim Lesen! [Hier nach Belieben Grinsekopf*In einsetzen] --95.119.143.151 11:58, 9. Jan. 2023 (CET)
Wenn du eine Rede bei einer Versammlung von Witwen und Witwern eine Rede hältst, würde ich beide ansprechen, in dem ich sie begrüße mit: "Sehr geehrte Witwen und Witwern, herzlich willkommen in der Wikipedia!" --Christian140 (Diskussion) 14:16, 9. Jan. 2023 (CET)
Richtiges Gendern? Ist das nicht schon ein Widerspruch in sich? Rolz Reus (Diskussion) 12:11, 9. Jan. 2023 (CET)
Nein, ist es nicht. Nur weil die meisten Menschen falsch gendern, heißt das nicht, dass es kein richtiges Gendern gibt. Es braucht halt mehr Zeit, Papier, Druckerschwärze und Speicherplatz. --Rôtkæppchen₆₈ 14:54, 9. Jan. 2023 (CET)
Da Gendern an sich schon falsch ist, kann es kein "richtiges" Gendern geben. (Und was heißt nun "richtig" und "falsch"? "Richtig" ist die dem üblichen Sprachgebrauch folgende Vorgehensweise, und in dem wird nicht gegendert, das macht im Alltag kein normaler Mensch, höchstens den obrigkeitlich aufgestülpten Sprachgebrauch ironisierend. Der Zwang zum Gendern und die dahingehende Propaganda, z. B. durch den penetranten Gebrauch in den Medien, stellt eine kulturelle Okkupation und Bevormundung durch eine besserwisserische ideologisierte Minderheit, die sich aufgrund eines selbstverliehenen Expertentums eine objektiv und demokratisch nicht begründete Regelungskompetenz anmaßt, dar. Deren zynische und möglicherweise sogar erfolgreiche Strategie besteht darin, Dissidenten solange mundtot zu machen, bis die verbleibende Mehrheit glaubt, daß schwarz wirklich weiß ist. (Beispiele dafür, insbesondere in der Sprache, gibt es inzwischen leider viele, z. B. das Neger- und Hexenverbot, die Rechtschreibreform, die Leichtersprache.) Solche ideologische Umerziehung durch Propaganda wurde auch schon vielfach beschrieben und analysiert, beispielsweise literarisch in 'Animals Farm' oder "1984". Daß das so ist, ist für selbständig denkfähige Menschen völlig unverkennbar. Die Frage ist eigentlich nur, warum "die" das eigentlich machen. Eine mögliche Erklärung wäre Dummheit oder Bosheit. Die noch mildeste Beurteilung wäre Verblendung: Die "Täter" glauben tatsächlich, durch ihr Vorgehen etwas Gutes bewirken zu können, und handeln deswegen aus einem pädagogischen Impetus heraus. Das eigentlich Beunruhigende ist, daß der Mechanismus so gut funktioniert und man mit diesen Methoden auch völkerrechtswidrige Angriffskriege, Heidenmissionierungen, Hexenverfolgungen, Kolonialisierung, Völkermord, Kreuzzüge, Geschlechterdiskriminierung, systematische Folter und die Einrichtung von Konzentrations- und Vernichtungslagern, manche würden vielleicht sagen: auch die Devastation von Siedlungen zum Abbau darunterliegender Bodenschätze, propagandistisch rechtfertigen und befördern kann. - Wenn es Kritik oder Widerstand gibt, haben die Propagandisten einfach nur schlecht gearbeitet.) --77.10.59.113 18:12, 11. Jan. 2023 (CET)
Auch wenn Du es wahrscheinlich nicht wahr haben willst, Sprache ist weder etwas absolutes, noch ist sie statisch. Die Nutzung beider Generika bzw. einer Neutralform im Alltagssprechen (Die Studentinnen und Studenten haben ihre Prüfungen mit Bravour bestanden, alternativ: Die Studierenden....) hat sich weitestgehende etabliert, auch wenn eher konservative Kreise das bestreiten oder bejammern. Das mit dem Binnenirgendwas wird sich wahrscheinlich nicht durchsetzen. Meist entwickelt sich die Umgangssprache auch nicht nach Vorgaben, von wem auch immer, sondern durch schlichten Gebrauch. Auch da gibt es Entwicklungen die mich grausen (z.B. weil/denn-Verhunzung), und andere, die ich begrüße (z.B. Nutzung beider Generika oder Neutralform). Beeinflussen werden wir das wohl nur in einem sehr begrenzten Maß können. Wie "Deutsch" dereinst klang magst Du bei der Lektüren des Hildebrandslied sehen, aber das wirst Du sicher kennen. Aber auch schon die "gehobene" Sprache des frühen 19. Jahrhunderts ist nichts mehr, was man heute mit "normalem" Deutsch in Verbindung bringt. --Elrond (Diskussion) 11:45, 12. Jan. 2023 (CET)
Und wenn sie divers sind? --2003:D5:FF4C:5500:BCCC:C7E3:5D7A:C1CD 14:57, 9. Jan. 2023 (CET)
Es gibt noch kein grammatisches Geschlecht divers. Stattdessen das „übrige“ Geschlecht Neutrum zu nehmen ist definitiv falsch. --Rôtkæppchen₆₈ 16:28, 9. Jan. 2023 (CET)
Warum ist das Neutrum eigentlich definitiv falsch? Warum höre ich immer wieder, es könne nicht als Oberbegriff für Personen verwendet werden. --2A0A:A540:A386:0:95C2:B71C:281D:4F20 21:08, 9. Jan. 2023 (CET)
Kein Mensch ist sächlich. --Rôtkæppchen₆₈ 22:02, 9. Jan. 2023 (CET)
Kein menschliches Wesen, auch kein Kind? --MannMaus (Diskussion) 17:22, 12. Jan. 2023 (CET)
Ganz einfach: neuter heißt „keines von beiden“ ( uter heißt „beide“; ne-uter ist die Verneinung davon). Das neutrale Geschlecht bezeichnet also Sachen, die weder männlich noch weiblich sind, und ist damit für Personen denkbar ungeeignet. Daher die Forderung nach einem Utrum, das es bis jetzt aber in der deutschen Grammatik nicht gibt. --Kreuzschnabel 23:25, 9. Jan. 2023 (CET)
Eben, weder männlich noch weiblich, keines von beidem, also genau das, was mit divers gemeint ist, oder irre ich mich da? Ich frage mich, wie du ein Utrum im Deutschen einführen willst. Anstelle von Femininum und Maskulinum? Daneben wäre mir keine Sprache bekannt, es sei denn, man zähle die Personalpronomen im Dänischen und Schwedischen als Maskulinum und Femininum neben dem Utrum. Oder willst du ausschließlich ein Utrum haben? Sollte das dann so geschaffen werden, wie es im Dänischen und Schwedischen und weitgehend auch im Nordniederländischen entstanden ist, also durch Aufgabe/Abschaffung des Femininums? --2A0A:A540:A386:0:2929:77D7:9B3F:F507 01:31, 10. Jan. 2023 (CET)
Auch diversgeschlechtliche Menschen sind keine Sachen, sondern Menschen. Neutrum ist für sie also unpassend. Neutrum wird in Deutschen nur für Sachen oder den Diminutiv und wenige andere Ausnahmen (Kind, Weib…) verwendet. --Rôtkæppchen₆₈ 01:44, 10. Jan. 2023 (CET)
Wenn das Neutrum zwar diverse Identitäten anspricht, aber dafür alle binären Personen (also fast alle überhaupt) ausschließt, ist das noch nicht so ganz der Stein des Kolumbus. – Wo genau habe ich gesagt, dass ich ein Utrum im Deutschen einführen will? Ich habe lediglich erklärt, wieso ein Neutrum kein Ersatz dafür sein kann. --Kreuzschnabel 01:45, 10. Jan. 2023 (CET)
Wobei der korrekte Vokativ latürnich „Sehr geehrte Witwen und Witwer“ lautet. Nurso. --Kreuzschnabel 16:04, 9. Jan. 2023 (CET)
Das SGB VI nennt diese Menschen Hinterbliebene. --77.1.151.72 16:59, 9. Jan. 2023 (CET)
Was aber nicht dem üblichen Sprachgebrauch entspricht (nach dem auch andere nahen Verwandten des/der Verstorbenen Hinterbliebene sind). --Digamma (Diskussion) 18:07, 9. Jan. 2023 (CET)
Was nichts daran ändert, dass die Ansprache bei der o. g. Versammlung so bei Witwe[r]n korrekt wäre und sich auch jede/r angesprochen fühlen wird. --77.1.151.72 19:08, 9. Jan. 2023 (CET)
Liebe Verwitwete, sehr geehrte Geburtshilfekräfte! --Rôtkæppchen₆₈ 19:23, 9. Jan. 2023 (CET)

Jedoch ganz wichtig: "Drucker:innenschwärze!"--Hopman44 (Diskussion) 19:29, 9. Jan. 2023 (CET)

Im Allgemeinen spricht man von Druckfarbe. Einen Text mit dem Wort „Druckerschwärze“ ordne ich automatisch irnkwann vor 1960 ein. --Kreuzschnabel 19:33, 9. Jan. 2023 (CET)
Im Französischen ist der Drucker - wie auch der Kopierer - übrigens weiblich (une imprimante). --Digamma (Diskussion) 15:46, 10. Jan. 2023 (CET)
Was im Kontext nur den Schönheizfehler hat, dass sich die -schwärze nicht vom Gerät herleitet, sondern vom Beruf. --Kreuzschnabel 23:15, 10. Jan. 2023 (CET)

Nach Phettberg: das Witwy, das Hexy, das Hebammy. Klingt arg lustig - rein logisch betrachtet ist das Konzept aber gar nicht mal so übel... --2A01:C23:69F7:ED00:C009:BC7F:A7B5:1EBA 21:25, 9. Jan. 2023 (CET)

Übel daran ist die Verwendung des Neutrums. Neutrum ist kein Utrum-Ersatz. Teilweise wird es auf Menschen angewandt sogar als menschenverachtend betrachtet, insbesondere bei diversgeschlechtigen Menschen. --Rôtkæppchen₆₈ 22:01, 9. Jan. 2023 (CET)
Das Neutrum wurde früher aber durchaus so verwendet. --Digamma (Diskussion) 15:48, 10. Jan. 2023 (CET)
Mitglied. --Sokkok (Diskussion) 02:09, 11. Jan. 2023 (CET)
Nochmal von vorn: Witwer ist mittlerweile ein ungebräuchliches Wort? Ich bin alt. Grüße Dumbox (Diskussion) 22:08, 9. Jan. 2023 (CET)
Natürlich ist das kein ungebräuchliches Wort. Wo steht denn so was in der bisherigen Diskussion? --Kreuzschnabel 23:13, 9. Jan. 2023 (CET)
Und wie gendert man "Bauer"? --Expressis verbis (Diskussion) 23:30, 9. Jan. 2023 (CET)
Landwirt:in. HTH --Kreuzschnabel 23:34, 9. Jan. 2023 (CET)
Man darf nicht mehr Bauer sagen? Sprachpolizei. --Expressis verbis (Diskussion) 23:40, 9. Jan. 2023 (CET)
Sie haben „Bauer“ gesagt. Sind Sie mit einer einstweiligen Erschießung einverstanden? --Kreuzschnabel 00:16, 10. Jan. 2023 (CET)
Nur wenn sich auch Diverse unter den Soldatenden befinden. --Expressis verbis (Diskussion) 15:54, 10. Jan. 2023 (CET)
Muss das nicht "Soldierte" heißen? --Digamma (Diskussion) 16:27, 10. Jan. 2023 (CET)
Unter den Soldbeziehenden. --MannMaus (Diskussion) 17:20, 12. Jan. 2023 (CET)

Nachdem ich mein Milchfläschchen ausgetrunken hatte, sagte meine Mutter immer zu mir: "Und jetzt mach ein kleines Bäuerchen"--Hopman44 (Diskussion) 09:15, 10. Jan. 2023 (CET)

was ist mit der Herrin des Hundes? --2001:9E8:A513:4800:2AD3:580F:AA90:D113 10:07, 10. Jan. 2023 (CET)
Das führt jetzt sehr weit vom Thema ab, aber ich frage mich tatsächlich öfter, was der analoge Ausdruck für die Rülpser meines Hundes wäre – Beispiel: Jetzt mach noch einen Hofhundwelpen, und dann ab aufs Sofa --Kreuzschnabel 12:43, 10. Jan. 2023 (CET)
Der Begriff lautet selbstverständlich canine Eruktation bei Hündinnen und caniner Ruktus bei Rüden. --Rôtkæppchen₆₈ 20:13, 10. Jan. 2023 (CET)

Taschengeldverdienst für Kinder geduldeter Asylbewerber

Guten Morgen allerseits! Wie gerne habe ich in meiner inzwischen weit zurückliegenden Jugendzeit das damals karge Taschengeld durch kleine Aushilfsarbeiten aufgebessert: Rasenmähen bei der betagten Nachbarin, Reinigung von Windschutzscheiben bei der Tankstelle nebenan, Austragen von Prospekten, etc etc. Vor ein paar Tagen sprachen mich Schüler (14, 18 J), Kinder geduldeter Asylbewerber, an, ob sie mir und meiner Frau nicht ein wenig helfen könnten. Meine Frage: Geht das mit dem "Verdienst" auch hier so unkompliziert wie früher? Was ist zu beachten?? Verdienstobergrenzen??? Grüße, 178.142.230.89 08:15, 10. Jan. 2023 (CET)

Die Obergrenze ist wohl 100 Euro/Monat. https://praxistipps.chip.de/mit-14-geld-verdienen-so-bessern-sie-ihr-taschengeld-auf_144434 Ob die Asylbewerbung ein Rolle spielt, weiß ich nicht. --Doc Schneyder Disk. 16:01, 10. Jan. 2023 (CET)
In dieser Frage geht es aber um Kinder, also nicht um regulaere Arbeitskraefte. -- Juergen 217.61.201.10 18:12, 10. Jan. 2023 (CET)
Der Fragesteller hat kein Land geschrieben. Woher weisst Du, welches Land gemeint ist ? -- Juergen 217.61.201.10 20:06, 10. Jan. 2023 (CET)
Wieso rückst du eigentlich deine Beiträge gleich um Lichtjahre ein? Normal nimmt man einfach einen Doppelpunkt mehr als der Bezugsbeitrag. --Kreuzschnabel 23:11, 10. Jan. 2023 (CET)
Das war nicht ich, sondern Benutzer:GregorHelms, der meine Beitraege ihres Kontexts beraubt und damit entstellt hat. Die Rueckgaengig-Funktion geht jetzt nicht mehr ... -- Juergen 217.61.201.10 00:43, 11. Jan. 2023 (CET)
Du hast recht, sorry für meinen Anwurf. Ich habe den Benutzer auf seiner Disk darauf hingewiesen, dass das nicht die Idee des Jahres war :) --Kreuzschnabel 07:11, 11. Jan. 2023 (CET)
Der Fragesteller nutzt eine IP-Adresse aus Deutschland. Da ist es denkbar bis wahrscheinlich, dass ihn die Situation in Deutschland  Deutschland interessiert. --Rôtkæppchen₆₈ 22:55, 10. Jan. 2023 (CET)
Es ging um die Herkunft der Kinder, nicht deren Aufenthaltsort. Siehe hier den von Benutzer:GregorHelms geloeschten Originalkontext zu meinem Beitrag. -- Juergen 217.61.201.10 00:46, 11. Jan. 2023 (CET)

Oh, oh. Will man 50,-- € mal bar an einer Kasse einer Bank für jemand Anderen (z.B. Sohnemann) zu einer anderen Bank einzahlen, bei der er ein Konto hat, gerät man schon unter den (General-)Verdacht der Geldwäsche oder dass es Schwarzgeld ist. E tännschen plies.!--Hopman44 (Diskussion) 10:27, 11. Jan. 2023 (CET)

Sagt wer? Ist das Hörensagen oder erlebte Realität. Wenn ja, mit welcher Begründung?--scif (Diskussion) 18:45, 11. Jan. 2023 (CET)

Ist doch nix anderes, als wenn du deinem deutschen Nachbarskind zehn Euro fürs Babysitten oder Katzensitten zusteckst, oder? Wo soll der Unterschied sein? Bei Sozialhilfeempfängern (egal welcher Staatsangehörigkeit) gibt es irgendwelche Obergrenzen des erlaubten Zuverdienstes. Die eigentliche Gefahr scheint mir eher darin zu bestehen, wenn du einen ausländerfeindlichen Blockwart als Nachbarn hast, der dich wegen Beschäftigung von Schwarzarbeitern anzeigt. --87.150.10.252 22:55, 11. Jan. 2023 (CET)

Egal ob die Eltern jetzt Grundsicherungsempfänger sind oder nicht, empfiehlt es sich in solchen Fällen immer in Naturalien zu zahlen. Das müssen die Eltern nicht beim Sozialamt anzeigen, wird nicht auf die Grundsicherung angerechnet und Geldwäscheverdacht gibt es auch keinen. --84.172.240.50 23:01, 11. Jan. 2023 (CET)
Tolle Idee. Gib den Schülern ein paar gesunde Äpfel und Möhren, und sie werden mit Begeisterung dafür deinen Rasen mähen.
Abgesehen davon: Soweit ich weiß, wäre das genaugenommen ein geldwerter Vorteil. Dann kannst du auch gleich Bargeld geben. --87.150.10.252 01:41, 12. Jan. 2023 (CET)
Das kommt ganz auf die Schüler an... Wir haben das gemacht, sogar gerne, und durften als „Gegenleistung” nach und nach den ganzen großen Kirschbaum leerfuttern, Kuchen und Kakao gab es auch immer, wenn wir vorbeikamen. Die Frage ist wohl, ob es von den Beteiligten als Geschäftsbeziehung oder als freundschaftliche Nachbarschaft wahrgenommen wird. --92.73.145.117 03:42, 12. Jan. 2023 (CET)
Ja, damals, vor'm Krieg, da war'n wir auch dankbar, wenn uns jemand einen verschrumpelten Apfel geschenkt hat... --87.150.9.111 13:53, 12. Jan. 2023 (CET)
Wirklich? Vor dem Krieg? Ein verschrumpelter Apfel? Wohl kaum. Meinst Du vielleicht "während des Krieges" oder "direkt nach dem Krieg"? Das wäre eher plausibel. Aber was hat das Schenken nun damit zu tun? Hier geht es doch um Geben und Nehmen... Wie auch immer, so lange ist es noch nicht her, und wer in den letzten Jahren im Supermarkt mal auf die Preise von frischen Kirschen geschaut hat, selbst zur Reifezeit, dem muß klar sein, daß wir dabei auch unter pekuniärem Aspekt keinen schlechten Schnitt gemacht haben. --92.73.145.117 16:51, 12. Jan. 2023 (CET)
Der kleine harmlose Sarkasmus war eigentlich dazu angetan gewesen, dich mit der Nase darauf zu stoßen, dass das am Thema vorbeigeht. Diese Schüler wollen nicht Kirschen oder Kuchen, die wollen eine Taschengeldaufbesserung. Bargeld.
Abgesehen davon geht deine Rechnung nicht ganz auf. Der Preis von Kirschen bemisst sich zwar zum einen nach Angebot und Nachfrage, zum anderen aber auch durch die recht aufwendige nichtmaschinelle Ernte. Die Handpflückung habt ihr in dem Fall ja selbst erledigt. Diese Arbeitskosten musst du auf der pekuniären Seite abziehen. --87.150.9.111 18:25, 12. Jan. 2023 (CET)
Es geht tatsächlich nicht am Thema vorbei, sondern trifft den Kern. Die Fagestellung war: »Geht das mit dem "Verdienst" auch hier so unkompliziert wie früher? Was ist zu beachten?? Verdienstobergrenzen???« Und genau auf den Unterschied zwischen einer Geschäftsbeziehung, wo 1:1 abgerechnet wird, und einer gut funktionierenden Nachbarschaft, bei der es keinem der Beteiligten darauf ankommt, ob die "Kasse stimmt", wenn solche gegenseitigen "Dienstleistungen" erbracht werden, zielte meine Antwort ab. Wir haben die Kirschen, die wir gefuttert haben, ja auch nicht gewogen, die Kuchenstücke nie gezählt, nicht gefragt, wieviele Quadratmeter Wiese wir da gemäht haben. Aufwendige nichtmaschinelle Ernte? Das Herumklettern in dem riesigen alten Kirschbaum war doch das Beste an der Sache. Und die Schüssel voll Kirschen, die wir ganz selbstverständlich dem älteren Nachbarsehepaar mit heruntergebracht haben, hatte sich ein paar Tage später in einen leckeren Kirschkuchen für uns verwandelt. In so einem Rahmen spricht dann auch nichts dagegen, den Nachbarskindern gelegentlich das Taschengeld aufzubessern, das gab es bei uns auch. Aber es darf halt keine systematische Geschäftsbeziehung daraus werden, womöglich noch mit Preisliste: 1x Rasenmähen € 10,--, 1 x Hecke schneiden € 10,--, 1 x Laub im Hof fegen € 5,--, 1 x Gartenzaun streichen ... u.s.w., das geht legal nicht und ging auch früher nicht, soweit ich weiß. Dafür braucht es heute zumindest eine Anmeldung des "Arbeitnehmers" als "Haushaltshilfe", mit genauer Tätigkeitsbeschreibung und Angabe der Arbeitszeiten, und Du wirst damit automatisch zum "Arbeitgeber", zum Unternehmer, mit allen Pflichten, und bist für die Anmeldung verantwortlich. Das gilt auch, wenn Du die Anmeldung pflichtwidrig unterläßt. Was ich skizziert habe, ist genau der Unterschied zwischen erlaubter nachbarschaftlicher Hilfe und einer Geschäftsbeziehung, also der Unterschied, nach dem hier gefragt wurde. Details siehe Haushaltsnahes Beschäftigungsverhältnis und Nachbarschaftshilfe#Deutschland. Dabei ist es auch unerheblich, ob es sich um einheimische Kinder oder "geduldete" Flüchtlingskinder handelt. Ob die Schüler zu einem Geben und Nehmen in diesem freundschaftlich-nachbarschaftlichen Rahmen bereit sind, oder ob es ihnen tatsächlich in Wahrheit um eine verkappte Anstellung als Schwarzarbeiter geht, kommt tatsächlich auf die Schüler an. --92.73.145.117 23:25, 12. Jan. 2023 (CET)

Freies Wissen aus Forschung und Lehre (Universität)

Haben "wir da schon was vorbereitet"? Gefunden habe ich via WM.de: Freies Wissen und Wissenschaft (2015): Massive Open Online Course und Open Access. Kategorie:Freies_Wissen hat 34 Einträge, aber nicht Universitäres (Open Educational Resources oder Wissensallmende sind zu unspezifisch). Nicht in der Kategorie war Offene Wissenschaft - aber der Artikel ist eine Schande für WP (und für die Wissenschaft)... Gruss, --Markus (Diskussion) 11:43, 10. Jan. 2023 (CET)

Was genau willst Du wissen? Deine Anfrage ist sehr kryptisch. --178.4.177.49 02:51, 11. Jan. 2023 (CET)
Ich möchte gern wissen, welche Artikel wir zum Thema "Freies Wissen aus Forschung und Lehre" haben, bezogen auf Universität. (Der kryptische Teil ist nur die Zusammenfassung der verwirrenden Ergebnisse meiner Suche.) Gruss, --Markus (Diskussion) 19:30, 11. Jan. 2023 (CET)
Geht es Dir um das Urheberrecht in der Wissenschaft oder worum geht es Dir bei Deinem noch immer sehr unscharf formuliertes Anliegen? Hast Du schon in Kategorie:Wissenschaftspraxis und Kategorie:Wissenschaft und Recht nachgesehen, ob da etwas für Dich dabei ist? --178.4.177.49 21:18, 11. Jan. 2023 (CET)
Danke für den ersten Link, der zeigt übersichtlich die Vielfalt der Möglichkeiten. Davon sind in unserer Wissenschaft und Lehre die einschränkenden Varianten sehr verbreitet, und die offenen eher beschränkt. Auch in unseren Artikeln sind offene nicht so sehr beschrieben. Laut WMDE wäre Freies Wissen ein wertvolles Ziel. Deshalb suche ich, was wir denn schon dazu haben? und mittelfristig Wege, wie wir das ausbauen können. Ich halte Freies Wissen und Freie Lehre für sehr relevant für Gegenwart und Zukunft, weltweit und für die Demokratie. Gruss, --Markus (Diskussion) 16:45, 12. Jan. 2023 (CET)

Wirtschaftlicher Betrieb beim Verbrennungsmotor

Unter Elastizität (Kraftfahrzeugtechnik) steht, der elastische Bereich zwischen hoechstem Drehmoment und hoechster Leistung sei gleichzeitig auch der wirtschaftliche Fahrbereich.
Ist es aber nicht vielmehr so, dass man am wirtschaftlichsten faehrt, wenn man jeweils die niedrigste Drehzahl waehlt, bei der der Motor noch die aktuell benoetigte Leistung liefern kann, also immer den hoechstmoeglichen Gang ? Die sich dabei einstellende Drehzahl liegt dann aber meist (also bei Geradeausfahrt mit Geschwindigkeitsbegrenzung) weit unter derjenigen des hoechsten Drehmoments, also auch weit ausserhalb des hier behaupteten wirtschaftlichen Fahrbereiches. -- Juergen 217.61.201.10 18:10, 10. Jan. 2023 (CET)

Ohne Quelle: Der Verbrauch ist am geringsten im höchsten Gang bei der dann niedrigsten noch gut fahrbaren Drehzahl. Ich nehme aber an, dass dann dafür der Verschleiß größer ist. --BlackEyedLion (Diskussion) 20:03, 10. Jan. 2023 (CET)
Dann stimmst Du also mit mir ueberein, dass die zitierte Aussage aus dem Artikel falsch ist ?
Oder handelt es sich vielleicht um ein Missverstaendnis/Fehlinterpretation ? Die angegebene Literatur ist leider nicht online. -- Juergen 217.61.201.10 20:07, 10. Jan. 2023 (CET)
Im Artikel wird nicht erklärt, was die Bezeichnung wirtschaftlicher Bereich bedeuten soll. --BlackEyedLion (Diskussion) 20:15, 10. Jan. 2023 (CET)
Zitat: "Je elastischer ein Motor, desto größer ist der wirtschaftliche Fahrbereich, auch elastischer Bereich genannt, der zwischen maximalem Drehmoment und maximaler Leistung liegt." Interpretierst Du das anders als ich hier oben in der Frage ?
Uebrigens koennte ich mir durchaus vorstellen, dass beim hoechstmoeglichen Gang der Verschleiss kleiner ist als oberhalb der Drehzahl des maximalen Drehmoments, weil der Verschleiss eben zu einem erheblichen Teil von der Anzahl der Motorumdrehungen abhaengt und weniger von der Belastung. Wissen habe ich darueber aber nicht. -- Juergen 217.61.201.10 20:27, 10. Jan. 2023 (CET)
Ich weiß nicht, was die Bezeichnung wirtschaftlicher Fahrbereich bedeuten soll. Der Verschleiß ist im Übrigen bei stark untertourigem Fahren mit stotterndem Motor bestimmt ziemlich groß, zum Beispiel durch schlechte Verbrennung und entstehende Rückstände. --BlackEyedLion (Diskussion) 20:47, 10. Jan. 2023 (CET)
Natürlich ist er das, aber davon war ja auch nicht die Rede, sondern vom höchsten noch vernünftig fahrbaren Gang (wo also die benötigte Leistung noch gut geliefert wird). --Kreuzschnabel 07:16, 11. Jan. 2023 (CET)
Der Satz ist im Bezug auf den Verschleiß unvollständig bis Quatsch. Zuallererst mal ist der Verschleiß bei kaltem Motor und hohem Drehzahlen aufgrund von Reibung, möglicherweise sogar bis hin zum Mozltorschaden, zu berücksichtigen. Motor kalt, Öl kalt, Öl nicht ausreichend viskos, Ölfilm reißt ab. Zusätzlich kann man je nach Ölqualität und Nutzungsdauer mit hohen Drehzahlen in vergleichbare Szenarios kommen. Hohe Drehzaheln = hohe Temperaturen. Hitze verhilft dem Öl auf jeden Fall zu beschleunigt zu altern, und altes Öl verändert seine Eigenschaften. Desweiteren sollte man den Motor mindestens über der vom Hersteller empfohlenen Untergrenze rpm nutzen, darunter ist auch ungesund wg schon angesprochenen Verunreinigungen durch unreichend verbrannten Treibstoff. Ohne zu googeln. Es dürfte da sicher tausende an Treffern zu dem Thema geben. -Ani--46.114.153.237 21:08, 10. Jan. 2023 (CET)
Hier geht es primaer um die Frage, ob die Aussage, der wirtschaftliche Fahrbereich sei mit dem elastischen Bereich zwischen hoechstem Drehmoment und hoechster Leistung identisch, stimmt. Der Verschleiss ist da erst mal nur insoweit bedeutsam, als er natuerlich neben dem Verbrauch auch die Wirtschaftlichkeit mitbestimmt.
Klar ist es unwirtschaftlich, wenn man mit stotterndem Motor faehrt, aber zumindest mein Auto faehrt ganz problemlos auch kurz ueber der Leerlaufdrehzahl im sechsten Gang in der Stadt, wenn es nichts mehr zu beschleunigen gibt. Und bislang bin ich davon ausgegangen, dass auch in diesem Fall, also praktisch ueberall in der Ebene ausserhalb einer 30er Zone, der 6. Gang bei konstanter Geschwindigkeit der wirtschaftlichste ist. Dagegen scheint aber der o. g. Artikel zu sprechen, sofern er denn Recht hat. Und das ist eben die Frage ... -- Juergen 217.61.201.10 22:55, 10. Jan. 2023 (CET)
Wenn man davon ausgeht dass niedrigere Drehzahlen zu weniger Kraftstoffverbauch führen und ferner davon ausgeht, dass die Leerlaufdrehzahl im Bereich dessen liegt was saubere Verbennung des Kraftstoffs bewirkt, ist die Frage der Wirtschaftlichkeit ja nur noch weiter durch den Verschleiß bedingt. Da dieser am niedrigsten sein sollte wenn alle vorher genanntem Bedingungen erfüllt sind, ist deine ursprüngliche Annahme korrekt. Mir scheint, das bei dem Satz um den es geht noch mindestens ein weiterer Faktor wie "Last" eine Rolle spielt, die Wirtschaftlichkeit also damit verknüpft wird wie effizient der Motor bei Beschleunigung wirkt. Anders kann ich mir den Satz nicht erklären, denn maximaler Drehmoment ist in der Regel erst bei recht hoher Drehzahl gegeben, was eigentlich Spritverbrauch und Alterung der Schmiermittel bewirkt. -Ani--46.114.153.237 23:09, 10. Jan. 2023 (CET)
Wenn der Bereich zwischen den beiden genannten Drehzahlen als wirtschaftlicher Bereich bezeichnet wird, ist das definitionsgemäß richtig. Es fehlt aber im Artikel eine Quelle, dass dieser Drehzahlbereich so bezeichnet wird. --BlackEyedLion (Diskussion) 23:11, 10. Jan. 2023 (CET)
Wenn Du diese Definition nachvollziehen kannst, dann erklaere uns doch bitte, warum trotzdem wie von -Ani- und mir angenommen der Verbrauch ausserhalb des wirtschaftlichen Bereichs am geringsten ist oder warum wir beide uns irren. -- Juergen 217.61.201.10 00:52, 11. Jan. 2023 (CET)
Wie kann man eine Definition nachvollziehen? Im Artikel steht, dass der Bereich zwischen den beiden Drehzahlen wirtschaftlicher oder elastischer Bereich genannt wird. Da gibt es nichts nachzuvollziehen. Man kann ihn nennen, wie man will. Es wäre jedoch eine Quelle gut, dass es tatsächlich Personen gibt, die ihn so nennen. Was hat der Verbrauch damit zu tun, wie dieser Bereich genannt wird? --BlackEyedLion (Diskussion) 07:35, 11. Jan. 2023 (CET)
Hat dein Fahrzeug einen Bordcomputer, der dir den momentanen Verbrauch im Armaturenbrett anzeigen kann? Wenn ja, probier es doch einfach mal aus, bei gleicher Geschwindigkeit mit unterschiedlichen Drehzahlen und unterschiedlichen Gängen zu fahren. Ich meine mich zu erinnern, als ich das selber mal ausprobiert habe, gesehen zu haben, dass die Bordelektronik mir den geringsten Verbrauch bei etwa 1600-1800 Umdrehungen/min angezeigt hat, bei einer Motordrehzahl von unter 1400 war der verbrauch dann ungefähr so hoch wie bei 2200-2400 U/min. Grüße, Grand-Duc ist kein Großherzog (Diskussion) 01:45, 11. Jan. 2023 (CET)
Aus der Sicht der Kurbelwelle ist der "wirtschaftliche Bereich" derjenige, bei dem der spezifische Verbrauch in g/kWh niedrig ist. Wie weit sich das Fahrzeug pro kWh bewegt hat, weiß der Motor nicht. Insofern kann es schon sein, dass der niedrigste Verbrauch pro km außerhalb der Definition des elastischen Bereichs ist. Die Übergänge sind nicht scharf. --31.212.212.83 02:31, 11. Jan. 2023 (CET)
Bis hierhin fast nur wilde Spekulationen und TF. Die Formeln und Aussagen hat sich der Wikiautor sicherlich nicht ausgedacht, sondern aus einem entsprechenden Fach(kunde)buch. Das ließe sich bei vorliegender Literatur leicht verifizieren und bequellen, ggf. korrigieren und ergänzen. Entsprechenden Baustein in den Artikel setzen und gut ist's. --31.212.212.83 01:59, 11. Jan. 2023 (CET)
"wilde Spekulationen und TF"; ist das im Artikel anders? Der ist defacto acuh quellenlos, denn die Literatur ist nicht unbedingt die Quellenangabe; Eingetragen wurde die Literatur nachdem die Formeln drin standen und auch von einem anderen Autor: guckst Du. zugleich stand zu dem Zeitpunkt nichts von Wirtschaftlichkeit im Artikel. Das wurde hier quellenlos eingetragen. da er noch aktiv ist pinge ich User:Max schwalbe mal an. ...Sicherlich Post 03:33, 11. Jan. 2023 (CET) und das "erledigt" habe ich mal entfernt. Nur weil ein Wikipedia-Artikel irgendwas behauptet muss das noch lange nicht korrekt sein und darf hinterfragt werden. Auch gehts bei der Frage ja ggf. auch um eine Erläuterung
Und wenn schon? Dass die Literatur erst nach den Formeln eingetragen wurde, heisst ja nicht, dass die Formeln falsch zitiert oder quellenlos sind. Im Fließtext steht "nach Wolfgang Flössel". Der hat ein Werk "Bergsteigefähigkeit und Literleistung" herausgebracht, auf den sich die anderen Werke womöglich beziehen. Im Grunde ist alles drin. Erklärungsbedürftig bleibt bestenfalls der Umstand, dass Spezifischer Verbrauch (nach Strecke) und Spezifischer Kraftstoffverbrauch (nach kWh) nicht identisch sind. --31.212.212.83 07:15, 11. Jan. 2023 (CET)
Im Grunde und womöglich sind keine Quellenangabe und wenn es für Dich persönlich erledigt ist; okay, gut! Du bist allerdings nicht der Fragende und es scheint mit nichten so klar. Daher habe ich Deine Erle entfernt. ...Sicherlich Post 08:44, 11. Jan. 2023 (CET)
Ist das ein neuer Maßstab? Wenn Quellenangaben erst nach den Inhalten hinzugefügt sind, sind diese anzuzweifeln? "Womöglich" aus meiner Sicht deshalb, weil ich es nicht nachprüfen kann, denn ich habe keinen Zugriff auf die genannten Fachbücher. Das ist sonst auch kein Problem und ich habe keinen Grund, die Plausibilität der Behauptungen anzuzweifeln. Woher die Diskrepanz in der Interpretation der Behauptungen kommt, hab ich schon erklärt. --31.212.212.83 11:40, 11. Jan. 2023 (CET)
Allerdings wäre es sinnvoller, das auf der Artikeldisk zu klären. --95.116.66.241 11:46, 11. Jan. 2023 (CET)
Sind es denn Quellenangaben? Nur weil jmd. Literatur hinzufügt, heißt das nicht, dass es die Quellenangaben sein sollen (Schrieb ich oben ja schon). Das ist auch kein neuer Maßstab. Und das du keinen Grund hast; Wie auch schon geschrieben: "okay, gut". ... Artikeldiskussionen zu nicht gerade Mainstream-Themen sind regelmäßig wertlos wenn man Feedback erwartet. Das ist ja auch nichts neues. ... Also; wenn es für Dich erledigt ist: wunderbar. Du bist allerdings nicht der TO ...Sicherlich Post 12:01, 11. Jan. 2023 (CET)
Es werden wohl zwei Dinge vermischt: Der wirtschaftliche Betrieb eines Verbrennungsmotors und der wirtschaftliche Betrieb eines privaten Automobils.
Der Motor läuft dann am wirtschftlichsten, wenn er pro kg Kraftstoff am meisten Kilowattstunden an der Antriebswelle produziert und die Abwärme auch noch genutzt werden kann. Das Auto ist dann am wirtschaftlichsten wenn es mich am wenigsten kostet, ich also pro Liter am meisten Kilometer fahren kann und keinen Verschleiss habe. -- QundA (Diskussion) 11:59, 11. Jan. 2023 (CET)
Siehe auch Volllast und Teillast. --95.116.66.241 14:21, 11. Jan. 2023 (CET)
Ja genau. Der "wirtschaftliche Fahrbereich" ist beim Verbrennungsmotor so zu verstehen wie oben von QundA gesagt, also eine gute ("wirtschaftliche") Ausnutzung des Kraftstoffs für Motorkraft bei geringem Verschleiß. Ob diese Motorkraft wirklich benötigt wird, ist eine ganz andere Frage: Am unteren Rand des wirtsch. Bereichs (nahe des max. Drehmoments) ist der Spritverbrauch am niedrigsten, aber die Leistung nicht maximal. Am oberen Rand (nahe der Nennleistung) ist die Leistung besonders hoch, aber der Spritverbrauch schon etwas erhöht. Ein starker Motor kann also meist am unteren Rand des wirtsch. Bereichs gefahren werden, verbraucht aber dort etwas mehr Sprit als ein kleinerer Motor. Der schwache Motor wiederum muss jedoch öfter bei höheren Drehzahlen gefahren werden, damit die Kraft ausreicht. Es ist also gar nicht so einfach zu sagen, wie groß/stark ein Motor sein muss, damit sein wirtschaftlicher Fahrbereich auch in der Fahrpraxis tatsächlich zu möglichst niedrigem Spritverbrauch führt (ganz zu schweigen von anderen Abhängigkeiten wie Fahrkompetenz des Fahrers, wie viel Gas wann gegeben wird, Drehmoment- u- Leistungskurvenverlauf, Fahrzeuggewicht, Fahrzeugzweck usw.). Zur Quellenlage: Das kommt in der KFT beiläufig regelmäßig immer mal vor, oder auch in div. Sachbüchern. Ich habe aber bisher leider keine Quelle gesehen, wo der Begriff mal ausführlich erläutert wird. Gibt es aber bestimmt...--Max schwalbe (Diskussion) 21:00, 12. Jan. 2023 (CET)
P.S. mit "kleiner" und "größerer" Motor ist jetzt stark verallgemeinert, das gilt so nicht immer. Beispiel: Gesetzlich sind Kleinkrafträder bekanntlich auf 50 Kubik Hubraum limitiert. Technisch ist das nonsens, denn bei 70 Kubik Hubraum lässt sich bei sonst identischer Konstruktion zumindest beim Zweitakter mehr Leistung bei weniger Spritverbrauch erzielen, innerhalb des wirtschaftlichen Fahrbereichs.--Max schwalbe (Diskussion) 21:08, 12. Jan. 2023 (CET)

Organ- und Blutspende intersexueller Menschen

In den Nachrichen kam das homosexuelle Menschen Blut spenden dürfen ohne Einschränkungen, Das Gesetz soll geändert werden. Dürfen auch intersexuelle Menschen Blut oder Organe spenden? Auf der Webseite der Bundeszentrale für gesundheitliche Aufklärung (BZgA) oder organspende-info.de steht nicht dazu. --Fiver, der Hellseher (Diskussion) 19:24, 10. Jan. 2023 (CET)

Es geht nicht um Homosexualität, sondern um Männer, die Sex mit Männern haben (sowie um Frauen, die Sex mit Männern, die Sex mit Männern haben, haben). Im Übrigen scheint mir Intersexualität nicht mit Homosexualität vergleichbar zu sein: Das Erste ist eine sexuelle Identität, das Zweite eine sexuelle Orientierung. --BlackEyedLion (Diskussion) 19:30, 10. Jan. 2023 (CET)
Kurzer Faktencheck: Sie dürfen das schon heute, so lange sie zuletzt nicht mehrere oder einen neuen Sexualpartner hatten. Und sie werden es auch in Zukunft in der gleichen Konstellation nicht dürfen, es wird nur das Wort Homosexuelle (oder MdSmMh) nicht mehr auftauchen. --Studmult (Diskussion) 15:15, 12. Jan. 2023 (CET)
Ich weis auch das unter bestimmten Bedingungen HIV-postive Menschen Organe spenden dürfen. Das war ja eine Zeitlang auch nicht möglich. Bei der Bundesärtzekammer wird die HIV-Infektion nur als Ausnahme bei Herz- Und Lungentransplantation aufgeführt. Link dazu: https://www.bundesaerztekammer.de/baek/ueber-uns/richtlinien-leitlinien-empfehlungen-und-stellungnahmen/transplantationsmedizin/wartelistenfuehrung-und-organvermittlung Die Intersexualität wird dort nicht erwähnt.Die Medzin bedrachtet Intersexualität in vielen Fällen immer noch als dringend behandlungswürdige Erkrankung mit entsprechenden Folgen für die Patienten. --Fiver, der Hellseher (Diskussion) 22:44, 12. Jan. 2023 (CET)

Grillkohle aus Garzweile ?

Könnte man die in Garzweiler gewonnen Kohle auch als Grillkohle verkaufen, wäre doch eigentlich besser weil Braunkohle besser brennt als Holzkohle?--2001:9E8:3F58:B900:7CC7:5BFC:9506:62E8 18:05, 12. Jan. 2023 (CET)

Man könnte sich auch einen Ring durch die Nase ziehen und dich daran vorführen. --Bahnmoeller (Diskussion) 19:30, 12. Jan. 2023 (CET)
Grillkohle besteht aus reinem Kohlenstoff, Braunkohle aus Garzweiler nicht. Man müsste die Braunkohle also erst durch eine Kokerei zu Braunkohlenkoks verarbeiten. Der eignet sich dann genausogut als Grillkohle wie Holzkohle, ist aber leider klimaschädlich. Hausaufgabe: Finde eine Kokerei, die Dir Garzweiler Braunkohle zu Braunkohlenkoks verarbeitet. --Rôtkæppchen₆₈ 19:57, 12. Jan. 2023 (CET)
Inwiefern Braunkohlenkoks klimaschädlicher als Holzkohle sein soll, wird wohl auf ewig Dein Geheimnis bleiben - das bei deren Verbrennung entstehende CO2 verhält sich jedenfalls genau gleich klimawirksam. Ich weiß nicht, ob Braunkohle überhaupt aus toten Bäumen entstanden ist, aber wenn, sind diese Bäume schon vor Jahrmillionen gestorben, im Gegensatz zu denjenigen, die für die Holzkohle gefällt wurden. Und was am Mangel von C-14 nun so klimaschädlich sein soll, ist auch nicht so recht eingängig. --77.3.83.252 23:26, 12. Jan. 2023 (CET)
Es ist schön, dass Dir meine Argumentation einleuchtet, auch wenn Deine Worte das Gegenteil vermuten lassen sollen. --Rôtkæppchen₆₈ 00:30, 13. Jan. 2023 (CET)
Äußerst schwach, selbst für Deine Verhältnisse... --77.8.121.249 00:58, 15. Jan. 2023 (CET)
(Langeweile kenne ich - deswegen bin ich hier - aber was Garzweile sein soll, ist mir unbekannt.) - Definiere "besser brennt" - inwiefern brennt Braunkohle "besser" als Holzkohle? Und welchen Vorteil hätte die garzweilige Kohle denn eigentlich? Ich vermute, daß es überhaupt kein Problem ist, ein paar Tonnen davon von RWE für'n Appel und 'n Ei zu kaufen - wenn es seitens der Verbraucher eine Nachfrage zum Grillen dafür gäbe, hätte das wohl schon längst jemand angeboten. Nach meiner Vorstellung soll Grillkohle praktisch keine Flammen erzeugen, sondern nur glühen und Infrarotstrahlung abgeben, die Verbrennung und Energiefreisetzung also direkt auf der festen Oberfläche stattfinden. Da Braunkohle diese Eigenschaften nicht hat, würde ich sie dafür deswegen nicht nehmen. Als Hausbrand mag sie in Grenzen ok sein, obwohl sie nur einen geringeren Brennwert als Steinkohle hat, was häufigeres Stochen des Ofens erfordert, und außerdem fällt dabei unverhältnismäßig viel Asche an, was auch nicht erwünscht ist. Wie hoch ist eigentlich der Grillkohleverbrauch? Vermutlich ist er gegenüber den Fördermengen dieser Grube vernachlässigbar gering, so daß der Betreiber des Tagebaus an diesem Geschäft hat kein Interesse hat. --77.3.83.252 23:47, 12. Jan. 2023 (CET)
Meines Erachtens ist Rotkaeppchen68's Gedankengang recht einleuchtend: Der in der Braunkohle befindliche Kohlenstoff war seit Urzeiten gebunden, durch seine Freisetzung wird das Kohlendioxid in der Atmosphäre also merklich gegenüber dem Zustand vor Beginn der Industrialisierung erhöht (natürlich nicht wesentlich durch einen einzigen Verbrennungsvorgang, aber durch den gesamten Verbrauch fossiler Brennstoffe in der Summe schon wahrnehmbar, und vor allem (viel) mehr, als wieder gebunden wird), während der Kohlenstoff in der Holzkohle aus dem immer noch lebendigen Kreislauf stammt. Jedes Jahr werden ganz natürlich, ohne menschliches Zutun, viele Millionen Tonnen CO² freigesetzt und wieder von wachsenden Pflanzen gebunden. Doch durch das intensive Verbrennen fossiler Brennstoffe übersteigt die Menge des freigesetzten Kohlendioxids inzwischen die des wieder gebundenen CO²s. Übrigens laut den üblichen Angaben zur Inkohlung erfolgt diese überwiegend aus Pflanzen, seien es nun Bäume oder andere Pflanzen gewesen. --Universal-InteressierterDisk.Arbeit 00:15, 13. Jan. 2023 (CET)
Von was für einem "lebendigen Kreislauf" träumst Du denn da? In der Realität sieht der "Kreislauf" bei der Holzkohleherstellung so aus: Am Anfang war die Welt wüst und leer, und der Geist Gottes schwebte über den Wassern. -> Urwald, unberührte Natur -> Kahlschlag, umweltschädliches Abfackeln der Schwelgase aus Holzabfällen zur Holzkohleherstellung -> Agrarwüste, Artenvielfalt futsch -> Desertifikation. Womit der Ausgangspunkt (wüst und leer) wieder erreicht wäre: Der HErr kann mit einer neuen Schöpfung loslegen, um den nächsten Durchgang dieses Kreislaufs zu starten. Zusätzlich wird dann noch die Holzkohle verbrannt und in Form von Zeh-Oh-Quadrat in die Atmosphäre freigesetzt, um den begonnene Desertifikationsprozeß zu Ende zu bringen ("denn alles, was entsteht, ist wert, daß es zugrunde geht", wie der olle Jeheimrath so treffend schrieb). Demgegenüber vermeidet die Verbrennung fossiler Brennstoffe die Biotopvernichtung und setzt nur Kohlendioxid frei. Die Verwendung von Braunkohle ist ein wichtiger Beitrag zur Energiewende und zum Atmosphärenschutz, wie Dir Habeck und Co. gerne erklären können. (Jetzt fange ich schon wieder an, zu predigen. Dabei hat unser Pastor das Atheisten wie mir ausdrücklich verboten.) --77.8.98.92 06:38, 13. Jan. 2023 (CET)
Richtig muß es heißen: „Demgegenüber vermeidet die Verbrennung fossiler Brennstoffe die Biotopvernichtung und setzt nur über Jahrmillionen hinweg gebundenes Kohlendioxid erneut in die Atmosphäre frei.” Wo siehst Du hierin einen Beitrag zum Atmosphärenschutz? Die Freisetzung sorgt dafür, daß wir wieder eine Zusammensetzung der Atmosphäre herbeiführen, wie sie zu dem Zeitpunkt geherrscht hat, als diese Kohlendioxidmengen eingelagert wurden. --92.73.145.117 07:04, 13. Jan. 2023 (CET)
Ob "seit Ewigkeiten" geologisch (fossil) gelagerter Kohlenstoff oder solcher aus Entwaldung und Desertifikation stammender freigesetzt wird, ist für die Atmosphäre gleich schädlich, insofern trägt die Verwendung von Holzkohle nichts zum Atmosphärenschutz bei, um mit diesem gebetsmühlenhaft wiederholten frommen grünen Märchen aus der industriefeindlichen Ideologienschmiede mal aufzuräumen. Wohl aber trägt der Erhalt von Biotopen durch die Verwendung von fossilen Energieträgern und damit deren klimatischer Funktion (Feuchtigkeitsspeicher, Temperaturausgleich) zum Atmosphärenschutz bei. Die verlogene Argumentation grüner staatstragender Parteien und ihrer Unterstützer ist eben: "Ihr dürft mit Kohlenstofffreisetzungen ruhig rumsauen, solange der nur aus grünen zertifizierten biologischen Quellen stammt, denn der ist dann Bestandteil des 'Kreislaufs'!" Nein, dürft ihr nicht - das Argumentationsschema ist hohler Unsinn, diese "Zertifizierungen" sind nichts wert. (Und was Minister Habecks Argumente und die seiner "Freunde" zu Lützerath angeht, mußt Du die fragen - da werden dann Begriffe wie "Kompromiß" und "für den Übergang erforderlich" und "vorzeitiger(!) Kohleausstieg" fallen (sic transit gloria mundi - argumentativ scheinen mir die Grünen derzeit nicht eben den besten Stand zu haben, nachdem sie zuvor schon in Sachen militärischer Sicherheit und bewaffneter Friedenssicherung urplötzlich Vernunft angenommen haben und ihnen daraufhin die Spinner und Utopisten in Scharen davongelaufen sind). (Und Du mußt mal lernen, Sprachebenen und rhetorische Stilmittel zu verstehen - das ist umso wichtiger, als das heutzutage zugunsten Genderunfug u. dergl. im Deutschunterricht unter den Tisch fällt. Manchmal steht in einem Text nämlich genau das Gegenteil von dem, was man daraus bei wortgetreuer Rezeption entnehmen zu können glauben könnte. Wenn man letzteres macht, macht man sich damit nämlich bei den verständigen Lesern zum Brot.) --77.8.98.92 09:54, 13. Jan. 2023 (CET)
»Ob "seit Ewigkeiten" geologisch (fossil) gelagerter Kohlenstoff oder solcher aus Entwaldung und Desertifikation stammender freigesetzt wird, ist für die Atmosphäre gleich schädlich,«
Leider wieder falsch. Der im laufenden Stoffkreislauf eingebundene Kohlenstoff (z.B. lebende Bäume) wird nach Ende der Lebenszeit der Pflanze ohnehin wieder in die Atmosphäre freigesetzt. Diese Freisetzung geschieht auch, wenn Du z.B. das Holz nicht verbrennst, sondern langsam im natürlichen Biotop verrotten läßt, oder vor seiner Zersetzung eine zeitlang nichtenergetisch (z.B. als Bauholz) nutzt. Wenn Du nun also fossile Energieträger verwendest, kommt deren Kohlenstoffgehalt wieder zu dem gegenwärtig im globalen Stoffkreislauf (in Biomasse und Atmosphäre) vorhandenen hinzu. Und um dieses Mehr an freigesetztem Kohlenstoff geht es, den kannst Du mit keinem rhetorischen Trick wegdiskutieren. Das Ergebnis ist eine Atmosphärenzusammensetzung wie vor ein paar Millionen Jahren. Und genau deswegen ist es für die Atmosphäre nicht "gleich schädlich", ob Du Fossiles oder Regeneratives verfeuerst. Das ist aber kein Freibrief zum "herumsauen" mit nachwachsenden Rohstoffen. --92.73.145.117 10:31, 13. Jan. 2023 (CET)
Leider immer noch richtig: Holzkohle stammt aus Bäumen, deren Lebenszeit jemand vorzeitig beendet hat, zusammen mit dem derjenigen der ganzen Gegend, was sich Kahlschlag oder Entwaldung nennt. Und wenn sie verbrannt wird, gibt es keinen Kreislauf, der den Kohlenstoff wieder in dieser Menge an diesen Ort zurückführt - bis dort ein neuer Wald entstanden ist (falls überhaupt jemals), vergehen Jahrhunderte. Das ist die traurige Realität und kein "rhetorischer Trick". (Verstehen mußt Du es schon selbst.) --77.8.98.92 02:14, 14. Jan. 2023 (CET)
Der Kohlenstoff wird während des gesamten Wachstums eingelagert, und das beginnt nicht erst nach 100 Jahren, wie Du hier glauben machen willst. Deine „traurige Realität” ist also doch nur ein rhetorischer Trick, das ist leicht zu verstehen. Genau wie das einleitende Bild vom armen ermordeten Baum, „dessen Lebenszeit jemand vorzeitig beendet hat”. Großräumiger Kahlschlag und Entwaldung sind bei uns schon lange kein Thema mehr. Natürlich ist es noch sinnvoller, einzelstammweise zu arbeiten, wie es in der naturnahen Waldwirtschaft üblich ist. Damit es dann ein Kreislauf wird, darf zu jeder Zeiteinheit (auch weltweit) natürlich nur so viel eingeschlagen werden, wie nachwachsen kann, nicht mehr. Das sind die Grenzen des Wachstums. Und so ein Kreislauf hat nichts mit dem ganzen Greenwashing bzw. Greenscamming zu tun, das von Politik und Wirtschaft unisono mittels einer Flut von Zertifikaten betrieben wird. Und das alles bedeutet noch immer nicht, daß es deswegen besser sei, fossile Brennstoffe zu verfeuern, und damit zusätzliches langfristig der Atmosphäre entzogenes Kohlendioxid dieser erneut wieder zuzuführen. Ganz im Gegenteil, aber das hatten wir ja schon. --92.73.145.117 07:03, 14. Jan. 2023 (CET)
Welches Wachstum denn, wieso "bei uns"? Es geht um global relevante Biotope wie z. B. den tropischen Regenwald. Mach doch einfach mal die Augen auf und nimm die Realität außerhalb Deines grünen Märchenbuch zur Kenntnis! "Einzelstammweise", vielleicht noch aus dem bayerischen Staatsforst, ja? Und wovon träumst Du nachts? Und selbst wenn irgendwo wieder was aufgeforstet wird (was de facto im großen und ganzen nicht stattfindet): es ist beinahe völlig belanglos, daß "Der Kohlenstoff wird während des gesamten Wachstums eingelagert, und das beginnt nicht erst nach 100 Jahren, wie Du hier glauben machen willst": "wird" ist ein lange dauernder Prozeß: der heute abgeholzte Wald steht nicht morgen wieder da und hat den ganzen Kohlenstoff aus der Grillkohle wieder aufgenommen, die ist und bleibt sehr lange in der Atmosphäre, und die verschwundenen Arten bleiben für immer ausgestorben. "naturnahe Waldwirtschaft", "üblich" - lächerlich. "Damit es dann ein Kreislauf wird, darf zu jeder Zeiteinheit (auch weltweit) natürlich nur so viel eingeschlagen werden, wie nachwachsen kann, nicht mehr." Wird es aber nicht, und langfristig sind wir alle tot. Wir haben jetzt Überbevölkerung, wir haben jetzt einen Mangel an Agrarflächen, auf denen es auch noch eine scharfe Konkurrenz um den Anbau von Nahrungsmitteln, Viehfutter und Energiepflanzen (und auch noch PV-Anlagen und WKA) geht, wir haben jetzt einen unstillbaren Hunger der Industrie nach Bauholz und anderen Holzprodukten (sowie die Brennholznachfrage in den Entwicklungsländern für die Kochfeuer), und letzteres bedeutet eben Kahlschlag, Entwaldung, Urwaldrodung: da wird nichts wiederaufgeforstet, sondern planiert und Soja und Mais und Kokain angepflanzt. Deine rhetorischen Tricks und Dein Greenwashing von Holzkohle ändern an dieser Realität gar nichts. --77.8.121.249 10:36, 14. Jan. 2023 (CET)
Btw: welche Biotope? Zumindest Europa ist seit spätestens dem 12./13. Jahrhundert eine einzige großflächige Wald- und Feldwirtschaft (abgesehen von kleinen biotopischen Inseln wie dem Bayrischen Wald). -Ani--46.114.155.174 10:43, 13. Jan. 2023 (CET)
Ach so, Polen liegt gar nicht in Europa? Und überhaupt: Wer redet denn von Europa? --77.8.98.92 02:23, 14. Jan. 2023 (CET)
Polen ist aber auch im Ganzen kein Biotop. Lies doch noch mal was ich geschrieben habe. -Ani--46.114.158.123 23:33, 14. Jan. 2023 (CET)
Hat auch niemand behauptet. Du hast Dir den Link also überhaupt nicht angesehen? Nun ja, mit dem selbständigen Denken und Sapere aude haben es manche halt nicht so... --77.8.121.249 00:48, 15. Jan. 2023 (CET)
Selbstverständlich habe ich den Link zur Kenntnis genommen. Du wopltest hiermit wohl beweisen dass es Biotope in Europa gibt. Da ich aber nicht behauptet habe das es keine gibt, war deine "Beweis" völlig unnötig, noch dazu als Pipelink, der in der Umsetzung mehr Relevanz suggerien soll als vorhanden ist. Der Name des Gebiets hätte gereicht - dass es in Polen liegt steht im Artikel.. Zudem ist dein Link nicht mal korrekt deklariert, denn das Gebiet ist nicht ausschließich auf Polen beschränkt, der größere Teil liegt im Nachbarland. Wenn du schon so verlinkst, dann mach es richtig und transparent. Mit diesen Spielchen diskreditierst du dich nur selbst. Das gleiche gilt auch für deine herablassende Semantik. Braucht niemand, und ist pädagogisch veraltet. Ein leichtes dich zu spiegeln und damit auszuhebeln, und wenn man möchte. -Ani--46.114.158.123 02:01, 15. Jan. 2023 (CET)
Grillkohle aus Braunkohle gibt es hier und da zu kaufen. Freiwillig würde ich die nie (wieder) verwenden, denn nach meiner Erfahrung ist die zum Grillen ganz besonders schlecht geeignet. Sie brennt widerwillig und stinkt. Ähnliche Aussagen lassen sich leicht in Grillforen finden. --31.212.244.31 03:09, 13. Jan. 2023 (CET)
Also genau das vermutete Verhalten. Aber was genau ist "Grillkohle aus Braunkohle"? Ist das Braunkohlekoks, oder einfach nur getrocknete (wenigstens das!) Braunkohle? Andererseits: Dein wenig begeisterter Bericht über das Produkt beantwortet die Frage des TO zumindest indikativ positiv: wenn es Grillkohle gibt, die aus Braunkohle hergestellt wird, dann kann man sie grundsätzlich wohl auch aus garzweiliger Braunkohle herstellen. (Ob die etwas taugt, ist eine völlig andere Frage.) - Was mich in dem Zusammenhang noch interessieren würde: warum wird kein Koks als Grillkohle angeboten bzw. verwendet? Welchen Vorteil hat Holzkohle? (Natürlich gibt es dazu einiges zu vermuten: Pyrolysekohle hat abhängig von Ausgangsmaterial und Prozeßführung unterschiedliche Zusammensetzungen und Eigenschaften. Bzgl. der Zusammensetzung gibt es die Aspekte Aschegehalt und Gehalt an teilweise giftigen organischen Kohlenwasserstoffen. Der Aschegehalt von Pflanzenkohle ist wahrscheinlich sehr niedrig, weil das Ausgangsmaterial keine Gesteinsanteile enthält. Die giftigen Anteile sind eine Frage der Prozeßführung: sie können durch Glühen bei ausreichend hohen Temperaturen - das zu Verlusten bei der Ausbringung führt - minimiert werden. Beim Einsatz als Heizmaterial ist dieses starke Ausglühen nicht erforderlich, weil die entsprechenden Bestandteile problemlos mitverbrennen, bei der Ausbringung von PyCCS-Kohle auf Agrarflächen schon, damit die Böden nicht vergiftet werden, und Grillkohle stinkt dann wahrscheinlich weniger. Holzkohle hat zudem noch eine niedrigere Dichte, also eine feinere Struktur als Koks, heizt damit vermutlich wegen der größeren Oberfläche mit höherer Leistung. Ist das für das Grillen wesentlich? Nebenbei würde mich noch interessieren, warum die Pufferküsser für die Feuerung Fettkohle und nicht Koks verwenden. Billiger?) --77.8.98.92 05:52, 13. Jan. 2023 (CET)
Kohlegefeuerte Dampflokomotiven sind in der Regel zu einer Zeit konstruiert worden, als die Montanindustrie ein höchst gewichtiger Wirtschaftsfaktor war (zweite Hälfte 19. Jhr und erste Hälfte 20. Jhr.). Die Stahlherstellung konsumiert viel Koks, insofern dürfte (persönliche Vermutung) damals eine Konkurenzsituation zwischen Mobilität mit Dampfmaschinen und Produktion von Stahl geherrscht haben. Stahl mit Steinkohle (Anthrazit ausgenommen) geht nicht, aber Dampfmaschinen heizen geht. Also werden die Kokereien für die Hochöfen produziert und die Dampflokomotivenkonstrukteure ihre Feuerbüchsen für Steinkohle ausgelegt haben. Diese damaligen Entwurfsentscheidungen wirken bei den erhaltenen Maschinen nach. Ich habe vor Kurzem in einer Eisenbahnzeitschrift (Eisenbahnkurier?) von einer Museumsbahn gelesen, die, ob der Lieferschwierigkeiten und -kosten für schlesische Steinkohle, eine ihrer Lokomotiven für eine Holzpelletfeuerung umgebaut hat. Holzpellets sollen in einer Energiegehaltsvergleichgröße her (entweder pro Masse oder pro Volumen, die genaue Angabe habe ich vergessen) mit dem bei der Deutschen Reichsbahn (DDR) getesteten Braunkohlestaub und Braunkohlekoks vergleichbar sein. Die Eisenbahner waren gemäß diesem Artikel zufrieden mit den Ergebnissen des Versuchs. Ansonsten: Dampflokomotiven könnten auch sicher mit Koks gefahren werden, vgl. en:Phoebe Snow (character). Dieser englische Artikel hat eine Bahngesellschaft zum Thema, welche Anthrazit verwendet hat. Grüße, Grand-Duc ist kein Großherzog (Diskussion) 23:09, 13. Jan. 2023 (CET)
Im gewissen Sinne liegst richtig @Grand-Duc. Sie waren auf gute Steinkohle ausgelegt. Es war aber vorallem eine Kostenfrage. Wobei eben bei Kohle auch nicht alle gerne benutzt wird zum verfeueren in Dampflokomotiven. Doch aus welcher Kohle wird Koks hegstellt? Richtig aus der "guten" Fettkohle, also genau die Kohlesorte die üblicherweise auch für das befeuerern der Dampflokomotiven verwendet wird/wurde. Also ist die Sache mit der Frage zu beantworten; Wieso solte man einen Brennstoiff zuerst verkoks/weiterverarbeiten, wenn der auch ohne diese Verarbeitungsstuffe direkt benutzbar ist? --Bobo11 (Diskussion) 09:36, 14. Jan. 2023 (CET)
Das ist soweit im gewissen Sinne richtig. --95.116.152.237 17:53, 14. Jan. 2023 (CET)
Sagt man nicht üblicherweise in gewissem Sinne? (Also unbestimmt und nicht bestimmt) --2A0A:A541:FDDC:0:61AA:613A:151A:1ABA 01:47, 15. Jan. 2023 (CET)
Vielleicht lehne ich mich zu weit aus dem Fenster, aber rostgefeuerte Lokomotiven kann man im Prinzip mit jedem festen Brennstoff betreiben, mag es auch mit Leistungsverlusten oder geringerer Lebensdauer einzelner Bauteile verbunden sein. --2A0A:A541:FDDC:0:61AA:613A:151A:1ABA 01:44, 15. Jan. 2023 (CET)
"Kann" vielleicht, aber da werden wohl verschiedene wirtschaftliche Aspekte eine ausschlaggebende Rolle spielen, u. a. natürlich auch Verschleiß und Lebensdauer. --77.8.121.249 01:57, 15. Jan. 2023 (CET)
Es ist möglich, dass die in Garzweiler gewonnene Braunkohle als Grillkohle verkauft werden könnte, da sie einen höheren Brennwert hat als Holzkohle und daher möglicherweise länger brennt. Allerdings gibt es einige Faktoren, die dies beeinflussen könnten. Einer davon ist die Qualität der Braunkohle. Braunkohle, die in Tagebaubetrieben wie dem Tagebau Garzweiler gewonnen wird, kann unterschiedliche Qualitäten aufweisen, abhängig von ihrem Alter und ihrer Lagerstätte. Einige Braunkohle kann sehr trocken und hochwertig sein, während andere feuchter und von niedriger Qualität sein kann. Ein weiterer Faktor ist die Verarbeitung der Braunkohle. Um als Grillkohle verwendet zu werden, muss die Braunkohle zuerst getrocknet und gereinigt werden, um unerwünschte Verunreinigungen zu entfernen. Dies kann zusätzliche Kosten verursachen und die Rentabilität beeinträchtigen. --2A02:3030:80D:1798:1F4F:3F8F:CD3:F843 18:27, 14. Jan. 2023 (CET)
Braunkohle einen höheren Heizwert als Holzkohle? Wie das? Holzkohle sollte doch wohl im wesentlichen reiner Kohlenstoff sein, während Braunkohle stark verunreinigter, ziemlich feuchter Kohlenstoff ist. Was gibt beim Verbrennen wohl mehr Energie ab? Der Reinigungsaspekt bzw. dessen Kosten sind sicherlich ein gewichtiger Gesichtspunkt, nur darf man auch nicht übersehen, daß die Herstellung von Holzkohle auch vergleichsweise aufwendig ist. (Wie aufwendig? Keine Ahnung: es zieht natürlich nicht mehr der Köhler mit seinen Leuten durch den Schwarzwald, holzt ihn ab und baut seine Meiler, sondern die Holz- und Sägewerksindustrie benutzt die Holzabfälle als Rohstoff (im Idealfall werden die durch Windwurf geschädigten und deswegen als Konstruktionsholz nicht mehr verwendbaren toten Bäume dafür verwendet) und pyrolysiert sie, und noch optimaler setzt sie das dabei anfallende Holzgas als Brenn- und Kraftstoff sowie als Prozeßgas ein. Es könnte gut sein, daß das für die Holzindustrie somit wirtschaftlich interessanter als die Herstellung von Preßspanprodukten oder Holzpellets ist. Was mögen die Gestehungskosten von Holzkohle sein? Wie bei allen Kuppelprodukten schwierig zu beurteilen. Das Produktspektrum der Holzindustrie ist schließlich sehr vielfältig, auch um die Nutzung der "Abfälle" konkurrieren sehr viele verschiedene Endprodukte. - Ich habe übrigens auch keine Ahnung, was Grillkohle - aus Holzkohle - im Baumarkt derzeit wohl so kostet; im Verhältnis zu fossilen Brennstoffen auf den Energieinhalt bezogen vermutlich teuer. Daß Braunkohlengrillkohle deswegen kostengünstiger sein könnte, hielte ich zumindest für vorstellbar, aber wenn die halt nichts taugt... Und die klimapolitischen Argumente verkneife ich mir jetzt mal. Die Wohnungsbaugesellschaft hier hat übrigens schon vor längerem das Grillen mit Kohle auf den Balkonen wegen der Geruchsbelästigung durch den Rauch verboten - Grillen an sich nicht, d. h. Grillen mit Elektro- oder Gasgrills ist natürlich weiterhin erlaubt. (Wobei man sich auch mal fragen darf, welchen Rauch Holzkohle eigentlich erzeugt - der Geruch von Qualm kommt ja wohl durch Schwelgase und Schwefelverbindungen zustande, und die sollten aus Holzkohle eigentlich gerade nicht freigesetzt werden.) Wenn man jetzt aber mal die Klimaschädlichkeit des Grillens mit elektrischen Grills, die Braunkohlestrom verbrauchen, mit dem des Grillen mit Brennstoff aus Braunkohle vergleicht, möchte ich mal stark annehmen, daß der Strom dabei weniger gut abschneidet. Wärmepumpengrills gibt es natürlich natürlich (noch) nicht: rein theoretisch könnte man mit Strom einen Kompressor antreiben, der Luft auf Gluttemperatur erhitzt, und dann damit Grillsteine beheizen. Wenn man die Abwärme des Grills dann auch noch zur Luftvorwärmung benutzt, sähen die Energiebilanz und der ökologische Fußabdruck vielleicht gar nicht mal so schlecht aus.) --77.8.121.249 01:44, 15. Jan. 2023 (CET)

Die konkrete Ausgangsfrage ist damit wohl ausreichend beantwortet; weitere Klimadiskussionen bitte im Café tätigen.

Archivierung dieses Abschnittes wurde gewünscht von: -Ani--46.114.158.123 23:41, 14. Jan. 2023 (CET)
Wo Du dann mit Deinen seltsamen Vorstellungen natürlich genauso alt aussehen würdest. --77.8.121.249 00:58, 15. Jan. 2023 (CET)
Schreibt jemand der offensichtlich nicht mal ansatzweise verstanden hat was im Intro dieser Seite steht. --46.114.158.123 01:47, 15. Jan. 2023 (CET)
Eine sehr seltsame Vorstellung ist es jedenfalls, wenn jemand uns hier die Verbrennung von fossilen Brennstoffen wie Braunkohle als "klimafreundlich" verkaufen will, und das auch noch mit Überzeugung verteidigt, weil dabei ja "nur" CO2 freigesetzt werde. Aus welchem Greenscamming-Think-Tank stammt diese absurde Idee eigentlich? --92.73.145.117 01:34, 15. Jan. 2023 (CET)
(...) entfernt. --Nordprinz (Diskussion) 07:21, 15. Jan. 2023 (CET)

Grammatikfrage

Ich bräuchte einmal die Hilfe eines absoluten Grammatik-Experten. Unter Diskussion:The Banshees of Inisherin#Abschnitt "Kritik" Dativ oder Genitiv? beanstandet ein Benutzer den Satz „Mit eines von Martin McDonaghs besten Werken und mit zwei herausragenden Hauptdarstellern ist The Banshees of Inisherin ein fein gemachter Schlechtfühl-Genuss“ und möchte „eines“ zu „einem“ geändert haben. Nachdem ich mir nun über beide Varianten intensiv den Kopf zerbrochen habe, kann ich nicht sagen, welche Version die richtige ist. Einerseits passt „eines“ ganz offensichtlich nicht zu „Werken“, andererseits ergibt für mich „eines der besten Werke“ schon wieder Sinn. Vielleicht findet sich ja jemand, der Licht ins Dunkel bringen kann. --Frederico34 (Diskussion) 23:01, 12. Jan. 2023 (CET)

Eines der besten Werke ist Nominativ. Nach mit kommt aber der Dativ, und der Dativ von eines ist einem. --BlackEyedLion (Diskussion) 23:02, 12. Jan. 2023 (CET)
mit ist eine Präposition, die den Dativ verlangt. --Doc Schneyder Disk. 23:13, 12. Jan. 2023 (CET)
So einfach kann’s manchmal sein, vielen lieben Dank euch beiden. --Frederico34 (Diskussion) 23:15, 12. Jan. 2023 (CET)
Naja, es gibt nach einigen Präpositionen die Tendenz zum hyperkorrekten Genitiv, - geht mir auch so. --Doc Schneyder Disk. 23:19, 12. Jan. 2023 (CET)
"Eines" ist ebenso Genitiv und Akkusativ. Wird das Wort nicht pronominal wie hier, sondern mit einem begleitenden Bezugssubstantiv verwendet, ist es sogar ein eindeutiger Genitiv. Beispielsatz: Den Anblick eines seiner besten Werke wird man nie wider vergessen. --2A0A:A541:FDDC:0:61AA:613A:151A:1ABA 01:07, 15. Jan. 2023 (CET)
Davon abgesehen, dass der Satz stilistisch unglücklich formuliert ist, scheint es mir so, dass mit hier nicht als Präposition, sondern als Adverb gebraucht wird: es ist mit (d. h. nebst anderen) eines seiner besten Werke. Grüße Dumbox (Diskussion) 23:37, 12. Jan. 2023 (CET)
Ergänzung: mit ist dann tautologisch, denn "eines seiner besten Werke" sagt ja schon, dass es auch andere gibt - sprich, man kann es einfach weglassen. Grüße Dumbox (Diskussion) 23:42, 12. Jan. 2023 (CET)
Im vorliegenden Fall würde der Satz ohne „mit“ aber auch nicht wirklich stimmig klingen. --Frederico34 (Diskussion) 23:47, 12. Jan. 2023 (CET)
Scheint zu stimmen. Es ist verwirrend, weil das zweite "mit" in „mit zwei herausragenden Hauptdarstellern“ wiederum die Präposition ist. Der Unterschied wird deutlicher, wenn man umstellt: „ Banshees of Inisherin mit zwei herausragenden Hauptdarstellern ist ein fein gemachter Schlechtfühl-Genuss.“ ergibt Sinn und bedeutet, der Film enthält herausragende Hauptdarsteller. „ Banshees of Inisherin ist mit einem von Martin McDonaghs besten Werken ein fein gemachter Schlechtfühl-Genuss.“ ergibt keinen Sinn, weil der Film BoI selbst offensichtlich eines von McDonaghs besten Werken sein soll und damit kein anderer Film/kein anderes Objekt gemeint ist, das irgendwie in einer "mit"-Beziehung zu dem Film steht. --Blobstar (Diskussion) 23:49, 12. Jan. 2023 (CET)
<BK> Der Satz ist Murks, egal ob "s" oder "m": "Mit einem von seinen besten Werken ist der Film ein Genuss." Es soll doch anscheinend gesagt werden, dass der Film eines seiner besten Werke ist? So heißt das aber im Prinzip, dass der Film ein Genuss ist und eines seiner besten Werke auch ein Genuss ist. --Optimum (Diskussion) 23:56, 12. Jan. 2023 (CET)
In dem Satz „Mit einem (oder eines) von Martin McDonaghs besten Werken […] ist The Banshees of Inisherin ein fein gemachter Schlechtfühl-Genuss“ ist mit bestimmt kein Adverb. In einem Satz wie „Eines von Martin McDonaghs besten Werken […] ist The Banshees of Inisherin mit ein fein gemachter Schlechtfühl-Genuss“ ist mit das genannte Adverb. Im Übrigen ist der Satz (der mit der Präposition) inhaltlich sinnvoll, wenn man unter Werk nicht den Film selbst, sondern das Arbeitsergebnis McDonaghs versteht; ähnliche Formulierung: mit einer von Martin McDonaghs besten Leistungen, mit einem von Martin McDonaghs besten Arbeitsergebnissen. --BlackEyedLion (Diskussion) 23:59, 12. Jan. 2023 (CET)
In der Originalquelle steht „Featuring some of Martin McDonagh's finest work […“], also wäre vermutlich die Übersetzung „Mit einer von Martin McDonaghs besten Arbeiten“ eine gute und treffendere Lösung. --Frederico34 (Diskussion) 00:02, 13. Jan. 2023 (CET)
Ich stimme zu. --BlackEyedLion (Diskussion) 00:03, 13. Jan. 2023 (CET)
Ich würde "Als eine von Martin McDonaghs besten Arbeiten..." schreiben, bin aber kein Deutschlehrer :) --Optimum (Diskussion) 00:07, 13. Jan. 2023 (CET)
Das würde ich nicht. Featuring bedeutet, dass eine Eigenschaft des Films und nicht der Film selbst beschrieben ist. --BlackEyedLion (Diskussion) 00:22, 13. Jan. 2023 (CET)
<BK>Ich dachte erst, so ein Satz im Abschnitt Kritik muss doch offensichtlich ein Zitat sein, was deswegen problematisch wäre, weil man es dann nicht einfach so ändern dürfte – aber es stellt sich heraus, es ist eine Übersetzung eines englischen Zitats. Da darf sich natürlich fragen lassen, nicht nur ob ein einzelner Buchstabe geändert werden muss, sondern ob das Zitat nicht überhaupt anders besser übersetzt werden könnte und sich damit die fragliche Stelle auch anders ausdrücken lässt, um die Stolperstelle zu umgehen.<BK> Dazu gehört, was meines Erachtens durch das englische Original logisch wird, was BlackEyedLion sagt, dass "Werk(e)" hier eine falsche Übersetzung von "work" ist. Frederico zitiert ja, dass die Stelle „Featuring some of Martin McDonagh's finest work“ heißt – some und nicht one, also nicht bloß eine seiner besten Arbeitsergebnisse, sondern einige, womit demnach Szenen/Stellen des Films gemeint sein müssten, die die Leistung des Regisseurs sind. Der Film enthält Szenen, die zu den besten des Regisseurs gehören, und herausragende Hauptdarsteller. Der Knackpunkt, warum "work" für den ganzen Film gehalten wurde, war, dass some als "eins" übersetzt wurde. --Blobstar (Diskussion) 00:11, 13. Jan. 2023 (CET)
Mit einigen von Martin McDonaghs besten Leistungen --BlackEyedLion (Diskussion) 00:23, 13. Jan. 2023 (CET)
Eine abstraktere Übersetzung für "work", die auch geht und umgeht, dass Leistungen oder Ergebnisse in den Plural gesetzt werden, wäre "das Wirken/Schaffen/Arbeiten/die Arbeit" des Regisseurs (nicht die Arbeiten): „einiges seines besten Schaffens“/„einiges an McDonaghs bester Arbeit“. Vielleicht lohnt sich auch, „featuring“ tatsächlich wörtlich als Inge-Form des Verbs zu übersetzen „Einiges vom Besten seines Schaffens präsentierend“. Da es ja die Übersetzung eines Zitats ist und nicht wörtlich das Zitat, wie es ist, böte sich auch indirekte Rede an, um den "und"-Satz in zwei aufzulösen und aus dem Partizip ein Prädikat sei: „Der Konsens lautet, dass der Film mit zwei herausragenden Hauptdarstellern einiges des besten Schaffens McDonaghs zeige/aufweise und ein fein gemachter Schlechtfühl-Genuss sei.“ oder so ähnlich. [Add:] Die bloße Beiordnung ist allerdings nicht nuanciert genug und trägt nicht den Nebensinn, dass "some of his finest work" und die zwei Hauptdarsteller Faktoren dafür sind, dass der Film ein fein gemachter Schlechtfühl-Genuss ist. „Der Konsens lautet, dass der Film mit zwei herausragenden Hauptdarstellern, da er einiges an McDonaghs besten Schaffens enthalte, ein fein gemachter Schlechtfühl-Genuss sei.“ --Blobstar (Diskussion) 00:49, 13. Jan. 2023 (CET)
Ich habe den Satz nun zu „Mit einigen von Martin McDonaghs besten Arbeiten und zwei herausragenden Hauptdarstellern ist The Banshees of Inisherin ein fein gemachter Schlechtfühl-Genuss“ geändert. Kann sprachlich sicherlich noch besser ausgedrückt werden, ist aber erst einmal eine sinngemäßere Übersetzung. --Frederico34 (Diskussion) 22:50, 13. Jan. 2023 (CET)
Archivierung dieses Abschnittes wurde gewünscht von: Frederico34 (Diskussion) 22:50, 13. Jan. 2023 (CET)

Volksabstimmung über Lützerath

Guten Morgen, wäre es denkbar über strittige Themen wie zum Beispiel der Braunkohletagebau in Lutzerath spontan eine Volksabstimmung abzuhalten, aufgrund der vielen Demonstranten Ist ja davon auszugehen, dass die Mehrheit der Deutschen gegen den Abbau von Braunkohle dort ist? Wäre es möglich bereits in zwei Wochen das Volk zu befragen bevor Rewe Fakten dort schafft ? (nicht signierter Beitrag von 2A00:20:D009:A138:ED16:E566:1896:7F12 (Diskussion) 00:52, 15. Jan. 2023 (CET))

Hatten wir das nicht vor einiger Zeit schon Mal mit spontaner Volksbefragung? --Elrond (Diskussion) 01:08, 15. Jan. 2023 (CET)
Ja, hier. Über Maskenpflicht Das war aber eine IP aus Frankfurt am Main. --88.72.111.240 04:03, 15. Jan. 2023 (CET)
kann sein aber bei lützerath sehe ich halt Gefahr im Verzug wegen dem Klima, außerdem warum stoppen die Grünen also Klimaschutzpartei dass nicht einfach ? (nicht signierter Beitrag von 2A00:20:D009:A138:ED16:E566:1896:7F12 (Diskussion) 01:11, 15. Jan. 2023 (CET))
Zu Gefahr im Verzug haben wir einen Artikel. Du meinst das aber wahrscheinlich anders und ein dringender Handlungsbedarf besteht schon seit 1986. --88.72.111.240 04:50, 15. Jan. 2023 (CET)
Die Grünen stoppen das deshalb „nicht einfach“, weil es eben nicht einfach so geht. Zunächst mal leben wir immer noch in einer Demokratie mit all ihren Prozessen und Interessenabwägungen und nicht in einer grünen Diktatur, wo diese Partei einfach machen könnte, was sie will. Zu Lützerath gibt es Verträge, rechtsgültige Vorgänge und Gerichtsurteile, über die können sich die Grünen nicht einfach hinwegsetzen, selbst wenn sie mit absoluter Mehrheit im Bunztag säßen und die Regierung stellten. Was lernt ihr heute einklich in der Schule? Die Politik kann nichts „einfach mal so“ machen, sie kann zunächst mal auch nur Weichen stellen, damit der Zug – Wirtschaft und so – in die gewünschte Richtung rollt. Mal eben alles umkrempeln geht nicht, dafür sind die Zusammenhänge viel, viel, viel zu komplex. Zur Idee einer „spontanen“ Volksabstimmung: Dir ist schon klar, was für einen Aufwand es bedeutet, so eine durchzuführen? Also so, dass sie auch fälschungssicher und repräsentativ ist? --Kreuzschnabel 08:13, 15. Jan. 2023 (CET) Nebenbei: Ist dir schon mal der Gedanke gekommen, dass RWE und Rewe eventuell nicht ganz dasselbe ist? --Kreuzschnabel 08:16, 15. Jan. 2023 (CET)
Weil sie es nicht wollen. (Und die Stimmung scheint zu meinem Graus ohnehin immer stärker in Richtung der rechten Klimawandelleugner bzw. der "weiter-so-Fraktion" zu kippen) Allerdings hat die Verteidigung dieses Ortes ohnehin keinen Sinn mehr, und immerhin wurden mehrere trotz bereits laufender Umsiedlung gerettet. Generell findet die Zerstörung von Orten, Baudenkmälern und Bodendenkmälern aus reinen Wirtschaftsinteressen meine tiefste Ablehnung, sei es nun für Braunkohleabbau, den Bau/die Errichtung und Flutung von Stauseen oder handele es sich um den Abbruch einzelner Denkmale für den Neubau von Straßen oder Häusern. --2A0A:A541:FDDC:0:61AA:613A:151A:1ABA 01:39, 15. Jan. 2023 (CET)

Sind das wirklich über 40 Millionen Demonstranten? Ich bin generell kein Fan von Volksabstimmungen, halte das Thema aber für besonders ungeeignet. Eine einfache Frage "Soll hier gebaggert werden?" reicht nicht aus. Und bei einer komplexen Befragung mit dutzenden voneinander abhängigen Fragen blickt am Ende niemand mehr durch. --Carlos-X 01:42, 15. Jan. 2023 (CET)

Warum 40 Millionen? In Nordrhein-Westfalen gibt es ca. 13 Millionen Menschen, die bei einem Volksentscheid wahlberechtigt wären. Außerdem wäre es viel einfacher und ginge viel schneller, wenn die politisch Versammelten einfach ihr Taschengeld zusammenlegen und der RWE Gelände und Abbaurechte abkaufen. --Rôtkæppchen₆₈ 03:56, 15. Jan. 2023 (CET)
Die Frage war eigentlich, ob es denkbar wäre, "über strittige Themen (...) spontan eine Volksabstimmung abzuhalten". Die Antwort ist: Nein. Man kann das zwar denken aber der Gedanke führt in mehrere Sackgassen. Ein Grund liegt in dem Wort "spontan" und der damit verbundenen Vorstellung "in zwei Wochen". Ein zweiter Grund liegt in den Quoren und Mindestanforderungen für die einen Entscheid initiierenden Initiativen. Eine Abstimmung soll, wenn sie fair und demokratisch sein will, den Befürwortern und Gegnern Zeit für eine öffentliche Debatte geben, von einer relevanten Anzahl von Menschen gewünscht sein und im Vorlauf über ausreichend Zeit für die Vorbereitung verfügen. Das beginnt damit, dass alle Wahlberechtigten angeschrieben und informiert werden müssen. Wir haben dazu den Artikel Volksentscheid und in dem Abschnitt Rechtliche Grundlagen von Volksentscheiden in Deutschland kannst du erfahren, welche Voraussetzungen, Fristen und andere Anforderungen für ein solches Verfahren und welche Procedere bundesweit oder je nach Bundesland gesetzlich gelten oder auch nicht. Das konkrete Thema Lützerath sollte im Cafe weiterbequatscht werden. Das ist nicht Kern der Frage und bietet Stoff für endlose und für die Frage irrelevante Meinungsbekundungen. Weil die Fristen und anderen Rahmenbedingungen bei Volksentscheiden meist gute Gründe haben (siehe dazu im Vergleich auch ähnliche Regeln in Volksabstimmung (Österreich) und Volksabstimmung (Schweiz)) ist aus meiner Sicht hier alles Nötige gesagt. Ich setze deshalb mal eine Erle. --88.72.111.240 04:36, 15. Jan. 2023 (CET)
Archivierung dieses Abschnittes wurde gewünscht von: 88.72.111.240 04:36, 15. Jan. 2023 (CET)

E-Auto-Kombis

Gibt es in D schon echte Kombis als E-Auto ? Bislang sehe ich in der Werbung nur SUVs. -- Juergen 217.61.201.10 23:18, 10. Jan. 2023 (CET)

Reparier Deine Suchmaschine und gib dann kombi bev ein. --Rôtkæppchen₆₈ 23:29, 10. Jan. 2023 (CET)
Das einzige Ergebnis aus meiner Suchmaschine ist ein Roewe Ei5 alias MG5, von dessen chinesischem Hersteller ich noch nie gehoert habe.
Heisst das, dass es von den bekannten Automarken bislang tatsaechlich kein Angebot gibt ? (Meine Frage bezieht sich nicht auf Plug-In-Hybriden, sondern auf echte E-Autos ohne Benzin.) -- Juergen 217.61.201.10 00:35, 11. Jan. 2023 (CET)
Die PKW-Varianten des Citroën Berlingo gibt es sogar nur noch als E-Auto - oder zählt Hochdachkombi nicht unter "echter Kombi"? --Rudolph Buch (Diskussion) 04:31, 11. Jan. 2023 (CET)
Mittlerweile sind ja viele Modelle in der Pipeline. Es bleibt aber die Frage, warum die E-Auto-Hersteller allesamt die Kombi-Freunde jahrelang so sträflich vernachlässigt haben. Ein Grund soll sein, dass sich SUVs auf dem Weltmarkt verkaufen lassen während Kombis eine weitgehend "deutsche Marotte" sind.--Doc Schneyder Disk. 11:26, 11. Jan. 2023 (CET)
Zumindest in den 1990ern und 2000ern waren in Skandinavien, speziell Schweden, PKW allgemein, sehr häufig der Schwedentraktor Volvo in der Kombivariante unterwegs. OK, auch kein Weltmarkt. --Elrond (Diskussion) 16:01, 11. Jan. 2023 (CET)
Anderswo sind eben eher Fahrzeuge wie der F-150 populär, die es mittlerweile sowohl als Hybrid, als auch als BEV gibt. --Rôtkæppchen₆₈ 16:24, 11. Jan. 2023 (CET)
Unattraktive Zielgruppe? Beim E-Auto geht es nicht um Vernunft, sondern um ein quasi-religiöses Verkaufsargument, funktionierende Maschinen - vom Steuerzahler bezuschusst - vorzeitig auszutauschen. Der (deutsche) Kombi ist das Sinnbild der vernünftigen Nutzbarkeit im Bereich Auto. --Wikiseidank (Diskussion) 14:08, 11. Jan. 2023 (CET)
+1 - Und das ausgelöst unter einer Staatscheffin, die in Physik promoviert hat (und es somit eigentlich besser weiss). --46.114.4.238 15:57, 11. Jan. 2023 (CET)
Es gibt keine Abwrackprämie. Das Ziel ist nicht, funktionierende Autos vorzeitig auszutauschen, sondern, dass wenn schon ein neues Auto gekauft wird, dass das dann ein E-Auto sein soll. --Digamma (Diskussion) 18:35, 11. Jan. 2023 (CET)

Die nächste Merkwürdigkeit: Als Plug-in-Hybrid gibt es zahlreiche Kombi-Modelle: Mercedes C-Klasse T-Modell Plug-in-Hybrid, Opel Astra Sports Tourer Plug-in-Hybrid, Cupra Leon ST Plug-in-Hybrid, Peugeot 308 Plug-in-Hybrid, BMW 5er Touring Plug-in-Hybrid, Skoda Superb Combi iV Plug-in-Hybrid, Mercedes E-Klasse T-Modell Plug-in-Hybrid, VW Arteon Shooting Brake Plug-in-Hybrid, BMW 3er Touring Plug-in-Hybrid (nicht signierter Beitrag von Dr. Peter Schneider (Diskussion | Beiträge) 16:55, 11. Jan. 2023 (CET))

Der MG5 ist ein echter Kombi und steht hier beim Mazda-Händler. --Ailura (Diskussion) 18:38, 11. Jan. 2023 (CET)

Das Nutzungsprofil eines Kombis passt nicht zu den tatsächlichen Fähigkeiten eines E-Autos bei dem derzeitigen Entwicklungsstand. Einen Kombi nutzt man in der Regel für lange Reisen mit der Familie - nur reicht die Reichweite eines E-Autos in der Regel nicht für eine Fahrt zum Urlaubsort, erst recht nicht wenn das Auto vollbeladen mit Frau, Kindern und jede Menge Gepäck ist. Und 8 Stunden Aufladen auf einem Rastplatz irgendwo auf dem platten Land kann man mit Kindern im Schlepptau nicht zumuten. Hingegen wird ein SUV tatsächlich in der Regel nur für die Fahrt zur Arbeit und zurück verwendet, da spielt die Reichweite dann nicht so eine große Rolle. -- 2A02:908:121:6600:89A:EB79:B648:4F79 22:50, 12. Jan. 2023 (CET)

Das Nutzungsprofil eines Kombis ist nicht belegt. M.E. fahren viele einen Kombi, weil sie im Einzelfall auch mal Möbel von Ikea nach Hause fahren können wollen, was mit einer Limosine nicht ginge. Also nichts mit Urlaub. Ich kenne Männer, die wegen des Nichtvorhandenseins von E-Kombis doch einen Verbrenner gekauft haben. (Im Einzelfall spielte auch die Mietsituation und fehlen individuelle Lademöglichkeit eine Rolle) Lg --Doc Schneyder Disk. 23:01, 12. Jan. 2023 (CET)
Aber das es derzeit noch kein reines E-Auto auf dem Markt gibt, was hohe und auch voluminöse Zuladung plus hohe Reichweite >600 km garantiert, ist wohl unstrittig. Nutzungsprofil hin oder her, die klassische Familienkutsche gibt es als E-Auto derzeit nicht. Ob Hochdachkombi oder Kombi ist dabei fast wurscht, den Kunden interessiert Zuladung und Reichweite. Derzeit steht dem wohl vor allem das Akkugewicht im Weg. Vorsorglich: wenn ich die Werbung zum MG5 lese, steht da was von Reichweite bis zu 400km. Heißt realistisch 350 oder noch weniger. Das habe ich mit meinem Erdgas-HDK seit über 15 Jahren. Dazu ein Einstiegspreis von 32.000 €, bezahlbare Energiewende stellt sich der deutsche Michel sicher bissl anders vor.--scif (Diskussion) 08:09, 13. Jan. 2023 (CET)

Eine "echtes" E-Auto (BEV) muss in der Regel von Anfang an als E-Auto konzipiert werden, weil die Batterien Platz benötigen. Diese lassen sich sehr gut im Fahrzeugboden platzieren. Dann wird das Fahrzeug aber insgesamt höher - und sieht aus wie ein SUV. Frage: Wie stuft man VW ID.3 oder Škoda Enyaq iV ein? Lt. Wikipedia sind das sehr wohl Kombis...

Wenn ein klassischer Verbrenner umgebaut wird, werden die Batterien meist im Kofferraum untergebracht. Beim Kombi würde sich da eine extrem unpraktische Zerklüftung ergeben, die den Kombi als Lastenesel unattraktiv macht. Damit stellt sich die Frage, ob es dafür überhaupt einen Markt gibt. Und damit sind wir bei der zweiten Problematik: Die Hersteller haben immer noch Engpässe bei der Verfügbarkeit von Batterien. Sie sollten sie also so verbauen, dass sie damit möglichst profitabel unterwegs sind. Einen Verbrenner auf E-Antrieb umzustellen ist teuer. Das macht also nur Sinn, wenn anschließend so viele Fahrzeuge verkauft werden, dass die Investitionen sich rechnen. Wenn der Markt für umgebaute Verbrennerkombis nicht groß genug ist, lohnt es sich nicht, die Ingenieure mit der Umkonstruktion zu beauftragen. Anders formuliert: Wenn die Hersteller dort eine Marktlücke vermuten würden, hätten sie ein entsprechendes Angebot im Programm. Da sie aber lieber SUVs bauen (bzw. das, was Du für SUVs hältst), scheint die Marktlücke nicht groß genug zu sein. P.S.: Der Kofferraum eines Enyaq ist nicht kleiner als der eines C-Klasse-Kombis. --Vertigo Man-iac (Diskussion) 08:26, 13. Jan. 2023 (CET)

Nach googln habe ich ein Auto gefunden, dass mich gestern (ich auf dem Fahrrad) überholte. Wie viele Autos werden denn von einem Porsche Taycan E verkauft? --Wikiseidank (Diskussion) 09:20, 13. Jan. 2023 (CET)
Siehe Porsche Taycan#Absatzzahlen. --Rôtkæppchen₆₈ 09:26, 13. Jan. 2023 (CET)
Die Entwicklungskosten, insb. für die BEV-Technik, können sogar auf noch mehr Fahrzeuge verteilt werden, weil der Audi e-tron GT die gleiche Plattform nutzt. Beide Modelle werden im Hochpreissegment angeboten. Es ist also davon auszugehen, dass die Marge auskömmlich ist.--Vertigo Man-iac (Diskussion) 09:31, 13. Jan. 2023 (CET)

Snookerfrage zum Masters

Beim Masters sind David Gilbert und Hossein Vafaei für gesperrte Spieler nachgerückt. Schaut man sich aber die Rangliste an, stehen die auf den Plätzen 18 und 19. Warum wurde nicht der auf Platz 17 stehende Gary Wilson als Nachrücker genommen?. Dazu habe ich nichts im Netz gefunden, auch nicht auf den Seiten der WST. Vielleicht weiß Snookerado etwas dazu. --Elrond (Diskussion) 23:50, 12. Jan. 2023 (CET)

Der Stichtag für die Turnierteilnahme war nach dem UK Championship. Das hat man auch für die Nachrücker beibehalten. --Carlos-X 00:12, 13. Jan. 2023 (CET)
Herzlichen Dank!. Das hatte ich so nicht mitgekriegt. Schade für Wilson. --Elrond (Diskussion) 08:52, 13. Jan. 2023 (CET)

ΚΟΙΝΟΝ ΔΩΡΟΝ: Studies and Essays in Honour of Valery P. Nikonorov on the Occasion of His Sixtieth Birthday presented by His Friends and Colleagues / Compiled and ed. by Alexander A. Sinitsyn and Maxim M. Kholod. — St. Petersburg: St. Petersburg State University Faculty of Philology, 2013.

Wo ist das Buch ISBN 978-5-8465-1362-4 zu finden? --Hamburgum (Diskussion) 07:50, 13. Jan. 2023 (CET)

In Warschau gibt es die lateinisch betitelte englische Fassung dieses Artikels: https://www.worldcat.org/title/899859299 . --Rôtkæppchen₆₈ 08:37, 13. Jan. 2023 (CET)

Mikojan-Gurewitsch MiG-21

 
 
MiG-21 on display on top of Verkkokauppa in Helsinki

Wie nennt man die lange Stange vorne und vor allem wozu ist die gut? Das sieht nicht nach einer Rakete aus... --2A01:598:B8B7:909B:50A4:F1F9:6442:2FD8 14:12, 9. Jan. 2023 (CET)

Das ist ein Pitotrohr. --Rôtkæppchen₆₈ 14:44, 9. Jan. 2023 (CET)
Bei der ersten Variante (wie hier) der MiG-21 ist das Staurohr an der Unterseite der Nase angebracht. Nach der MiG-21P-Variante ist dieses Rohr oben am Lufteinlass angebracht. Spätere Versionen verschoben den Befestigungspunkt des Pitotrohrs um 15 Grad nach rechts, vom Cockpit aus gesehen, und hatten einen Notfall-Pitotkopf auf der rechten Seite, direkt vor dem Baldachin und unterhalb der Augenlinie des Piloten. --2003:D5:FF4C:5500:BCCC:C7E3:5D7A:C1CD 14:48, 9. Jan. 2023 (CET)
Kurz gegoogelt: Das ist das Pitotrohr für Überschallflug. Ein zweites „normales“ sitzt rechts außen am Cockpit. Das lange Rohr ist aber nicht nur ein Pitotrohr; die im unteren Bild sichtbaren „Flossen“ übermitteln auch die exakten aktuellen Anströmwinkel (horizontal und vertikal) an den Zielcomputer, der sie einkalkulieren muss. --Kreuzschnabel 17:09, 9. Jan. 2023 (CET)
Handelt es sich bei der "langen Stange" (grün im zweiten Foto) nicht um den sogenannten "Nasenmast", an dessen Spitze sich das Pitotrohr und anderen Aufnahmen für die Strömungssensorik (silbern bzw. grau) befinden? --Burkhard (Diskussion) 22:12, 9. Jan. 2023 (CET)
Die Bezeichnung „Nasenmast“ fiel noch nicht, aber alles andere wurde schon gesagt, ja. --Kreuzschnabel 01:27, 10. Jan. 2023 (CET)
Fast. Dass das Pitotrohr den Staudruck misst, aus dem sich die Fluggeschwindigkeit berechnen lässt, ist wohl schon klar. Die Luft, die den Flugzeugrumpf umströmt, hat aber u.U. eine andere Geschwindigkeit als die Luft in einiger Entfernung zum Rumpf. Daher positioniert man das Pitotrohr häufig vorne auf der Bugnase in einem "noch unverfälschten" Luftbereich. Die MiG 21 hat gar keine Bugnase, weil das Triebwerk im Rumpf sitzt. Am Rumpf käme zur veränderten Umströmung noch ein luftansaugendes Strahltriebwerk hinzu, also keine gute Position. Daher diese lange Stange, die irgendwie so aussieht, als hätten die Konstrukteure diesen Punkt nicht so richtig durchdacht.
Was mich etwas stutzig macht: die F 35 hat zwei kleine Pitotrohre vorne links und rechts am Bug. Aber vielleicht ist da die Elektronik schon so weit, dass sie mögliche Fehler herausrechnet. --Optimum (Diskussion) 20:08, 10. Jan. 2023 (CET)
Die MIG 35 hat nur eins unter der Nase. Daß sowas vor 60 Jahren anders aussah, ist kein Wunder. Ralf Roletschek (Diskussion) 11:40, 11. Jan. 2023 (CET)
Hier sieht man sie beide. Oder was ist das sonst? --Optimum (Diskussion) 12:49, 11. Jan. 2023 (CET) Ach so, Du meintest MIG.
Ja, ich meinte die MiG. Man muß schon halbwegs aktuelle Modelle vergleichen. Der Starfighter hatte eine ähnliche Stange an der Nase - nur mittig, weil seitliche hintere Lufteinlässe. --Ralf Roletschek (Diskussion) 23:28, 11. Jan. 2023 (CET) Es ist auch typisch russisch, daß solchen Sachen keine besondere Bedeutung zugemessen wurde. Klaut jemand die Stange, fliegt der Vogel trotzdem, nur eben ohne Tacho ;) Fliegen und schießen muß es, alles andere ist nebensächlich. Also wird der Vogel erstmal auf Fliegen und Schießen entwickelt, bis jemand kommt und meint, man müsse ja auch die Geschwindigkeit messen. Gut, dann schweiß dein Rohr da irgendwo dran!

Der Nasenmast (engl. "nose boom", allgemeiner auch "en:Air data boom") ist weder eine russische Eigenheit noch obsolet, siehe z. B. North American X-15, Rockwell-MBB X-31 oder aktuell an dieser 2020 zum Forschungsflugzeug umgerüsteten Reims-Cessna F406 der TU Braunschweig (https://magazin.tu-braunschweig.de/m-post/update-zum-neuen-forschungsflugzeug/20160229_wipaff_bild1). Er kommt immer dann zum Einsatz, wenn die Datenaufnahme möglichst unbeeinflußt von der am Rumpf anliegenden Strömung erfolgen soll bzw. muss (nicht alles lässt sich "rausrechnen").

--Burkhard (Diskussion) 21:33, 13. Jan. 2023 (CET)

Auf welchen Wegen gelangen USA-Panzer an die NATO-Ostflanke?

Auf welchen Wegen gelangen USA-Panzer an die NATO-Ostflanke? Spielt Bremerhaven dabei auch eine Rolle (wie bei den 'Militärübungen')? --Gelöbnix (Diskussion) 20:00, 11. Jan. 2023 (CET)

Ich bin mir nicht sicher, ob die USA es lustig fänden, wenn potentielle militante Putinversteher den genauen Transportweg zwecks Festlegung optimaler Hinterhalte einfach hier in der WP:Auskunft nachlesen könnten. Im Klartext: Das werden die planenden Militärs nicht an die große Glocke hängen. --Kreuzschnabel 20:36, 11. Jan. 2023 (CET)
Genau, deshalb lässt sich sowas auch nur aus putinnahen Medien mit Neigung zum Landesverrat entnehmen, zum Beispiel hier oder ganz aktuell heute hier. --Rudolph Buch (Diskussion) 20:46, 11. Jan. 2023 (CET)
Ich vermute hier Ironie. Das sind natürlich Fake News. ;-) --95.116.66.241 21:28, 11. Jan. 2023 (CET)
Geheimhaltung wird erst relevant, wenn die Waffen Natogebiet verlassen haben. Putin wird die Waffen wohl kaum auf Natogebiet zerstören wollen. --31.212.212.83 23:52, 11. Jan. 2023 (CET)
Frachtschiffe, Züge und Schwerlasttransporter. sуrcrо.педія 20:43, 11. Jan. 2023 (CET)
D.h. der Hafen von Bremerhaven (Interview Kujat) rangiert da unter ferner liefen? --Gelöbnix (Diskussion) 23:25, 11. Jan. 2023 (CET) PS: Und jeder US-Panzer in Europa ist automatisch ein NATO-Panzer?
Was genau ist eigentlich das Problem? Haben wir bei der BW NATO und Nicht-Nato-Gerät? Stellst du dich so ahnungslos oder hast du sie wirklich nicht? Ist jemand ernsthaft der Meinung, das es die Presse braucht, um die Russen davon zu informieren, dass da was Großes übers Wasser kommt? Und bist du wirklich so naiv? Das ein Hafen, in dem 40 Jahre die Amerikaner stationiert waren, keine Vorrichtungen mehr für Anlandungen von Truppen hat? Was ist denn konkret der Hintergrund deiner Fragen?--scif (Diskussion) 00:44, 12. Jan. 2023 (CET)
Ist jeder US-Panzer in Europa ein NATO-Panzer und wurde auch von dieser angefordert? --Gelöbnix (Diskussion) 02:05, 12. Jan. 2023 (CET)

Bevor du solche Fragen stellst, solltest du dich vielleicht mal mit der NATO und dem NATO-Stationierungsabkommen befassen. Ich traue dir diese Transferleistung zu. Wenn Truppen im Rahmen der NATO dauerhaft stationiert sind, muß ich nichts anfordern. Es sei denn, du begibst dich in das Lager der VT und meinst, das wir immer noch die amerikanische Besatzungszone haben. Angesichts der derzeitigen milit. Lage im Osten des NATO-Beitrittsgebietes ist es ja nicht fern der Realität, das NATO-Partner diese Truppen zum Schutz angefordert haben. Wer natürlich meint, das die Russen mit Wattebällchen schießen, nun denn. Es sollte auch zur Kenntnis genommen werden, das es zur Präsenz amerik. Truppen in Europa im Rahmen des NATO-Bündnisses speziell bei den osteuropäischen NATO-Mitgliedern etwas andere Auffassungen als deinen POV geben mag. Darüber hinaus sind Geographiekenntnisse auch von Vorteil: sowohl Bremerhaven als auch Antwerpen liegen an der Nordsee, also nicht im sofortigen Wirkungsbereich der Baltischen Flotte. Um Irritationen zu vermeiden, ist es daher denkbar, das man nicht explizit in deutschen, polnischen oder baltischen Ostseehäfen anlandet. Zudem ist die Verlegung solcher Truppen per Schiene oder Achse auch ein Test für die dt. Infrastruktur. Deutschland will sich ja nicht zuletzt wegen seiner zentralen Lage zum Cheflogistiker innerhalb der NATO etablieren. Es ist letztlich ein Unterschied, ob Schiffe in internationalen Gewässern provoziert oder angegriffen werden oder Züge auf dem Territorium eines souveränen Staates. Über allem steht natürlich die Grundfrage, auf wessen Seite man steht. Wer natürlich immer noch meint, das im Rahmen einer militär. Spezialoperation bösartige Nazis in der Ukraine mit Angriffen auf die Zivilbevölkerung vernichtet werden sollen bzw die Ukraine nie ein souveräner Staat war und ganz natürlich zu Rußland gehört, nun da ist dann für mich die Kommunikation erschöpft. Was ich dir aber per se nicht unterstelle. Es ist aber niemandem verboten, bei der Thematik selber zu denken und zu recherchieren.--scif (Diskussion) 08:01, 12. Jan. 2023 (CET)

Vereinfacht gesagt: Man kann "sichere" Häfen am Atlantik und den damit verbundenen Meeren nutzen und dann per Bahn (gerade wenn es um Panzer geht, per Radlader ist das immer so ein Ding, ginge im Notfall aber wohl teilweise auch) an den Einsatzport transportieren. Ob nun Bremerhaven oder wie gerade das niederländische Vlissingen [5], nutzt man eben das was zur Verfügung steht und die entsprechende Infrastruktur hat. Bis 1989 hatte man eben in Bremerhaven den Bremerhaven Port of Embarkation, die Navy (wohlgemerkt nicht die Army) hat heute sonst noch Einrichtungen in Italien und Spanien (aber eben nicht notwendigerweise geeignet für Panzertransport). Und natürlich hat man auch den Luftweg zur Verfügung (kommt halt auf die Menge an die man verschiffen will), was dann eben vorwiegen Rammstein in Deutschland wäre (über Notfall mit Autobahnen reden wir ja gar nicht). Und generell macht man Übungen wohl nicht zum Spass [6]. Dort kann man die Standardwege sich ja gut anschauen. Also ja, Bremerhaven spielt weiterhin eine Rolle bei sowas, genauso wie viele andere Häfen.--Maphry (Diskussion) 08:47, 12. Jan. 2023 (CET)

Das ist doch alles unstrittig. Mich irritiert das NATO-Bashing. Ramstein wäre ja dann eher was für Manpower, Abrams einzeln zu fliegen wäre sehr aufwändig. Autobahnen sähe ich nur für den Panzertransport als Notfall, alles was luftbereift ist, fährt regelmäßig Autobahn. Das größte Problem auf deutschen Autobahnen ist eh unsere Polizei. Es wurden schon polnische Tieflader mit amerik. Panzern rausgezogen, weil irgendwelche Genehmigungen fehlten, kein Witz. Denkbar sind zukünftig vielleicht wieder Anlehnungen an REFORGER.--scif (Diskussion) 09:01, 12. Jan. 2023 (CET)

Und auch das ganze Geraune von scif u.a. ("Haben wir bei der BW NATO und Nicht-Nato-Gerät?") zeugt von Ahnungslosigkeit, die umgekehrt dem Fragesteller unterstellt wird. Nur weil Deutschland Mitgliedsstaat der NATO ist, ist nicht das komplette Gerät der Bw NATO-Gerät. Das gilt auch für die US-Streitkräfte. Die jeweiligen Mitgliedsstaaten sind nach bestimmten Schlüsseln und Bereitschaftserklärungen verpflichtet, Truppen, Material und "Fähigkeiten" der NATO zur Verfügung zu stellen. Diese Panzer, um bei der Frage zu bleiben, würden direkt der NATO-Kommandostruktur unterstellt und sind damit "NATO-Panzer", die NATO selbst nennt das "assigned". Dann gibt es Panzer, die für den Fall der Fälle reserviert sind, die NATO spricht von "earmarked". Das könnte man auch noch als "NATO-Panzer" bezeichnen. Aber kein Mitglied der NATO unterstellt alle seine Truppen der NATO! Das ist auch sehr wichtig, denn wenn zum Beispiel die Bundeswehr im Inneren der BRD im Katastropheneinsatz ist, dann kann sie dafür nicht wertvolles Gerät einsetzen, das im Notfall die NATO braucht. Sie muss dafür Großgerät und Personal nehmen, das entweder von Anfang an nicht der NATO unterstellt war oder aber für diesen Zweck bei der NATO abgemeldet wurde. Das Territoriales Führungskommando der Bundeswehr führt per Definition nur Truppen, die keine NATO-Truppen sind. Die US-Streitkräfte haben ähnliche Unterscheidungen. Wenn zum Beispiel US-Truppen außerhalb des Bündnisgebiets aktiv sind (Afrika, Pazifik), dann nehmen die USA dafür Einheiten, die ausdrücklich nicht der NATO assigniert oder earmarked sind. Sonst würden sie nämlich die Soll-Stärke der NATO schwächen, indem sie eingeplante Kräfte anderweitig einsetzen. Jedes NATO-Mitgliedsland hat eine Spielmasse von Kräften für eigene Zwecke. Und vor allem bedeutet das auch, dass NATO-Staaten untereinander bilaterale Vereinbarungen über die Stationierung von Truppen treffen können, die absolut nichts mit der NATO zu tun haben. Dazu kommen noch EU-Missionen. An EUFOR Althea sind beispielsweise auch NATO-Staaten beteiligt, die militärische Kräfte nach Osteuropa (außerhalb des NATO-Bündnisgebiets!) schicken, ohne dass das NATO-Truppen sind. Ganz im Gegenteil, die NATO würde denen kräftig Ärger machen, wenn sie sich erlauben würden, NATO-Truppen dorthin zu schicken, dann fehlt der NATO nämlich was in ihrer Planung. Es ist also eine vollkommen berechtigte Frage danach, wie viele der US-Truppen in Polen oder anderen osteuropäischen Staaten nun NATO-Truppen ("NATO-Panzer") sind, und wie viele per bilateralen Abmachungen dort sind. Der Unterschied ist für die Kräfteplanung der NATO von extrem großer Relevanz.--2A02:8109:BD40:65C4:6445:2181:EF7D:53B4 11:25, 12. Jan. 2023 (CET)

Und, dann wirst du uns ja in deiner großen Weisheit gleich verkünden, wieviel NATO-Panzer denn da jetzt rollen. Mit diesem ganzen Unterstellungskram wirst du sicher recht haben. Ob das dann die Gegenseite groß interessiert, welcher Panzer wem unterstellt ist, steht auf nem anderen Blatt. Für die zählt erstmal das Gerät. Und im Bündnisfall ist für solche Feinheiten eh keine Zeit mehr. Zudem läßt sich die Frage nach der Aufschlüsselung in Nato und bilaterales Gerät nur mit EInblick in die bilateralen Vereinbarungen beantworten. Sind die öffentlich?--scif (Diskussion) 11:44, 12. Jan. 2023 (CET)
Bei all Deiner Expertise (die diversen Unterstellungen habe ich mal überlesen): Die Frage ist doch, ob die NATO einen solchen Aufmarsch von US-Truppen und einen langjährig herbeidiplomatierten Krieg zwischen den USA und Russland überhaupt mitmachen soll? --Gelöbnix (Diskussion) 12:29, 12. Jan. 2023 (CET) PS: Ein solches Verhalten in einer US-finanzierten Kulisse schiene mir jedenfalls nuttig.
Bei all deiner Expertise: und einen langjährig herbeidiplomatierten Krieg zwischen den USA und Russland Ahja. Und du möchtest ernstgenommen werden? Hab ich was verpaßt in Bezug auf die Krim oder in Bezug auf den Marsch auf Kiew? Und wo steht geschrieben, dass ein NATO-Land seeelenruhig zuschauen muß, wenn ein direkt angrenzender Nachbar Krieg treibt.--scif (Diskussion) 13:03, 12. Jan. 2023 (CET)
Wenn man weiter denkt ist schon die Diplomatie Schuld. Hätte die Ukraine nicht den "diplomatischen Versprechungen" geglaubt Russland würde die Grenzen respektieren und daraufhin seine Atomwaffen abgegeben hätte Russland sich Sicherlich nicht getraut die Ukraine zu überfallen. Daher scheint es mir klug diesbezüglich auf Diplomatie zu pfeifen und sich Atomwaffen zu besorgen. Die sind wohl eine höhere Sicherheitsgarantie für den einzelnen Staat als diplomatische Versprechungen. ...Sicherlich Post 13:10, 12. Jan. 2023 (CET)
bemerke gerade; hier ist die Auskunft! Das triftet doch etwas richtung WP:Cafe ab. Bitte ggf. dort weiterführen und hier bei Sachfragen bleiben ...Sicherlich Post 13:13, 12. Jan. 2023 (CET)
Triftet? --2A02:3030:80D:A878:8DC2:DF35:8B2C:A29F 22:04, 13. Jan. 2023 (CET)

Läßt sich das ungefähr in Prozenten ausdrücken: Wie viele der US-Truppen in Polen oder anderen osteuropäischen Staaten sind nun NATO-Truppen ("NATO-Panzer")? --Gelöbnix (Diskussion) 13:43, 12. Jan. 2023 (CET)

Screenshot-Programm für Grafiken, die größer als der Bildschirm sind.

Ich bin hier in der Familienforschung unterwegs. Ein Problem, bei der Darstellung von Ahnentafeln oder Stammbäumen ist immer wieder mal, daß diese in leserlicher Zoomstufe größer als der Bildschirm sind. Man muß also scrollen.

Gibt es ein Screenshot-Programm, das Grafiken größer als der Screen abspeichern kann (ich weiß, es wäre dann streng genommen kein "Screenshot" mehr)? Ich stelle mir da so vor: mit der linken Maustaste wird der linke, obere Punkt des gewünschten Bereichs definiert. Mit gedrückter linker Maustaste fahre ich dann zum rechten, unteren Punkt des gewünschten Bereichs. Sollte der rechte, untere Punkt außerhalb des Bildschirms liegen, soll das Familienforschungsprogramm automatisch "mitscrollen".

--2001:A61:24DD:C401:D0E9:3DA5:60AD:50F9 17:10, 12. Jan. 2023 (CET)

Vorschlag: (bei Firefox ): rechte Maustaste, "Bildschirmfoto aufnehmen", "ganze Seite speichern ", Kopieren oder Herunterladen . Grüße --2003:C6:1721:19D6:90B3:62E5:939D:C8C8 18:08, 12. Jan. 2023 (CET)
Leider läuft das Programm im Browser nicht. Trotzdem danke. --2001:A61:24DD:C401:99A5:173B:2324:FFD4 19:33, 12. Jan. 2023 (CET)
Das wesentliche beim Screenshot ist, daß das gespeichert wird, was auf dem Bildschirm dargestellt ist, ohne daß Funktionen der Programme verwendet werden, die gerade etwas auf dem Bildschirm darstellen. "... soll das Familienforschungsprogramm automatisch mitscrollen" funktioniert also nicht. Einen Export bietet das Familienforschungsprogramm wohl nicht. Wenn ein Rasterbild angezeigt wird, kann man das mit "Rechtsklick -> Grafik speichern" abspeichern (Firefox). Wenn es keine Rasterbild ist, wird von dem Programm vielleicht nur der Teil des Stammbaums erzeugt, der auf den Bildschirm paßt. Dann gibt es nichts "Verdecktes", was ein anderes Programm lesen könnte. 91.54.32.105 18:28, 12. Jan. 2023 (CET)
"Einen Export bietet das Familienforschungsprogramm wohl nicht." Nein, leider nicht. Trotzdem danke. --2001:A61:24DD:C401:99A5:173B:2324:FFD4 19:33, 12. Jan. 2023 (CET)
Das wesentliche beim Screenshot ist, daß das gespeichert wird, was auf dem Bildschirm dargestellt ist
Grundsätzlich ist das richtig, aber es gibt Möglichkeiten, Screenshots von Programmfenstern zu erzeugen, die größer als der zur Anzeige verwendete Monitor sind (ausreichende Hardwareleistung vorausgesetzt). Zumindest unter Linux ist es kein Problem, ob es unter Windows auch möglich ist, weiß ich nicht. Der Weg ist: Die X Oberfläche mit xrandr so groß definieren wie zur kompletten Darstellung des gewünschten Inhalts benötigt (in Pixeln). Der zugeordnete Monitor zeigt dann nur noch die linke obere Ecke der gesamten X Oberfläche an. Maximiert man nun ein Programmfenster, wird bei einem Screenshot im "fullscreen" Modus auch der auf dem Monitor nicht sichtbare Bereich mit aufgenommen, der gesamte gewünschte Inhalt wird also erfaßt. Bei dieser speziellen Konfiguration ist es erforderlich, die Programme anzuweisen, ihre Fenster standardmäßig im Sichtbereich des Monitors zu öffnen, um vernünftig arbeiten zu können. Wie gesagt, ob man das auch unter Windows so einfach realisieren kann, ist mir unbekannt, würde mich aber durchaus interessieren. --92.73.145.117 00:58, 13. Jan. 2023 (CET)
Genealogen nutzen das Genwiki. Dort findest Du: https://wiki.genealogy.net/Stammbaumdrucker. Damit kannst Du die Daten als PDF oder neu SVG (aktuelle Version 8.0.3.5 von 10.2022) exportieren und auf Rollen- oder Bannerware (bis 300 m) drucken oder auf 100 x 100 = 10.000 A4-Seiten aufteilen und zusammenkleben. Reicht auch für "grössere" Sippen, bis 60 Generationen... Die 60 € (privat) oder 480 € (gewerblich) sind den Leistungsumfang mehr als wert. Gruss, --Markus (Diskussion) 20:32, 12. Jan. 2023 (CET)
PS: das Handbuch gibt es hier, läuft auf Windows, OSX und Linux unter Wine. Datenbasis: GEDCOM, UTF-8, incl. Gedcom-Konverter, Import-Präprozessor und Export-Postprozessor und Konvertierungsprotokoll. Cooles Teil! Gruss, --Markus (Diskussion) 20:58, 12. Jan. 2023 (CET)
Interessanter Link, werde ich mir mal genauer ansehen. Vielen Dank.--2001:A61:24DD:C401:24D1:7EFE:1AE7:7245 21:10, 12. Jan. 2023 (CET)
Bin etwa spät, aber Screenpresso kann etwas das sich "Screenshot mit Scrollen" nennt, da kann man ein Fenster weiterscrollen und so einen in einer Richtung längeren Screenshot erzeugen. Funktioniert auf Anhieb nur so lala, aber mit etwas Übung geht es halbwegs. --Windharp (Diskussion) 10:02, 13. Jan. 2023 (CET)
Hi Windharp, eigentlich genau, was ich suche, aber eben nur eigentlich. Meine Grafiken sind oft so groß, dass ich quer und längs scrollen muss und das Programm unterstützt leider nur Scrollen in einer Richtung ("und so einen in einer Richtung längeren Screenshot erzeugen". Ich danke dir trotzdem. Das im Programm verlinkte Youtube-Tutorial ist übrigens eine Katastrophe. --2001:A61:24DD:C401:4529:AD4F:AE8F:9076 14:24, 13. Jan. 2023 (CET)

FFP2-Maske

Im Artikel Atemschutzmaske#Partikelfiltrierende Halbmaske (FFP-Maske) lese ich, dass die Masken nach prozentualer Durchlässigkeit beurteilt werden. Also für FFP2 sind das 11% Durchlässigkeit. Das bedeutet aber auch, dass eine Maske mit 1m² Filterfläche immer noch eine FFP2-Maske ist? --2001:9E8:A513:4000:340D:92D1:905:542F 18:54, 12. Jan. 2023 (CET)

Wir kennen zwar nicht Deine Physiognomie, aber Du wirst sicherlich 100%-ig wenigstens an Atemnot leiden, sollte sie dicht anliegen.--Caramellus (Diskussion) 20:42, 12. Jan. 2023 (CET)
Je größer die Fläche des Filters ist, umso geringer ist der Atemwiderstand. Dann strömt die Luft auch langsamer durch das Filter und die Rückhaltefähigkeit steigt. --BlackEyedLion (Diskussion) 22:23, 12. Jan. 2023 (CET)
Das gilt aber nur für das Ausatmen. --2001:9E8:A51F:3600:2B06:847F:E599:9F7B 16:17, 13. Jan. 2023 (CET)
Nein. Was @BlackEyedLion schrub, gilt natürlich für beide Richtungen. --31.212.244.31 16:39, 13. Jan. 2023 (CET)

Wieso so schwer zu finden?

Wieso haben sich in den Wissenschaften die Zitierweise mit Klammern, Namen und Jahreszahl durchgesetzt? Wie etwa "(Name Jahr)".
Es ist dadurch sehr schwer zu finden, besonders wenn der Name geläufig ist und im Jahr viele Publikationen von dieser Person herauskamen. Schon allein der Titel würde helfen. Weiß jemand da rat?--2A02:908:424:9D60:71E2:53D2:BCC3:150E 20:19, 12. Jan. 2023 (CET)

Weil es am Ende des Textes ein Literaturverzeichnis gibt + da nur ein Wissenschaftler mit der Jahreszahl vorkommt. Und wenn nicht, dann wird tatsächlich feiner differenziert. -- southpark 20:33, 12. Jan. 2023 (CET)
+1. Es gibt keine Schwierigkeiten, die Verweise aufzulösen, wenn man einfach ins Literaturverzeichnis schaut. Als Ergänzung aber noch: Die Frage beansprucht eine Absolutheit, die so nicht gegeben ist. Verschiedene Wissenschaften zitieren auch unterschiedlich. In vielen Geisteswissenschaften ist zum Beispiel "Name, Kurztitel" üblich.--2A02:8109:BD40:65C4:6445:2181:EF7D:53B4 20:49, 12. Jan. 2023 (CET)
Oder "Name, Jahr, Seitenzahl". Der entscheidende Punkt ist tatsächlich, daß es bei dieser Art von Quellenangabe immer ein Literaturverzeichnis gibt, entweder am Ende des Buches oder am Ende jedes Kapitels oder Abschnitts, in dem diese Kurzangaben eindeutig zugeordnet sind. Merkwürdigerweise habe ich genau diese Frage hier in der Auskunft schon häufiger gelesen: „Welches Werk ist hier gemeint ...”, und stets gab schon ein kurzer Blick ins Literaturverzeichnis des betreffenden Buches sofortigen Aufschluß. Wird dieses Wissen über den Umgang mit Fachliteratur heute nicht mehr in der Schule vermittelt? --92.73.145.117 22:30, 12. Jan. 2023 (CET)
Es ist auch nicht sinnvoll möglich, die Titel von Veröffentlichungen in den Volltext zu schreiben. Beispiele aus den letzten Veröffentlichungen in Nature: STEREO neutrino spectrum of U-235 fission rejects sterile neutrino hypothesis und A ligand insertion mechanism for cooperative NH3 capture in metal-organic frameworks. Wie soll man das im Fließtext unterbringen, insbesondere wenn mehrere Verweise aufeinander folgen? Es scheint dem Fragesteller aber auch nicht möglich zu sein, eine sinnvolle Überschrift zu erstellen. --BlackEyedLion (Diskussion) 22:49, 12. Jan. 2023 (CET)
Mag ja sein, dass der Fragesteller so seine Schwierigkeiten hat. Ich würde aber positiv hervorheben, dass er interessiert ist, diese Schwierigkeiten zu lösen. Jeder fängt mal klein an. Grüße, --Sokkok (Diskussion) 20:54, 13. Jan. 2023 (CET)
Autor-Jahr-Zitierweise#Regeln. --Christian140 (Diskussion) 08:49, 13. Jan. 2023 (CET)

Artikel "Seeadler" im 4. Absatz werden verschiedene Größen bei den Weibchen angegeben..Außerdem sind die Größen einmal in Zentimeter und dann in Millimeter angegeben.

Der 4. Absatz vom Artikel "Seeadler" muss überarbeitet werden.

--2003:E4:270E:8200:DD80:7F4C:CEF4:3A13 10:19, 13. Jan. 2023 (CET)

Dann mal ran an die Arbeit! -Ani--46.114.155.174 10:50, 13. Jan. 2023 (CET)
Dabei aber nicht Flügellänge und -spannweite verwechseln... --Magnus (Diskussion) 10:55, 13. Jan. 2023 (CET)
In welchem Artikel genau? --31.212.244.31 12:51, 13. Jan. 2023 (CET)

Fehler in Motorradelektrik

Bei einem 125er-Honda-Motorrad wird die (überprüfte fehlerfreie) Batterie nicht mehr geladen, die Ausgangsspannung des Reglers ist zu niedrig. (Info: Die "Lichtmaschine" besteht aus einer dreisträngigen Ankerwicklung und wird permanent erregt, also die übliche Konstruktion.) Den Werkstätten fällt nichts Besseres ein, als Ankerwicklung und Regler versuchsweise kostenpflichtig austauschen zu wollen, ohne dem Fehler zuvor meßtechnisch auf den Grund zu gehen - die Original-Ersatzteile kosten jeweils rund 300 Euro. Der Halter hat, wie im Werkstatthandbuch empfohlen, in die Anschlußleitung der Lichtmaschine hinein die drei Gleichstromwiderstände bei jeweils offener dritter Leitung gemessen und keine Auffälligkeiten (Abweichungen von den Sollwerten im Handbuch) feststellen können. Im Betrieb wird die Anschlußleitung von der Lichtmaschine zum Regler ziemlich warm, da fließen also kräftige Ströme, ohne daß der Regler gleichstromseitig Ladeleistung abgeben würde. (Meiner Empfehlung, die Ankerströme und -spannungen zu oszillographieren, konnte mangels Meßmitteln nicht nachgekommen werden.) Wenn man an der Lichtmaschine bei laufendem Motor testweise drei 230-V-60-W-Glühlampen im Dreieck als Last anschließt, leuchten die schwach - geschätzte Spannung deutlich unter 50 V. (Kfz-Glühlampen sind als Testlast nicht geeignet, weil die dabei durchbrennen.) Da es relativ unwahrscheinlich ist, daß an der Lichtmaschine selbst etwas kaputtgeht, was bei der Gleichspannungswiderstandsmessung nicht sofort auffällt - in Frage kommen eigentlich nur Wicklungsunterbrechungen oder Windungsschlüsse; ein Masseschluß liegt auch nicht vor - fiel der Verdacht auf einen defekten Regler. Es wurde über eBay bei einem Händler aus Lettland ein Ersatzregler für an die 40 Euro bestellt, dessen Einbau keine Besserung bzw. Veränderung brachte. Meine Annahme war, daß sich im Regler - hineinschauen kann man nicht, der ist vergossen - eingangsseitig ein dreisträngiger Brückengleichrichter, bestehend aus insgesamt sechs Dioden, befindet, und an dessen Ausgang ein Längsregler angeschlossen ist. Deshalb empfahl ich, an den Eingang des Reglers jeweils in den sechs möglichen Positionen bzw. Polaritäten den Ausgang eines Labornetzteils mit 15-20 V Spannung anzuschließen - in jeder Lage müßte der Regler dann am Ausgang die korrekte Ladespannung von etwas über 14 V bereitstellen. (Ein Drehstromtransformator zur Simulation der Lichtmaschine stand leider nicht zur Verfügung.) Dabei stellte sich heraus, daß der Ersatzregler an zwei von drei Strängen eingangsseitig einen Kurzschluß aufwies, also: Kurzschluß zwischen L1 und L2 sowie zwischen L1 und L3, nicht aber zwischen L2 und L3; wenn man zwischen letztere Gleichspannung anlegt, gibt der Regler am Ausgang die bestimmungsgemäße Spannung aus. (Interessanterweise ergab die "Gegenprobe" bei dem alten Regler, daß der bei allen sechs Möglichkeiten korrekt arbeitet. - Meine Vermutung: der ist gar nicht defekt, aber das Ersatzteil aus Lettland ist ein Boogie part. Sowas kommt vor: wenn bei einer Wareneingangsprüfung zu viele defekte Exemplare auffallen, wird die Lieferung nicht angenommen. Der Hersteller prüft die Teile dann aber nicht einzeln und verschrottet sie auch nicht, sondern verkauft die ganze Palette an entsprechende Händler. Für deren Kunden ist das nun ein Glücksspiel, ob sie ein defektes oder einwandfreies Exemplar erwischen.) Bleibt das ungelöste Ausgangsproblem. Es deutet ja nun alles auf einen defekten Anker hin, aber so richtig kann ich mir die Symptome auch nicht erklären. (Für die Blitzmerker: Die Frage ist natürlich, was wahrscheinlich Sache ist und sinnvollerweise etatschonend getan werden sollte, um den Fehler zu beheben.) --77.10.59.113 19:38, 11. Jan. 2023 (CET)

"... und an dessen Ausgang ein Längsregler angeschlossen ist ..." Eher nicht. Dann würde die Eingangsspannung bei geringer Last stark hochlaufen. Bei mittlerer Ausgangslast müsste der Regler hohe Verlustleistungen abführen. Normalerweise schließen solche Regler die Eingangsspannung kurz, wenn die Ausgangsspannung zu hoch wird (Shuntregler). Teste mal, ob die eingangsseitigen Kurzschlüsse bei geringerer Spannung auch noch vorhanden sind. Was passiert, wenn der Eingang langsam hochgedreht wird? Den Ausgang dabei nicht unbelastet lassen, sondern den Akku und vielleicht eine geringe Last parallel dazu. Wenn das Netzteil bei Deinen Versuchen zu kräftig war (keine vernünftige, zu langsame Strombegrenzung), kann es den Regler zerstört haben. In Reihe zum Netzteil muss eine Strombegrenzung vorhanden sein. Das kann eine Glühlampe sein. --31.212.212.83 23:44, 11. Jan. 2023 (CET)
Deine Überlegungen und Vorschläge sind vernünftig, aber trotzdem nur begrenzt hilfreich. Die Lichtmaschinenleistung ist beim Motorrad, im Gegensatz zum Auto, ziemlich knapp ausgelegt: es gibt eine "Grundlast" aus Zündung, Motorsteuergerät, Benzinpumpe(?) und Beleuchtung von vielleicht 10 A, und dann bleiben lediglich vielleicht 1 A Ladestrom übrig, mit dem der Akku auf Ladung gehalten werden kann. (Der wird dann gelegentlich durch das Bremslicht, das Blinklicht oder die Hupe etwas stärker belastet, und außerdem muß er natürlich noch den Anlasser und alles andere versorgen, während der Motor noch nicht läuft und die LiMa deswegen auch noch keinen Strom liefert.) Man kann also davon ausgehen, daß die LiMa im Normalbetrieb wie ein Fahrraddynamo praktisch im Kurzschluß und damit als Konstantstromquelle läuft. (Dieses Betriebsverhalten ergibt sich dadurch, daß sowohl die Leerlaufspannung als auch die synchrone Reaktanz frequenz- und damit drehzahlproportional sind.) Kann sein, daß der Regler unter diesen Bedingungen sinnvoll als Shuntregler arbeiten kann, weil es dann nämlich die Maschinenreaktanz als nicht-dissipativen "Vorwiderstand" benutzen kann und dadurch nicht so viel elektrische Leistung in Wärme umsetzen muß. Das Labornetzteil gibt eher zu wenig als zuviel Strom ab, denn es kann nur 2-3 A, ist also quasi "von Natur aus" strombegrenzt und bringt weniger als die LiMa. (Ich weiß auch nicht, ob der Längs- oder Querregler nun ein "analoger" oder ein Schaltregler mit PWM ist.) Wie auch immer: ich verstehe nicht, was da denn nun faul sein kann. Im Prinzip läuft es darauf hinaus, ob der Anker oder der/die Regler defekt sind und eines davon oder beide ausgetauscht werden müssen. Da der Regler schon ausgetauscht wurde und das keine Veränderung bewirkt hat, spricht einiges dafür, daß es der Anker ist (wobei es möglich ist, daß der aus seltsamen Gründen Überspannung abgibt und dadurch beide Regler irgendwie geschossen hat, aber danach sieht es nun auch wieder nicht aus, wenn die sich mit dem Labornetzteil erwartungsgemäß verhalten). Natürlich habe ich noch diverse wilde Theorien über Windungsschlüsse und Oberwellen im Hinterkopf, die aber leider alle den Nachteil haben, daß sie reichlich spekulativ und unplausibel sind. Bin eher ein bißchen ratlos... (Und der Halter ist kein E-Techniker, der versteht meine Überlegungen streckenweise einfach nicht und ist mit den leistungselektronischen Details ziemlich überfordert. Daß dazu im Werkstatthandbuch auch keine Einzelheiten stehen, ist auch kein Wunder: das richtet sich niveaumäßig natürlich an "Schrauber" mit Kfz-Gesellenbrief und nicht an Ingenieure.) --77.10.59.113 01:21, 12. Jan. 2023 (CET)
"... Die Lichtmaschinenleistung ist beim Motorrad, im Gegensatz zum Auto, ziemlich knapp ausgelegt ...". Das ist mir bekannt und spricht nicht gegen einen Shuntregler. Im Gegenteil. Ein Labonetzteil mit 2-3A ist, je nach Topologie, durchaus in der Lage deutlich belastbarere Halbleiter zu zerstören. Dazu muss es am Ausgang nur eine entsprechend große Kapazität haben.--31.212.212.83 01:51, 12. Jan. 2023 (CET)
Hm. Interessanter Aspekt. Tatsächlich hatte ich angenommen, daß ein LNT "von Hause aus" eine Strombegrenzung hat - was aber nicht zutrifft, wenn es ausgangsseitig einen Entladestromstoß produzieren kann. Kann es? K. A. (Das Problem ist nicht/kann nicht auftreten, wenn man die Spannung erst dann hochdreht, wenn der LNT-Ausgang an einen Reglereingang angeschlossen wurde. Wurde sie? Weiß nicht, war nicht dabei... Werde fragen.) Trotzdem: was wäre das Szenario? Alter Regler war "zufällig" defekt - solange der funktionierte, hatte ihn niemand mit einem LNT o. ä. "gequält" - und der "neue" "Schrott ab Werk"? (Der wurde unkaputtgetestet eingebaut.) Denn wenn der alte wirklich futsch ist und der neue zunächst funktioniert hätte, dann müßte ihn was nach dem Einbau umgebracht haben? Überspannungsstöße aus der in welcher Weise defekten Ankerwicklung? Ziemlich schwer vorstellbar... (Wie man es dreht und wendet, man brauchte adäquate Meßmittel: einerseits die Möglichkeit, den LiMa-Strom zu oszillographieren - idealerweise im Vergleich mit einer "gesunden" Maschine - andererseits eine regelbare Drehstromquelle, um zu schauen, ob es die Elektrik vom Regler bis zur Batterie "tut", wenn sie daraus beaufschlagt wird. - Hatte noch vorgeschlagen, mittels Brückengleichrichter und Lade-C die Spitzenwerte der einzelnen Stränge der LiMa zu messen - diese DVM-Schätzeisen für den Effektivwert sind bei variablen Frequenzen und möglichen Oberwellen in einer potentiell EMV-verseuchten Umgebung einfach zu unzuverlässig - aber Gleichrichter pipapo ließ sich angeblich nicht auftreiben...) --77.3.83.252 10:52, 12. Jan. 2023 (CET)
Gerade moderne Labor(schalt)netzteile haben Ausgangskapazitäten, was der Stabilität geschuldet ist. Selbst bei sehr kleinen Stromvorgaben kann es bei einem Kurzschluss ordentlich knallen. Die ganzen Überlegungen sind etwas wenig systematisch. Vielleicht ist noch gar nichts defekt, sondern nur Mist gemessen worden. Man braucht keine besonderen Messmittel. Wenn es ein Shuntregler ist, wie ich nach wie vor denke, reicht ein Labornetzteil und eine Glühlampe, um die grundlegende Funktion des Reglers zu prüfen. Langsam die Spannung am Eingang hochdrehen und wenn die Lampe angeht, hat der Regler seinen Eingang kurzgeschlossen. Die Anfangs geschilderten Kurzschlüsse sind ggf. die korrekte Funktion, vor allem, wenn mit 15 bis 20V getestet wurde. Es kann sein, dass nicht an allen Strängen geregelt wird. Dazu müsste man den genauen Schaltplan kennen. --31.212.244.31 16:02, 13. Jan. 2023 (CET)
Der gemessene Eingangskurzschluß tritt nur an einem Regler und nur zu einem der drei Anschlüsse von jeweils den beiden anderen her und nur in einer der beiden möglichen Polaritäten auf. Die plausibelste Erklärung dafür ist für mich, daß eine der sechs Dioden des Brückengleichrichters durchlegiert ist und nicht mehr sperrt, und das ist nicht so tragisch, weil dann eben während eines Sechstels der Periode ein Strang der LiMa kurzgeschlossen ist. (Na gut, der Regler ist damit natürlich trotzdem Schrott, weil man ihn nicht "aufbohren" - vergossen - und die defekte Diode wechseln kann). Das erklärt aber nicht, wieso die Batterie nicht geladen wird. Dafür habe ich aber inzwischen eine ganz andere Hypothese: LiMa und Regler sind vollkommen i. O., aber Benzinpumpe oder Zündspule schlucken zuviel Strom, so daß schlicht und einfach für die Batterie kein Ladestrom mehr übrig ist, da die LiMa halt recht knapp dimensioniert ist. Und das wird in zwei Wochen ziemlich simpel dadurch getestet, daß die Sicherungen von Pumpe und Steuergerät - das auch die Zündung versorgt - einzeln gezogen und dort dann der aufgenommene Strom gemessen wird, dann sieht man, ob der Strom in der Summe zu hoch ist - das würde auch das Warmwerden der Drehstromleitung zum Reglermodul erklären. (Und wenn dort tatsächlich ein Shunt-Regler werkelt, dann hat der sich schlafen gelegt, denn bei Unterspannung im Bordnetz hat der schließlich gar keinen Grund zur Veranlassung.) Möglicherweise wird die Batterie auch schon bereits wieder geladen, steigt die Bordnetzspannung also ausreichend an, wenn einfach mal die Scheinwerfersicherung gezogen und damit 70 W Last abgeworfen werden. Ich habe einige sinnvolle Messungen angeregt, die zur Klärung beitragen werden, aber ich denke, daß wir das hier beenden können - vielen Dank für die nützlichen Anregungen! --77.8.98.92 02:58, 14. Jan. 2023 (CET)

Lage der KGB-Repräsentanzen in Bonn und Köln

Wo waren die Резидентура КГБ в Кельне (ФРГ) bzw. Резидентура КГБ в Бонне (ФРГ)? (Straße, Hausnummer) --Gelöbnix (Diskussion) 19:58, 13. Jan. 2023 (CET) PS: Danke im voraus.

2003:F7:DF0D:B800:9DBA:2C45:102B:C2B5 00:23, 14. Jan. 2023 (CET)

Unabhängigkeit der Justiz in anderen Ländern

Liebe Auskunft, trotz Gewaltenteilung wird in Deutschland die Rechtsprechung durch die ausführende Gewalt in Gestalt der Justizministerien organisiert. Gibt es große, westlich orientierte und/oder wirtschaftlich hochentwickelte Länder, in denen die Justiz selbstorganisiert ist, zum Beispiel, aber nicht nur wie es in Deutschland der öffentlich-rechtliche Rundfunk ist? Vielen Dank für Antworten! --BlackEyedLion (Diskussion) 22:39, 11. Jan. 2023 (CET)

Richter Udo Hochschild benennt Italien in [7] als Beispiel. --L47 (Diskussion) 07:44, 12. Jan. 2023 (CET)
Mal langsam. wird in Deutschland die Rechtsprechung durch die ausführende Gewalt in Gestalt der Justizministerien organisiert Auch ein Justizminister kann nicht in die richterliche Unabhängigkeit eingreifen. Man möge den Gegenbeweis antreten. Was anderes sind die Staatsanwaltschaften, die weisungsgebunden sind. Dies wird von der EU auch kritisiert. Die StA spricht aber kein Recht, hier sollte man die Worte schon genau nehmen.--scif (Diskussion) 08:04, 12. Jan. 2023 (CET)
Kommt auf "eingreifen" an. Personal-/Stellenbesetzungsangelegenheiten sind bereits der Versuch der Einflussnahme.--Wikiseidank (Diskussion) 08:18, 12. Jan. 2023 (CET)
@Scialfa: Beweis ist es keiner, aber Indiz: Leitung des Oberlandesgerichts Stuttgart – Streit um OLG-Chefsessel: Ministerium klagt gegen Richter und Streit um OLG-Chefsessel – Neue Richtervereinigung kritisiert BW-Justizministerin scharf. --Rôtkæppchen₆₈ 08:21, 12. Jan. 2023 (CET)
Leute, bitte zwischen Stellenbesetzungen und der ureigenen Rechtsprechung im Prozeß unterscheiden. Das die Politik bei Stellenbesetzungen eingreift ist ein alter Hut, nicht selten erleben wir das bei der Wahl von neuen BGH-Richtern. Man möge mir aber mal erklären, was ein neuer OLG-Chef mit der Urteilsfindung eines Richters am AG oder eines OLG-Senats zu tun hat. Die Empörung bezieht sich darauf, das die justizinternen Personalvorschläge ignoriert werden. Das hat aber nichts mit der Unabhängigkeit der richterlichen Urteilsfindung zu tun. Ich gehe dann mit, wenn ein Justizministerium klar verfassungsfeindliche Richter durchdrücken würde.--scif (Diskussion) 08:47, 12. Jan. 2023 (CET)
warum hat es 70 Jahre gedauert bis man merkt, dass nicht alles so toll ist wie man es sich eingeredet hat? --2001:9E8:A504:1900:2B3D:E4A8:7A89:5E44 08:49, 12. Jan. 2023 (CET)
Nach BK: Du möchtest uns konkret was mit deinem sinnbefreiten Beitrag sagen? Besser als zwischen 33 und 45 allemal.--scif (Diskussion) 09:05, 12. Jan. 2023 (CET)
Um die Ausübung der Rechtsprechung ging es mir in der Frage nicht, sondern um deren Organisation. Anlass meiner Frage war ein Gespräch zwischen Franziska Giffey und Sandra Maischberger, dass in einem anderen Bundesland bereits Urteile gegen Silvester-Straftäter ergangen sind (ob das stimmt, muss hier nicht diskutiert werden), weil die dortige Justiz personell besser ausgestattet ist.
Das gut belegte Beispiel Italien reicht mir erst mal. In der Quelle wird leider nicht klar, wie sich die Rechtsprechung dort finanziert. Interessant wäre noch, ob sie (wie in Deutschland der öffentlich-rechtliche Rundfunk) ihre Finanzierung selbst festlegt. --BlackEyedLion (Diskussion) 09:01, 12. Jan. 2023 (CET)
Das solltest du dann vorher aber als Ausgang deiner Frage vielleicht auch mal so formulieren. Dadurch entsteht doch eine völlig andere Diskussion. Geht damit los, ob die Justiz in ITA auch so föderal organisiert ist.--scif (Diskussion) 09:05, 12. Jan. 2023 (CET)
In meinem Fragebeitrag kommt in beiden maßgeblichen Sätzen das Wort organisiert vor. Im Übrigen betrifft meine Frage ja gerade nicht die Lage in Deutschland, sondern in anderen Ländern. --BlackEyedLion (Diskussion) 09:15, 12. Jan. 2023 (CET)
Ist die italienische Justiz denn wirklich selbstorganisiert? Auch in Italien soll es ein Justizministerium geben. --2A0A:A541:FDDC:0:28D0:E784:ADE4:D0E0 19:05, 14. Jan. 2023 (CET)
USA Unterhalb der Bundesebene.
Problematischer ist Italien, wo die Juristen sich gegenseitig kontrollieren sollen. Na, ob das eine gute Idee ist? --2A02:908:424:9D60:F0D7:4631:DBFF:AE50 09:53, 13. Jan. 2023 (CET)
Richter in den USA werden von der jeweils zuständigen Regierung ernannt oder vom Volke gewählt. Selbstorganisation sieht meiner Meinung nach anders aus. --2A0A:A541:FDDC:0:28D0:E784:ADE4:D0E0 19:07, 14. Jan. 2023 (CET)
Die Schweiz hat eine selbstorganisierte Justiz. Die Unabhängigkeit der Justiz wird durch die Verfassung und Gesetze gewährleistet. Richter und Staatsanwälte werden unabhängig von politischen Einflüssen ernannt und können nur aus wichtigen Gründen entlassen werden. Es gibt auch eine Justizaufsicht, die grundsätzlich unabhängig, sowohl von der Exekutive als auch von der Legislative. Kritisiert wird teils allerdings das Fehlen von unabhängigen Instanzen, die Beschwerden gegen Richter und Staatsanwälte prüfen, sowie die Möglichkeit, dass die Aufsicht durch die Exekutive beeinflusst werden kann. --2A02:3030:80D:1798:1F4F:3F8F:CD3:F843 18:59, 14. Jan. 2023 (CET)

Suche einen Kinder- oder Jugendfilm

Ich schaute in den 80ern oder 90ern eine Serie oder einen Kinder- oder Jugendfilm, bei dem ich mich nur noch an wenige Details erinnere: Es lief wahrscheinlich auf ARD oder ZDF. Hauptfigur ist ein Junge, der irgendwann vor 1920 (vielleicht zur Zeit der Industrialisierung oder noch weit davor?) von einem dunkel gekleideten Typ (evtl. mit einem Zylinder und einem Gehstock und evtl. mit entstelltem Gesicht?) "besessen" wird. Der Junge muss arbeiten und wird schlecht behandelt, der dunkle Typ bekommt das Geld. Eine Szene, an die ich mich noch erinnere ist, dass der Junge in einem Bett aufwacht und dort mit Ketten gefesselt wurde (evtl. von seinen Eltern oder Ersatzeltern).

Kann damit jemand etwas anfangen und kennt sogar den Titel? -- 146.52.19.124 01:15, 12. Jan. 2023 (CET)

Möglicherweise suchst Du Der Prinz und Bettelknabe von Mark Twain?
Außerdem käme noch David Copperfield von Charles Dickens in Frage.--92.73.145.117 03:26, 12. Jan. 2023 (CET)
Ich habe jetzt in einige der Verfilmungen reingesehen, aber das ist es wohl nicht. Ich habe es düsterer Erinnerung. Produziert wurde das, was ich suche, vermutlich in den 80ern oder vielleicht in den 70ern. Soweit ich mich erinnere, wurde der Junge an diesen dunkeln, böse erscheinenden Typen verkauft. --146.52.19.124 12:13, 12. Jan. 2023 (CET)
Klingt für mich eher nach Oliver Twist (ebenfalls von Dickens) und dem Bandenchef Fagin, der Oliver gefangen hält und zu einem Dieb abrichten will. --Geoz (Diskussion) 13:48, 12. Jan. 2023 (CET)
Darauf bin ich bei meiner Recherche auch schon gestoßen. Die zeitliche Einordnung (Frühindustrialisierung) könnte passen, aber ich hab die düstere Gestalt jünger und hässlicher als diesen Fagin in Erinnerung. --146.52.19.124 15:13, 12. Jan. 2023 (CET)
Eventuell Silas (Fernsehserie) - da wurde der Protagonist wiederholt verkauft, eingesperrt, entführt usw. --Chuonradus (Diskussion) 14:28, 12. Jan. 2023 (CET)
@TO, könnte das Timm Thaler oder eine der Verfilmungen von Die rote Zora und ihre Bande gewesen sein? --Rôtkæppchen₆₈ 15:13, 12. Jan. 2023 (CET)
Über die aktuelle ZDF-Mediathek, wo gerade Silas und Timm Thaler verfügbar sind, bin ich überhaupt wieder auf das Thema gestoßen. Silas ist es jedoch nicht (was ich auch zuerst dachte). Timm Thaler spielt zu sehr in der Moderne. Ich glaube, dass es keine der Weihnachtsserien ist. In die rote Zora habe ich jetzt auf Youtube reingeschaut, aber nichts wiedererkannt. Das war es wohl auch nicht.
Zu der Szene mit dem im Bett angeketteten Kind ist mir noch etwas eingefallen: Ich glaube, dem Kind wurde eingeredet, es könne nicht laufen. Irgendwann hat es herausbekommen, dass es angekettet ist und nur deswegen seine Beine nicht bewegen konnte. Und es kann auch sein, dass es das nur geträumt hat. --146.52.19.124 15:32, 12. Jan. 2023 (CET)
Es gibt über 20 Verfilmungen von Oliver Twist und auch über 20 verschiedene Darsteller (und sicher auch (leicht?) unterschiedliche Interpretationen) des Fagin Hast du alle durchgeschaut? --77.3.79.71 22:54, 13. Jan. 2023 (CET)
Exakt das habe ich mich auch gefragt, und wollte schon gezielt die Verfilmung aus Großbritannien aus dem Jahr 1985 ansprechen, mit Eric Porter als Fagin. Bleibt aber die Frage: Wie kann man als Normalsterblicher ohne Zugriff auf der Öffentlichkeit nicht zugängliche Medien- und Filmarchive diese Durchsicht aller dieser Verfilmungen realisieren, um die richtige zu finden? --92.73.145.117 23:08, 13. Jan. 2023 (CET)
Ich glaube, wie gesagt, dass die Produktionszeit in den 70ern oder 80ern war. Und es war kein Zeichentrickfilm. Das reduziert die Anzahl der möglichen Oliver Twist-Verfilmungen auf 2. Wenn man das Musical von 1968 dazunimmt, dann auf 3.
Ich habe auf Youtube in einige Verfilmungen aus der Zeit reingeschaut und fand, dass der Fagin dort immer ähnlich aussah. Der Mann, der den Jungen so schlecht behandelt in dem Film, den ich suche, ist meiner Erinnerung nach auch dunkler (schwarz?) gekleidet, jünger (vielleicht in seinen Dreißigern) und hässlicher.
Zu der Serie von 1985 fand ich mehrere Folgen: [8][9][10][11][12]. Davon deckt sich nichts mit meiner Erinnerung.
Den Film von 1982 gibt es auf Youtube: [13]. Auch das ist er nicht. Und auch das Musical ist es nicht. Wahrscheinlich ist es einfach nicht Oliver Twist. --146.52.19.124 01:20, 14. Jan. 2023 (CET)
Ein Bekannter von mir brachte mich auf Die schwarzen Brüder ([14]). Ich bin mir ziemlich sicher, dass ich den dortigen Schornsteinfeger meinte, der Kinder von armen Familien kauft und dann für sich arbeiten lässt.
Ich habe die Serie jetzt im Schnelldurchlauf durchgeschaut, konnte aber die Szene mit dem Kind, das ans Bett gekettet war, nicht finden. Anscheinend habe ich zwei Serien/Filme durcheinandergebracht. Da ich aber außer dieser Szene dann nichts weiter dazu weiß, kann das alles sein und ich muss wieder bei Null anfangen.
Immerhin konnte die Hälfte meiner Erinnerung aufgeklärt werden. Danke für eure Mithilfe! --146.52.19.124 23:50, 14. Jan. 2023 (CET)

Infektion privater Endgeräte durch „infiziertes WLAN“

Ich hoffe, dass ich es schaffe, meine Frage so zu stellen, dass darauf eine sinnvolle Antwort kommen kann.

Wenn man sich mit einem privaten Endgerät – Laptop, Smartphone, Tablet – in das WLAN einer Institution (Schule, Uni, Firma, Flughafen …) einloggt, das Rechnernetz dieser Institution aber dummerweise gerade großflächig mit Computerviren (Trojaner, Ransomware o.dgl.) infiziert ist und/oder auch menschliche Hacker im Netzwerk ihr Unwesen treiben, ist dann davon auszugehen, dass auch das private Endgerät infiziert wird?

Mir ist bewusst, dass eine beliebte Standardantwort hier „ja“ lautet (Google ist daher auch nicht so aufschlussreich). Mir geht es vielleicht eher darum, welche Möglichkeiten die Viren/Hacker überhaupt haben, Geräte zu infizieren, die lediglich beim WLAN eingeloggt sind. Es geht um private Geräte, die nicht von der fraglichen Institution administriert werden (also auch keinen normalen Zugriff für deren Systemadmins bereitstellen oder dergleichen). So, wie ich das verstehe, ist WLAN „passiv“, reagiert also nur, wenn ein Gerät eine Anfrage stellt. Demnach könnte es einem mit ihm verbundenen Gerät auch nur Malware unterjubeln, indem es deren Daten mit der Antwort auf eine Netzwerk-Anfrage (HTTP oder so was) mitschickt. Aber so was kann ja auch bei einem nicht-infizierten WLAN passieren, wenn die Malware genuin aus dem Internet kommt … Darauf sollten Virenschutzprogramme auf dem Endgerät also ohnehin vorbereitet sein.

Eine etwas sinnvollere Formulierung meiner Frage ist also vielleicht: Wodurch kann im eingangs geschilderten Szenario ein höheres Risiko entstehen, dass man sich auf dem privaten Endgerät Malware einfängt, als durch das Surfen im Internet ohnehin schon besteht? --2A02:8108:50BF:C694:BC5A:B3DD:BC5D:4255 14:45, 12. Jan. 2023 (CET)

Das gliedert sich in mehrere Teilfragen auf. Das WLAN an sich ist ja erstmal nur eine Verbindung zwischen mehreren Rechnern und als solche selten „selbst“ infiziert bzw. infektiös. Höchstens beim Verbindungsaufbau könnte ein kompromittierter WLAN-Router ungute Datenpakete auf deinem Gerät abspeichern, aber ich wüsste nicht, wieso der da überhaupt was abspeichern (können) sollte. Dann beschränkt sich die Frage auf: 1. Kann über das WLAN ein anderes daran hängendes Endgerät auf dein Gerät Malware übertragen? 2. Kann diese Malware auf deinem Endgerät Schaden anrichten? Beides ist durch gewollte Dateiübertragung sicher nur schwer möglich, und durch Softwarefehler in Programmbibliotheken, die an der Netzwerkkommunikation beteiligt sind, vermutlich deutlich einfacher. Dem begegnest du am besten durch häufige Updates (nicht nur Apps, sondern auch System). Die „Passivität“ eines WLAN würde ich nicht unterschätzen, da findet ständig Verkehr in beiden Richtungen statt. Auf einen Ping von einem anderen Rechner aus wird dein Handy vermutlich antworten, und hat ein Angreifer erstmal die Netzwerkadresse (IP), kann er fröhlich abklopfen, was für Dienste auf deinem Gerät denn gerade offen haben und ob man denen was unterjubeln kann. In 99.999 von 100.000 Fällen lohnt sich aber die Mühe nicht, der Nutzen ist zu klein, wenn du nicht gerade ein ganz großer Fisch bist. --Kreuzschnabel 16:01, 12. Jan. 2023 (CET)
Es kommt hier auch extremst auf das verwendete Betriebssystem an. Windows ist da wesentlich anfälliger als ungerootetes Android. Windows hat zahlreiche Ports offen, die mitunter Hintertüren für böswillige Angreifer bieten. Aus dem Internet sind diese schlecht bis gar nicht erreichbar, weil ein guter Router die meisten Angriffe abblocken sollte. Sitzt der Angreifer hingegen im LAN zwischen Router und AP, so sind die offenen Ports aller ins (W)LAN eingebuchten Geräte frei zugänglich. Der WLAN-Admin kann seine APs so einstellen, dass die einzelnen über denselben AP eingebuchten Endgeräte nicht über WLAN miteinander kommunizieren können, sondern nur mit dem Internet. Sitzt der Angreifer im verkabelten Segment, so ist diese Einstellung wirkungslos. --Rôtkæppchen₆₈ 16:25, 12. Jan. 2023 (CET)
Dein zweiter Satz lässt durchscheinen, dass ein gerootetes Android im Sinne des hier verhandelten Szenarios unsicherer ist als ein ungerootetes. Worin besteht dieser Unterschied? Zunächst mal heißt „gerootet“ doch nur, dass jeder Userprozess auf Anfrage Root-Rechte bekommen kann. Ich als User muss aber immer erstmal Ja sagen :) außer es gibt da einen pösen Prozess, der von mir arglos Root-Rechte bekommen hat und diese ohne Nachfrage nun auch anderen erteilt. Oder ist auf einem gerooteten Androiden auch generell mehr zum Netz hin offen? Ich gestehe, da hab ich mich noch nicht schlau gemacht. --Kreuzschnabel 17:23, 12. Jan. 2023 (CET)
Wenn du nicht grad wirklich arg vom Fach bist (fast keiner) ist alles, wo rumgefrickelt wurde, schon per se bedenklich(er) einzuschätzen, dazu müssen wir noch lange nicht beim Rumspielen mit Rootrechen sein, dann ist es eh'n no-brainer und da ist dem Käppchen ganz schwer zu widersprechen. Das mit Windows find ich dagegen zu pauschal, mal davon abgesehen, dass mobile Systeme heute generell denk ich sicherer einzuschätzen sind als (realistische) Desktop-Workstationen (weniger Angriffsfläche, weniger Legacy, *viel* mehr Augen, Mindshare, etc.). Also bei meinen Windows-Boxen ist gar nix offen, das hängt selbstverständlich ganz davon ab, was man dort am Laufen hat, oder laufen lässt. Aber das gilt nun für jede Plattform. So pauschal ist das jedenfalls keine Ansage. -82.83.169.131 19:11, 12. Jan. 2023 (CET)
IMHO ist das so pauschal eine Aussage, da Android offene Ports bis 1024 ohne Root gar nicht zulässt, während Windows standardmäßig einige Ports öffnet, über die es dann angreifbar ist, beispielsweise Port 135 (Remote Procedure Call), 445 (Server Message Block), 808 (Intel Graphics Command Center) etc pp. --Rôtkæppchen₆₈ 20:44, 12. Jan. 2023 (CET)
Die meisten mobile Anbieter betreiben ein NAT und zudem Port hiding. Die Adresse des Endgerätes ist "privat" im Netz des Providers und nicht die öffentliche Adresse die nach außen sichtbar ist. Du kannst das einfach sehen, wenn du die per DHCP zugewiesene Adresse mit z.B. whatismyipdotcom vergleichst. Auch ein Ping oder Portscan mit z.B nmap (auch nicht erzwungen) ist von außen nicht möglich. --79.222.86.181 12:04, 13. Jan. 2023 (CET)
In diesem Abschnitt geht es nicht um mobiles Internet, sondern um WLAN. Wenn der Angreifer im (W)LAN sitzt, ist es vollkommen gleichgültig, ob ob und wieviele NAT-Firewalls zwischengeschaltet sind und ob die öffentliche Adresse und offene Ports des WLAN-Gateways sichtbar sind oder nicht. --Rôtkæppchen₆₈ 15:47, 13. Jan. 2023 (CET)
in der Frage schwingt unterschwellig die Ansicht mit, dass Computerviren in Computernetzen vagabundieren und vorbeikommende Rechner befallen. Computerviren sind immer Programme, die entweder auf einem bestimmten Rechner laufen, oder die in einer Datei eingepackt sind, die man sich selbst auf den Rechner/auf das Handy runterlädt und die dann absichtlich oder automatisch auf dem eigenen Gerät gestartet werden. Zur Frage ob es einen Unterschied gibt zwischen "Surfen im Internet" und "besuchen eines Wlans": ja, das ist ein Unterschied. In einem Wlan lassen sich neu beitretende Geräte sehr leicht erkennen. Das liegt zum einen daran, dass ein Wlan ziemlich klein ist und der IP-Adressbereich begrenzt ist. Zum anderen teilen Geräte im Wlan ihre MAC-Adresse mit. Man kann also neue Teilnehmer im Wlan schnell erkennen und man kann relativ leicht erkennen, was es für ein Gerät ist. Zum anderen besteht oft die Wahrscheinlichkeit, dass man sogar physisch beobachtet wird. Das alles macht es für einen Angreifer leicht, spezielle Angriffe zu starten. In einem Wlan ist es auch ziemlich einfach, die Nameserveranfragen zu manipulieren, so dass Seitenabrufe umgelenkt werden oder man-in-the-middle-Angriffe eingeleitet werden. --2001:9E8:A51F:3600:2B06:847F:E599:9F7B 16:27, 13. Jan. 2023 (CET)
Man-in-the-middle-Angriffe sollten (zumindest bei HTTPS-Verbindungen) aber durch die Zertifikate unterbunden werden, oder? (Soweit ich weiß, sind die genau dafür da.) --2A02:8108:50BF:C694:89D6:F6A8:A5F5:713 19:37, 14. Jan. 2023 (CET)

Spezielle Funktionswerte finden

Mathematische Funktionen liefern unendlich viele Zahlenwerte. Aber nur wenige davon sind nicht irrational oder als Ausdruck darstellbar, z.B. sin(135°) =1/√2. Wie findet man die Argumente die diese speziellen Werte liefern, vor allem wenn das alles andere als naheliegend ist, z.B. K(√(17 - 12·√2)) = 2(2+√2) √π³ / Γ²(-1/4) ? --46.114.3.251 01:51, 14. Jan. 2023 (CET)

Deine Aufgabenstellung ist in allgemeiner Form zu schwierig, dazu gibt es zu viele Funktionen, auch solche mit nur irrationalen Werten. Beim Sinus findet man unendlich viele rationale Werte durch Pythagoreische Tripel über die rationale Parametrisierung des Einheitskreises, siehe dazu etwa [15]. Der Schlüssel dazu ist die Formel  . --95.222.50.72 10:07, 14. Jan. 2023 (CET)
Das ist nicht nur schwierig, sondern kann in der Allgemeinheit wohl gar nicht beantwortet werden. Denn man müßte jede Art von Funktionen besonders betrachten. Polynome mit rationalen Koeffizienten liefern für rationale Argumente einen rationalen Funktionswert. Für bestimmte irrationale oder komplexe Argumente (algebraische Zahlen) ist der Funktionswert ebenfalls rational. Für die Sinus- und Kosinusfunktion wurden schon rationale Funktionswerte genannt (s.a. Parameterdarstellung), allerdings ist das Argument des Sinus oder Kosinus, das sich aus den Umkehrfunktionen ergibt, in den wenigsten Fällen kein rundes Vielfaches von  . Beim Sinus und Cosinus hängt es vermutlich von der Konstruierbarkeit ab, ob das Ergebnis durch rationale Zahlen und Wurzeln ausgedrückt werden kann, jedoch ist z.B.   auch nicht mehr einfach (s. Siebzehneck). Beim natürlichen Logarithmus würde es mich wundern, wenn es außer   andere rationale Paare von Argument und Funktionswert gibt. Mit Umkehrfunktionen kann man bei vorgegebenen (z.B. ganzzahligen) Funktionswerten die Argumente berechnen, die gewünschte Funktionswerte liefern. Bei der Wurzelfunktion sind es die Quadratzahlen, deren Wurzel ganzzahlig ist, beim natürlichen Logarithmus ganzzahlige Potenzen von  . So könnte man nach und nach immer weitere Arten von Funktionen betrachten... 91.54.32.105 14:07, 14. Jan. 2023 (CET)
Zumindest stetige Funktionen können nicht nur irrationale Werte liefern. Das folgt aus dem Zwischenwertsatz. --Digamma (Diskussion) 14:12, 14. Jan. 2023 (CET)
Und wenn die Funktion zwischen a und b stetig ist, gibt es umgekehrt zu allen rationalen oder ganzzahligen Funktionswerten zwischen dem minimalen und dem maximalen Funktionswert im Intervall von a bis b auch mindestens ein Argument der Funktion, dessen Funktionswert eben diese rationale oder ganzzahlige Zahl ist. 91.54.32.105 15:06, 14. Jan. 2023 (CET)
  scheint mir auf   sogar äußerst stetig zu sein.;) --95.222.50.72 16:43, 14. Jan. 2023 (CET)
Ja, und zu jedem rationalen Funktionswert, der zwischen dem Minimum und dem Maximum dieser Funktion liegt, gibt es ein  , so daß   rational ist. 91.54.32.105 17:15, 14. Jan. 2023 (CET)
Ja, aber Digammas Aussage ist trotzdem falsch. --95.222.50.72 18:15, 14. Jan. 2023 (CET)
Ja, damit hast Du recht. Mir ist es nicht aufgefallen und ich habe Deinen Kommentar auf meine Aussage bezogen. 91.54.32.105 18:40, 14. Jan. 2023 (CET)
Digammas Aussage sollte stimmen, sobald man konstante Funktionen ausschließt, da die rationalen Zahlen dicht in den reellen Zahlen liegen. --Sabrieleauftistik (Diskussion) 19:55, 14. Jan. 2023 (CET)
Aber auch nur auf Intervallen;). Z.B. ist   stetig und hat nur irrationale Werte. Aber ich glaube, diese akademische Diskussion ist für den OP eher uninteressant.  -Funktionen sind für ihn wahrscheinlich ausreichend allgemein. --95.222.50.72 20:18, 14. Jan. 2023 (CET)
…wobei der Zwischenwertsatz sich nur auf solche Funktionen bezieht, also hat Digamma das vermutlich implizit vorausgesetzt. --Sabrieleauftistik (Diskussion) 20:32, 14. Jan. 2023 (CET)

Mauer, Zaun oder tote Hecke

Letztens gesehen und ich kenne keinen Namen: im Abstand von ca 80 cm werden zwei einfache Zäune parallel gezogen (eigentlich reichen die Zaunpfosten, wenn sie dicht genug gesetzt werden). Der Zwischenraum wird gefüllt mit Schnittgut von trockenen Ästen, Laub, Grassoden und anderem trockenen Gartenabfällen, die nicht sofort kompostieren. Das ganze ist ca 150 cm hoch, wie eine sehr dichte Trockenhecke im Winter. Tiere und Insekten leben in dieser natürlichen "Mauer". Mitunter wachsen auch weiter Pflanzen, was aber nicht geplant ist. Wie heißt sowas?

--2003:C9:8735:BE00:C4C8:1C58:D5A2:56A0 21:35, 14. Jan. 2023 (CET)

Variante der Benjeshecke? --Stemmerter (Diskussion) 22:15, 14. Jan. 2023 (CET)
Mal nach Reisighecke oder Totholzhecke gegoogelt? Sind aber offenbar auch nur Synonyme von Benjeshecke. Übrigens sind Insekten auch Tiere ;-). --Blutgretchen (Diskussion) 23:00, 14. Jan. 2023 (CET)

Bürostuhl vs. Gamerstuhl

Mein Home Office verlangt nach einer neuen Dauersitzmöglichkeit. Habe über Bürostühle allerlei gelesen. Aber eigentlich gefallen mir Gamer-Stühle von der Anmutung her besser. Nun heißt es überall: Bürostühle setzen auf Ergonomie, Gamer-Stühle "nur" auf Komfort. Meine Frage an User von Gaming-Stühlen, also aus eigener Erfahrung, auch aus innerfamiliärer Beobachtung vielleicht: Was findet ihr an Gamer-Stühlen vorteilhaft? Entweder im Vergleich mit Bürostühlen oder an sich? Die oft genannten Nachteile interessieren mich nicht so, sondern Vorteile, die vielleicht nicht so bekannt sind, die ihr aber selbst bemerkt habt. --Aalfons (Diskussion) 20:06, 14. Jan. 2023 (CET)

Ich habe einen alten, aber bequemen und ergonomisch sicherlich brauchbaren Bürostuhl. Auf Dauer macht einen das rückenfaul. Ein Gamerstuhl, denk ich, noch mehr. Das wollte ich nicht und habe einen kurzen Ausflug zu einem Sitzball gemacht. Das war nix, u.A. wegen des großen Volumens. Das Ding war immer im Weg. Nun sitze ich meistens auf einem höhenverstellbaren Bürohocker oder stehe am Schreibtisch. Dem Rücken tut das gut. --31.212.244.31 20:38, 14. Jan. 2023 (CET)
Gamerstuhl ist Werbesprech, keine (Ergonomie-) Vorteile. Homeoffice/-Schule war bei uns schon, ca. 10 Stühle ausprobiert. Ergebnis: ist individuell. --Wikiseidank (Diskussion) 22:02, 14. Jan. 2023 (CET)
Okay, es weiß also niemand einen verborgenen Charme des Gamerstuhls zu benennen. (Warum das Werbesprech ist, erschließt sich mir nicht, die Unterschiede sind ja deutlich; ist aber auch egal.) (nicht signierter Beitrag von Aalfons (Diskussion | Beiträge) 22:32, 15. Jan. 2023 (CET))
Wo siehst du Unterschiede? Design ist nicht automatisch Funktion. Falls jemand noch ein Foto hat, aus dem damals noch so bezeichneten "iMac-Store" aus dem "Stilwerk" in Berlin? Die hatten (in meiner Erinnerung um 2000) einen ergonomischen "PC-Work-Chair" --Wikiseidank (Diskussion) 08:26, 16. Jan. 2023 (CET)
(Erle raus) Soweit das bei den Produktvergleichen zu sehen ist, haben Gamerstühle immer integrierte Kopfstützen, neigen dazu, wie Schalensitze auszusehen (was ich ganz interessant finde), auf die Armstützen wird wenig Wert gelegt, weil Steuergeräte nicht bei aufgelegten Unterarmen bedient werden (also in dem Fall untauglich für's Home Office). Und da man Gamern nachsagt, besonders hibbelig zu sein, könnten die Federwege anders ausgelegt sein als bei Bürostühlen, oder die Wippfunktion. Vielleicht haben sie deshalb sogar ein besonders großes Drehkreuz, da habe ich die Durchschnitte nicht verglichen. Optisch haben sie durch die Farben klare Vorteile. (Es gibt wohl auch welche mit integrierten Anschlüssen, manche kommen aber in Kindergrößen, mit Liegefunktion, hochklappbaren Armlehnen, aber das alles brauche ich nicht.) So könnte jemand zum Beispiel sagen: Es gibt Gamer-Stühöe, die so cozy sind, dass man gar nicht mehr aufstehen mag. Aber da das hier niemand berichtet, ist es offenbar nicht der Fall. --Aalfons (Diskussion) 21:15, 16. Jan. 2023 (CET)
Möglicherweise ein weiterer Unterschied: Bürostühle sind in Qualitäten erhältlich, die einen Einsatz im gewerblichen Bereich erlauben, und damit zugleich eine lange Haltbarkeit bei nichtgewerblichem Einsatz versprechen. "Gamerstühle" speziell für Spieler sind eher ein typisches Endverbraucherprodukt, das nur eine bestimmte Lebensdauer haben "darf", damit der Umsatz des Herstellers langfristig garantiert ist. --92.73.133.185 21:59, 16. Jan. 2023 (CET)
Ich denke auch, dass das, auf die Produktkategorie bezogen, der wesentliche Unterschied ist. Es gibt natürlich auch billige Bürostühle, die nichts taugen, aber wenn man für ein Markenprodukt ordentlich etwas anlegt, bekommt man eine Qualität und Haltbarkeit, die es bei Gamerstühlen so sicher nicht gibt. Meinen Bürostuhl, auf dem ich täglich mindestens acht Stunden verbringe, habe ich 1992 bei der Geschäftsauflösung eines Bürobedarfshändlers zum Ausverkaufspreis von damals 1425 DM gekauft. Die Sitzfläche wurde schon dreimal neu bezogen, aber die Mechanik samt Federung ist noch immer top und wird mich überleben. Ich glaube nicht, dass das irgendein „Gamerstuhl“ schafft. Bezüglich des einzelnen Teils hilft nur Probesitzen, probesitzen, probesitzen. Also beim professionellen Bürofachhandel statt im Internet kaufen. --Jossi (Diskussion) 00:42, 17. Jan. 2023 (CET)
Mnja, daa hast du Glück gehabt! Ich hatte bisher einen Ergo, vor ca. 15 Jahren bei Warentest sehr gut bewertet, nun völlig – auch mechanisch – verschlissen. Okay, offenbar war die Hoffnung, bei den Gamer-Stühlen eine Offenbarung zu finden, deplatziert. Platziere ich mich halt wieder official(-ly). --Aalfons (Diskussion) 02:23, 17. Jan. 2023 (CET)
Archivierung dieses Abschnittes wurde gewünscht von: Spieltheorie gescheitert, danke allseiz. --Aalfons (Diskussion) 02:23, 17. Jan. 2023 (CET)

Verwandte von Verwandten, aber eigentlich nicht einem selbst

Wie bezeichnet man das? Beispielhaft sei Marc der Großneffe von Edeltrauds Vater und Lisa die Großnichte von Edeltrauds Mutter, beide seien also mit Edeltraud verwandt. In welchem Verhältnis stehen Marc und Lisa?

--2A0A:A541:FDDC:0:42B:BEF3:971A:503C 17:30, 14. Jan. 2023 (CET)

Großcousins n-ten Grades, wobei n die Anzahl der Stufen bis zum gemeinsamen Vorfahr ist. Oder hab ich dich falsch verstanden? --Kreuzschnabel 17:42, 14. Jan. 2023 (CET)
Das kenne ich als "verschwippt". Aber ich habe gerade erstaunt festgestellt, dass der Begriff nicht im Duden steht, also vielleicht nur regional? --88.64.175.189 17:47, 14. Jan. 2023 (CET)
Ich habe (Halb-)Schwestern, die nicht miteinander Verwandt sind. Keine Ahnung, wie sich das nennen soll. Darüber hab ich mir noch nie Gedanken gemacht. --31.212.244.31 17:48, 14. Jan. 2023 (CET)
Das ist aber eigentlich eine falsche Verwendung von "Halbschwester". Wenn sie nicht verwandt sind, dann sind es Stiefschwestern. --88.64.175.189 17:52, 14. Jan. 2023 (CET)
Das kann man als entfernte angeheiratete Verwandte bezeichnen, s.a. Schwägerschaft. Und das hat nichts mit Edeltraud zu tun, sondern damit, daß ihre Eltern geheiratet haben. Lisa ist die Großnichte der angeheirateten Großtante von Marc (angeheirateten Großtante: weil sie die Ehefrau des Großonkels ist). 91.54.32.105 18:13, 14. Jan. 2023 (CET)
Aber beide sind mit Edeltraud (und deren Nachkommen) verwandt.
Und ansonsten gibt es keine Begriffe für solche Situationen? --2A0A:A541:FDDC:0:28D0:E784:ADE4:D0E0 18:27, 14. Jan. 2023 (CET)
Zum Kommentar mit den Stiefschwestern: Ich denke, er oder sie meinte seine oder ihre Halbschwestern. Ähnliches habe ich auch in der Verwandtschaft (wobei die inzwischen fast alle verstorben sind): Die erste Frau meines Großvaters (mütterlicherseits) hatte bereits einen Sohn, als er sie heiratete (ob sie verwitwet war oder der Sohn unehelich, das weiß nicht). Die beiden bekamen dann vier oder fünf Kinder, das müsste ich nachschauen (bis auf eine sind die inzwischen alle tot). Dann starb seine erste Frau, und mein Großvater heiratete meine Großmutter, mit der er dann auch noch einmal drei Kinder hatte (innerhalb von 20 Jahren). Die Kinder aus der ersten Ehe meines Großvaters waren also über ihre Mutter die Halbgeschwister seines Stiefsohnes und über ihren Vater die seiner jüngeren Kinder. --2A0A:A541:FDDC:0:28D0:E784:ADE4:D0E0 18:23, 14. Jan. 2023 (CET)u
Falsch! Es sind meine Halbschwestern, weil wir uns jeweils einen Elternteil "teilen" (A bzw. B). Die Halbschwestern von A/C und von B/D sind nicht miteinander verwandt. Mit Stiefgedöns hat das gar nichts zu tun. --31.212.244.31 18:37, 14. Jan. 2023 (CET)
Dein jetziger Beitrag (18:37, 14. Jan.) erklärt es mir besser als der vorausgegangene (17:48, 14. Jan.), in welchen du den Ausdruck »(Halb-)Schwestern« verwendet hast. Diesen Ausdruck habe ich so interpretiert, dass er »Schwestern« und »Halb-Schwestern« gleichermaßen umfassen soll. Nicht miteinander verwandt wären aber nur Halbschwestern der verschiedenen Abstammungen A/C und B/D. Eine »Schwester« von dir hätte aber – zumindest im Sinne von Blutsverwandtschaft – die gleiche Abstammung wie du selbst (nämlich A/B). Wäre sie dadurch nicht mit jeder deiner Halbschwestern in gleicher Weise verwandt wie du selbst?
MfG --Dirk123456 (Diskussion) 19:55, 14. Jan. 2023 (CET)
Doch, ist sie. Die A/B-Schwester habe ich außen vor gelassen, weil es für die Verwandtschaft unter A/C und B/D irrelevant ist. --31.212.244.31 11:48, 15. Jan. 2023 (CET)
Den Teil der Ursprungsfrage (17:30, 14. Jan.), ob es konkrete Bezeichnungen für die jeweiligen »Beziehungsrollen« von Lisa zu Marc und von Marc zu Lisa gäbe, muss man wohl verneinen. Selbst wenn man es mit komplexen Ausdrücken wie »Schwiegernichte soundsovielten Grades« versuchen würde, könnte man es kaum ausdrücken, an welcher Stelle im Beziehungsgefüge eine Partnerschaft maßgeblich wäre (»Schwieger-«) und an welcher Stelle ein gemeinsamer Vorfahr (umgangssprachlich »Blutsverwandtschaft«). Für so etwas gibt es keine einzelnen Wörter, die allgemein verstanden werden könnten, sondern man nimmt eher Schemata, um es zu erklären – bspw. Familienstammbäume. Ich könnte mir vorstellen, dass sich die drei Personen der Einfachheit halber eher mit »Lisa«, »Marc« und »Tante Edeltraut« anreden würden.
MfG --Dirk123456 (Diskussion) 11:28, 15. Jan. 2023 (CET)

Gebrochenes Aluminium

Es gibt Gusseisen, das bricht beim Biegen, das liegt an seinem hohen Kohlenstoffgehalt. Ich habe hier ein Aluminiumstück, das sich auch nicht biegen lässt, sondern bricht. Was ist die Ursache bei diesem Teil? Auch ein hoher Kohlenstoffanteil? --176.199.208.119 22:41, 14. Jan. 2023 (CET)

Wir haben dazu den Artikel Aluminiumlegierung, wo das alles erklärt wird. Aluminiumlegierungen enthalten keinen Kohlenstoff, obwohl bei der Aluminiumherstellung im Hall-Héroult-Prozess Kohleanoden verwendet werden. Diese reagieren aber mit dem entstehenden Sauerstoff zu gasförmigen Kohlenstoffoxiden und lösen sich nicht im Rohaluminium auf. --Rôtkæppchen₆₈ 23:09, 14. Jan. 2023 (CET)
Aluminium hat im Vergleich zu Gusseisen einen viel niedrigeren Kohlenstoffgehalt. Ein häufiger Grund, warum Aluminium beim Biegen bricht, ist die Anwesenheit von Fehlstellen wie Risse oder Porosität. Diese können entstehen, wenn das Aluminium nicht richtig behandelt wurde oder wenn es unter hohen Belastungen arbeitet. Es kann auch sein, dass das Aluminium eine niedrigere Wärmebehandlung hatte, was die Festigkeit beeinträchtigt. --2A02:3030:80C:B016:B3F:340F:2463:93D2 00:47, 15. Jan. 2023 (CET)
Antwort: Nein, es liegt nicht am Kohlenstoff, sondern an anderen Legierungsbestandteilen. Es gibt Al-Legierungen, die gut zu biegen sind und andere die hart und spröde sind. Spontan fällt mir AlMgSi05 als gut biegbare Legierung ein. Sehr hart aber eben nicht biegbar sind Legierungen mit Blei- oder Kupferanteil. Von "Flugzeugaluminium" mit Zinkanteil weiß ich zufällig, dass es sehr hart ist. Ob es gleichzeitig biegbar oder spröde ist, weiß ich nicht. Die Eigenschaften kann man im Metallhandel nachlesen. --31.212.244.31 11:33, 15. Jan. 2023 (CET)

Möbelschloss

Ich suche ein Möbelschloss, das sich ohne Schlüssel verschließen, aber nur mit Schlüssel wieder entriegeln lassen soll. Gibt es sowas und wie nennt sich das? Von Fenstern kenne ich etwas Ähnliches: Da ist im Griff ein Schloss, ist das reingedrückt, ist der Fenstergriff verriegelt und mit einem Schlüssel kann man das Schloss "rausspringen" lassen. --Magnus (Diskussion) 16:13, 12. Jan. 2023 (CET)

Warum nicht ein einfacher Schnapper, siehe Schlossfalle? --BlackEyedLion (Diskussion) 16:17, 12. Jan. 2023 (CET)
Rolladenschlösser funktionieren meistens wie gewünscht. --Ralf Roletschek (Diskussion) 16:26, 12. Jan. 2023 (CET)
Tja, naja, nicht so ganz. Die Tür soll normalerweise zu öffnen sein, ohne offenzustehen. Das Verschließen ist optional. Ein Druckzylinderschloss scheint das Gesuchte zu sein. --Magnus (Diskussion) 16:30, 12. Jan. 2023 (CET)
Sowas? --Ralf Roletschek (Diskussion) 16:37, 12. Jan. 2023 (CET)
Verschliessbare Schreibtische mit einer Lamellenabdeckung haben genau solche Schlösser. Also ja, so was gibt es. Den Rest erledigt eine Anfrage beim Beschlägehersteller deines Vertrauens. Yotwen (Diskussion) 22:25, 15. Jan. 2023 (CET)

Verrückte Idee: ChatGPT als Experimentator?

Haben schon mal Leute versucht, ChatGPT dazu zu bringen ein Experiment zu designen? Kann die KI das?--2A02:908:424:9D60:B4AA:6DE:2CF7:65DA 16:12, 14. Jan. 2023 (CET)

Warum sollte ChatGPT das können? Dafür ist die AI nicht trainiert. Es gibt außerdem genug spezialisierte AIs, die schon seit längerer Zeit zum Design von Experimenten genutzt werden (z.B. hier). ChatGPT ist dafür denkbar ungeeignet.--Zentraler Leser (Diskussion) 16:27, 14. Jan. 2023 (CET)
Das weiß ich. Ich bin nur Neugierig, wie viel der AI-Chatbot mit rein verbaler Beschreibung hinbekommt! --2A02:908:424:9D60:B4AA:6DE:2CF7:65DA 16:51, 14. Jan. 2023 (CET)
Warum fragst du das nicht den Bot? --95.116.152.237 21:09, 14. Jan. 2023 (CET)
+1 frage ChatGPT (erschreckend)--Wikiseidank (Diskussion) 22:05, 14. Jan. 2023 (CET)
Q: Haben schon mal Leute versucht, ChatGPT dazu zu bringen ein Experiment zu designen? A: Ich bin mir nicht sicher, ob jemand es direkt versucht hat, mich dazu zu bringen, ein Experiment zu entwerfen, aber ich kann dazu verwendet werden, um eine Vielzahl von Aufgaben im Zusammenhang mit Experimenten zu unterstützen. Zum Beispiel kann ich verwendet werden, um Hypothesen zu generieren, die in einem Experiment getestet werden sollen, oder um Datenanalysepläne zu erstellen. Ich kann auch verwendet werden, um den möglichen Auswirkungen von Experimenten auf verschiedene Ergebnisse vorherzusagen. --95.116.152.237 22:14, 14. Jan. 2023 (CET)
ChatGPT kann sowohl einfache z. B. chemische Experimente als auch Kochrezepte u.ä. "erfinden". Konkrete Mengenangaben sind aber schwierig und das Ergebnis ist nicht unbedingt zufriedenstellend oder sinnvoll. Außerdem gibt es einige allgemeine Floskeln wie "Beachten sie Sicherheitsmaßnahmen", "Folgen Sie den Anweisungen des Herstelles" und "Verwenden Sie geeignete Stoffe/Sensoren" o.ä. ohne wirklich konkret zu werden, wobei man mit mehrfachem Nachfragen teilweise konkretere (aber nicht unbedingt sinnvolle) Vorschläge enthält. -- Jonathan 13:24, 15. Jan. 2023 (CET)

Meereshöhe ermitteln

Hallo, ich bin gerade auf der Suche nach der Angabe, auf welcher Höhe über dem Meeresspiegel der Aorounga-Einschlagkrater liegt. In den Artikeln finde ich nichts und Google Earth zeigt die Meereshöhe leider auch nicht mehr an. --109.42.113.164 14:30, 15. Jan. 2023 (CET)

Mein Google-Earth zeigt mir an der Stelle des Wikipedia-Markers 678 Meter an. --Proofreader (Diskussion) 14:49, 15. Jan. 2023 (CET)
Topographic map Nga Ahorangi (Diskussion) 14:50, 15. Jan. 2023 (CET)
OpenTopoMap hat Höhenlinien --31.212.244.31 14:57, 15. Jan. 2023 (CET)

Zu wieviel Prozent ist Pompeji erforscht

Zu wieviel Prozent ist Pompeji erforscht beziehungsweise ausgegraben Danke im Voraus --Diefenbachmax (Diskussion) 15:03, 15. Jan. 2023 (CET)

Einfach selbst nachlesen: Pompeji#Moderne Archäologie: Von den 1920er Jahren bis heute: „Mittlerweile sind etwa zwei Drittel der Stadt freigelegt.“ --BlackEyedLion (Diskussion) 15:09, 15. Jan. 2023 (CET)

Was alles wird als "Ukraine-Kontaktgruppe" bezeichnet?

Was alles wird als "Ukraine-Kontaktgruppe" bezeichnet? --Gelöbnix (Diskussion) 18:34, 15. Jan. 2023 (CET)

.Was soll eigentlich mit dem zusammengestoppelten Wissen passieren? --scif (Diskussion) 18:51, 15. Jan. 2023 (CET)
Z.B. eine BKL angelegt werden ... --Gelöbnix (Diskussion) 19:14, 15. Jan. 2023 (CET) PS: vgl. Konferenz in Ramstein

Audials Radiotracker Bedienen

So funktioniert es:

  • ERSETZE diesen Abschnitt durch eine konkrete Frage.
  • Lass nur die nachfolgende Zeile (--95.91.220.124 19:16, 15. Jan. 2023 (CET)) am ENDE deiner Frage stehen. Sie wird in deine Signatur umgewandelt.

--95.91.220.124 19:16, 15. Jan. 2023 (CET)

Zu faul zum Googeln? Ergebnis. --158.181.68.122 19:21, 15. Jan. 2023 (CET)

Wahlergebnisse verfälschen durch (nur scheinbar) unabhängige Kandidaten

Ich habe gerade einen Artikel zur Wahlrechtsreform gelesen und eigentlich klingt das nicht so schlecht. Bei der Sonderregel für unabhängige Kandidaten frage ich mich aber, ob es da nicht Möglichkeiten gibt, die Zusammensetzung des Bundestags zu verfälschen. Beispiel: 10 Kandidaten der CSU treten bei der Bundestagswahl als unabhängige Kandidaten an, die CSU stellt in den Wahlkreisen keinen Kandidaten auf. Die CSU erhält Abgeordnete im Verhältnis zu den Zweitstimmen. Gewinnen die 10 Kandidaten, dann sitzen zusätzlich 10 weitere Abgeordnete im Bundestag, die sich nach der Wahl der CSU-Fraktion anschließen können. Gibt es irgendeinen Mechanismus, der einen solche "Optimierung" verhindern würde? --Carlos-X 19:49, 15. Jan. 2023 (CET)

Wenn das ganz offen gemacht wird, könnte man vielleicht die Wahl der Unabhängigen erfolgreich anfechten. Wenn es aber nicht gerichtsfest nachweisbar ist, wird man nichts dagegen machen können. Die Unabhängigen müssen auch gar nicht der Fraktion beitreten, bereits ein identisches Abstimmverhalten würde den Zweck erfüllen. Deswegen empfände ich eine Vergrößerung der Wahlkreise mit moderater Einschränkung des Länderproporzes als sinnvoller zur Mandatsreduzierung. In Zeiten des Internets ist es doch relativ egal, ob ein Abgeordneter 150.000 oder 250.000 Menschen vertritt. --95.222.50.72 20:07, 15. Jan. 2023 (CET)
Die Fraktionsgröße kann manchmal wichtig sein und das Wiedereintreten würde jeden Zweifel zur Unabhängigkeit ausräumen, aber natürlich wäre das Abstimmverhalten das Wichtigste. Auch eine Vergrößerung der Wahlkreise würde nicht alle Probleme lösen. Und schon heute wissen die meisten Menschen nicht, wen sie mit ihrer Erststimme wählen. --Carlos-X 20:23, 15. Jan. 2023 (CET)
Es kann sich sowieso jeder irgendeiner Fraktion anschliessen oder beliebige andere Gruppen bilden. --2001:9E8:A511:2F00:E0B8:2227:F7F3:E0AD 20:10, 15. Jan. 2023 (CET)
Das ist die Gegenwart. Aber es soll ja das Gesetz für die Zukunft gemacht werden. Da ist man noch offen für Veränderung des Bestehenden. --95.222.50.72 20:16, 15. Jan. 2023 (CET)
Das wird sich auch in Zukunft nicht ändern, denn eine Einschränkung des Freien Mandats der Bundestagsabgeordneten wäre mit ziemlicher Sicherheit ein Verstoß gegen Art. 38 Abs. 1 Satz 2 GG[16]. --2003:F7:DF0D:B800:4D44:A567:22AC:AD7F 22:45, 15. Jan. 2023 (CET)
Inwiefern? Wenn man z.B. nicht zu verschiedenen Fraktionen gleichzeitig gehören darf, verstößt das auch nicht gegen § 38. Dass grundsätzliche Entscheidungsfreiheit besteht, bedeutet nicht, dass man sich an keinerlei Regeln halten muss. Aber die Fraktionszugehörigkeit ist wie oben erwähnt eh sekundär, es geht mehr um das Abstimmungsverhalten. --95.222.50.72 00:09, 16. Jan. 2023 (CET)
Ein Präzedenzfall könnte der Fall der "Jungen Liste" bei der Münchener Kommunalwahl von 1990 sein. Auch wenn diese wohl ziemlich offensichtlich eine Nebenorganisation der CSU war, führten die Nichtzulassung und die daraus resultierenden Gerichtsverfahren dazu, dass die Kommunalwahl 1994 wiederholt werden mussten [17]. Nun handelt es sich natürlich um Wahlen auf einer anderen Ebene und zu einer anderen Zeit, aber zeigt vielleicht doch auf, dass eine Nichtzulassung aufgrund angenommener taktischer Tarnkandidaturen zum Nutzen einer ebenfalls antretenden Partei von Gerichten sehr kritisch geprüft würde. -- 79.91.113.116 11:32, 16. Jan. 2023 (CET)
Das ist in der Tat interessant, auch wenn es in meinem Beispiel keinen zweiten Wahlvorschlag geben würde. Ich habe mir noch die darüber Gedanken gemacht, dass 2 Kandidaten derselben Partei verboten sind. --Carlos-X 11:57, 16. Jan. 2023 (CET)

wird der Verstoß zugegeben?

Wenn man in flagranti von der Polizei bei einem Verkehrsverstoß erwischt wird, soll man ein Formular ausfüllen und unterschreiben. Man soll ankreuzen, ob der Verstoß zugegeben wird oder nicht. Zu was dient das, wenn man das Formular sowieso nicht ausfüllen muss? Ist das ein Intelligenztest?

--2001:9E8:A51F:3600:2B06:847F:E599:9F7B 19:49, 13. Jan. 2023 (CET)

Anekdote meinerseits: Ich habe beim Delikt Fahren auf Feldweg beides ausprobiert. Den Verstoß zuzugeben war etwa halb so teuer. Spart vermutlich Verwaltungsaufwand. Beim ersten Mal dachte ich, es wäre ein Test, um mir Vorsatz nachzuweisen, und hab nichts angekreuzt. Dem war aber nicht so. —2A02:8071:52D0:3820:A4F8:3CF:3F3E:253F 21:38, 13. Jan. 2023 (CET)
Also doch ein Intelligenztest. Wer schlau ist, kreuzt das Richtige an. --92.73.145.117 21:43, 13. Jan. 2023 (CET)
Lies doch einfach die beigefügte Rechtsfolgenbelehrung. Dann musst Du nicht so hilflose Spekulationen betreiben und falsche Schlussfolgerungen daraus ziehen. Der Intelligenztest besteht darin, dass Du erkennst, dass die Frage zur Wahrung Deiner verfassungsmäßigen Rechte dient und mitnichten dazu, Dich trickreich zu irgendwelche Einlassungen hinzureißen, die Dich rechtlich schlechterstellen. Und selbst wenn es zu einem Bußgeldbescheid oder Strafbefehl kommt – auch den musst Du nicht akzeptieren, sondern kannst Rechtsmittel dagegen einlegen. --2003:F7:DF0D:B800:9DBA:2C45:102B:C2B5 21:50, 13. Jan. 2023 (CET)
Lies doch einfach die beigefügte Rechtsfolgenbelehrung.
Die Bedeutung von deren Inhalt muß man als Nicht-Jurist ja auch erst mal korrekt verstehen. Normalerweise hat man in solchen Situationen keinen Juristen zur Hand, um sich das Geschwurbel dieser Belehrung in ein verständliches Deutsch übersetzen lassen, vor allem hinsichtlich aller Konsequenzen, die daraus erwachsen können, was dort alles steht. Selbst die anwesenden Beamten sind regelmäßig damit überfordert, es vollständig und korrekt zu erklären, wenn man eine Frage zu diesem Text hat. --92.73.145.117 22:40, 13. Jan. 2023 (CET)
Wenn Dich die rechtlichen Folgen von Rechtsverstößen geistig überfordern, nimm doch einfach einen Rechtsanwalt oder verzichte auf Rechtsverstöße. Und entwickle hier bitte keine abstrusen Privattheorien. Dafür ist das Café nebenan da. --2003:F7:DF0D:B800:9DBA:2C45:102B:C2B5 23:50, 13. Jan. 2023 (CET)
Das dient aus Sicht der Bußgeldbehörde der Verwaltungsvereinfachung, weil zugegebene Verstöße ohne weitere Ermittlungen abgearbeitet werden können. Aus Sicht des Betroffenen dient es zur möglichen Geldersparnis, weil zugegebene geringfügige Verstöße eventuell (=behördenabhängig) mit einem Verwarnungsgeldangebot statt mit einem Bußgeldbescheid geahndet werden, was dem Betroffenen rund 30 Euro Verwaltungs- und Zustellgebühren erspart. Handlungsvorschlag: Geringfügige Verstöße stets zugeben, sich zu anderen Verstößen nicht äußern. --Rudolph Buch (Diskussion) 22:25, 13. Jan. 2023 (CET)
Daß kann eigentlich nicht sein. Denn Verwarnungsgelder sind viel unkomplizierter für die Behörde als Bußgelder. Verwarngelder kann eine Behörde im Prinzip nach belieben ggü. jedem aussprechen. Die Behörde könnte dir morgen ein Verwarngeld über 10€ senden, weil die zu viele Bananen gegessen hast. Das ist im Prinzip das gleiche wie wenn ich dir schreibe: "Rudolph du hast heute zu viel gepostet, ich schlage vor du bezahlst mir 10€". Ein Bußgeld dagegen ist eine ganz andere Hausnummer. Deswegen kann die Behörde bei einem zugegebenen Verstoß problemlos ein Bußgeld verhängen. --2001:9E8:A538:7A00:9313:7E3F:58CA:E97E 10:45, 14. Jan. 2023 (CET)
"Verwarngelder kann eine Behörde im Prinzip nach belieben ggü. jedem aussprechen." Aha, ist mir neu. Die Rentenversicherung kann mir also ein Verwarngeld aufbrummen, weil ich meinen Rasen nicht gemäht habe? Wow! --95.116.152.237 15:16, 14. Jan. 2023 (CET)
Ach du je…
Ich denke es mir zwar oft, aber sage bzw. schreibe es nur selten, hier muss ich aber einfach einhaken: selten so einen kompletten Unsinn gelesen.
Keine deutsche Behörde kann irgendwas einfach nach Belieben machen, für alles, was in die Rechte eines Bürgers eingreift, braucht diese eine Ermächtigungsgrundlage, also ein Gesetz oder eine Verordnung, welche das erlaubt.
Die gesamte „Bestrafungs-Sache“ ist in Deutschland mehrstufig aufgebaut: Für schwere Vergehen und Verbrechen gibt es das Strafrecht mit vollem Programm, also Staatsanwaltschaft, Gerichtsurteile usw.
Bei leichteren Vergehen gibt es das Bußgeldrecht, aber auch nur dort, wo es rechtlich vorgesehen ist. Hier kommen wir dann bereits zur Ausgangsfrage, wo das Kreuzchen einen erheblichen Unterschied in der Bearbeitung und in den Kosten macht. Ist der Delinquent einsichtig und gibt sein Vergehen zu, ist es unwahrscheinlich und auch wenig erfolgversprechend, wenn er dann hinterher mit einem Anwalt um die Ecke kommt und alles anfechtet. Standardisierter Bußgeldbescheid wird verschickt, Bußgeld wird gezahlt – wenig Aufwand und Sache ist erledigt. Würde ich bei Bußgeldsachen aber nicht immer empfehlen, bei manchen Vergehen kann das Schuldeingeständnis als Vorsatz gedeutet werden und dann verdoppelt sich das Bußgeld ganz schnell. Weiterhin werden Vergehen im Bußgeldbereich auch aktenkundig gemacht und wenn man da schon mehrere gleichartige Vergehen gesammelt hat, gibt es kein Bußgeld mehr, sondern es kommt gleich der Staatsanwalt. Klassisches Beispiel sind hier notorische Raser, die innerhalb kurzer Zeit mehrere Geschwindigkeitsverstöße gesammelt haben.
Die geringste Stufe ist dann das Verwarnungsgeld. Hier würde ich das Kreuz beim Schuldeingeständnis fast uneingeschränkt empfehlen. Nach Bezahlung der Verwarnung behält man hier im Regelfall seine weiße Weste, lediglich bei Fahranfängern in der Probezeit könnte das böse enden. Wenn man das Verwarnungsgeld nicht annimmt, ist man automatisch im Bußgeldverfahren mit erhöhten (Verwaltungs-)Kosten. Das sollte man also nur machen, wenn man den Verstoß komplett anfechten möchte.
46.227.217.186 22:58, 16. Jan. 2023 (CET)

Wieviel Leute braucht man für einen Leopard-Panzer?

Wie viele geschulte Leute braucht man, um einen Leopard-Panzer zu fahren, einzusetzen, zu warten, zu reparieren und Ersatzteile zu besorgen? Da so ein Panzer sich doch deutlich von einem Bobby-Car unterscheidet, kann man mit ihm sicher nicht mal einfach morgens in die Schlacht fahren und ihn abends wieder in die Garage stellen. Es gibt doch bestimmt regelmässig Fristen, alle 200km muß die Kette nachgespannt werden, alle 30 Schuß muß der Munitionsfördergurt gewechselt werden und solche Dinge (die ich hier beispielhaft erfunden habe). Man muß ganz bestimmte Schmiermittel auf Lager haben und einiges an Spezialwerkzeug ist sicher auch nötig. Und wie lange dauert es dann, bis man die notwendige Gruppe von Leuten ausgebildet hat? Lernt man so eine Panzerelektronik in einer Woche? --2001:9E8:A511:2F00:E0B8:2227:F7F3:E0AD 17:15, 15. Jan. 2023 (CET)

Welchen Leopard meinst Du: Leopard 1 oder Leopard 2? Welche Ausführung: A1, A2, A3, A4, 1A5 usw. bzw. 2A0, 2A1, 2A2, 2A3, 2A4 usw.? --158.181.68.122 18:02, 15. Jan. 2023 (CET)
Einen. --Gelöbnix (Diskussion) 18:33, 15. Jan. 2023 (CET) PS: Und eine funktionierende Panzerfaust.
Also Leopard 2 werden ja unter anderem bei der Bundeswehr eingesetzt. Dort umfasst ein Panzerbataillon in etwa 500 Soldaten, die nach unserem Artikel in etwa 44 Leopard 2 betreuen. Das könnte in etwa hinweise darauf geben wieviele man benötigt. Zumeist in der NATO umfasst eine Kompanie mindestens 14 Panzer, drunter wirst du die Zahlen wohl kaum bekommen können (denn einzelne Panzer machen wenig Sinn).--Maphry (Diskussion) 18:44, 15. Jan. 2023 (CET)
also 14 Kampfpanzer mit 100-150 Leuten. Da ist dann aber kein Nachschub und keine Instandhaltung dabei? --2001:9E8:A511:2F00:E0B8:2227:F7F3:E0AD 20:23, 15. Jan. 2023 (CET)
Ein Bataillon umfasst neben den Kampfkompanien noch eine Stabs- und Versorgungskompanie, in der u.a. Nachschub und Instandsetzung angesiedelt sind. --Digamma (Diskussion) 21:31, 15. Jan. 2023 (CET)

Hinweis @IP 2001:9e8:a511:2f00:e0b8:2227:f7f3:e0ad: Das Löschen von Diskussionsbeiträgen lassen wir für die Zukunft mal schön sein. Hier verbessert ja auch niemand Deine mangelhafte Orthografie und Typografie. --158.181.68.122 21:26, 15. Jan. 2023 (CET)

@IP 2001:9e8:a511:2f00:e0b8:2227:f7f3:e0ad: Drücke Dich klar aus, dann braucht man nicht nachzufragen. In der Entwicklung vom Leopard 1 A1 zum Leopard 2 A7 wurde jeweils darauf Wert gelegt, den Wartungsaufwand zu minimieren, was dementsprechend unterschiedliche Zahlen des Wartungspersonals zur Folge hat. --158.181.68.122 00:11, 16. Jan. 2023 (CET)

Wenn ich dir sage "Leopard 2A4" dann wirst du mir trotzdem keine genauen Zahlen liefern, also was soll das? --2001:9E8:A528:9300:2FE6:F278:10BE:C5D7 09:45, 16. Jan. 2023 (CET)
Das weißt Du also vorher? Weshalb fragst Du dann eigentlich? Ich hätte mir vielleicht die Mühe gemacht und in meiner umfangreichen Literatursammlung als Modellbauer mit Schwerpunkt Leopard 1 & 2 nachgeschlagen, aber für Dich lohnt es sich nicht wirklich. Ende Gelände  Vorlage:Smiley/Wartung/;-) . --158.181.68.122 12:27, 16. Jan. 2023 (CET)
Ja genau, als wenn Du die Gelegenheit zum Klugschnacken hättest vergehen lassen... --157.143.23.117 18:19, 16. Jan. 2023 (CET)
Die Frage ist ja oben beantwortet. Genauer wird es nicht sinnvoll gehen. Sollte jmd. in der Tat Zahlen haben, die sich auf einzelne Ausführungen beziehen; nur zu, wäre ja Sicherlich auch interessant. ...Sicherlich Post 09:52, 16. Jan. 2023 (CET)

Hello!

Ist Hello! oder HELLO! das korrekte Lemma für Hello!? --fossa net ?! 18:50, 15. Jan. 2023 (CET)

Schaust Du hier: Wikipedia:Namenskonventionen#Unternehmen, Wikipedia:Namenskonventionen#Abweichungen von den Rechtschreibregeln und hier. --158.181.68.122 19:06, 15. Jan. 2023 (CET)
Thx, das angewandt würde in Hello resultieren. Das halte ich für ausgemachten Quark. Ich sehe bei ¡Hola! hat man sich zum Schaden eines sorbischen Dorfes danach vorgegangen. Ich plädiere für WP:IAR. --fossa net ?! 19:20, 15. Jan. 2023 (CET)
Dann eher so: Hello!. Laut Impressum ist das Ausrufezeichen ein Namensbestandteil. --158.181.68.122 19:28, 15. Jan. 2023 (CET)

Das braucht dann aber einen Klammerzusatz zur Differenzierung vom Status-Quo-Album. Jedenfalls gibt es in der en WP en:Hello! (magazine) für die Zeitschrift und en:Hello! (album) für Status Quo. --Proofreader (Diskussion) 20:00, 15. Jan. 2023 (CET)

Nicht unbedingt. Mit dem Status Qiluo Album muß man nicht rechnen, de:WP ist bezüglich Alben-Lemmas ziemlich restriktiv, verglichen mit en:WP. -Ani--46.114.153.141 20:45, 15. Jan. 2023 (CET)
Okay, also ohne Zusatz; falls das Album doch noch einen Artikel bekommt, kann man den Artikel zum Magazin ja immer noch auf ein Lemma mit Zusatz verschieben. Hello! scheint mir ansonsten auch die sinnvollste Schreibung hier zu sein. --Proofreader (Diskussion) 22:04, 15. Jan. 2023 (CET)
Hallo @user:Proofreader, dann siehst Du das hier doch sicher ähnlich. --fossa net ?! 22:25, 15. Jan. 2023 (CET)
Ja, zumal alle anderen Interwikis, bis auf die türkische, den Artikel ebenfalls mit Ausrufungszeichen im Lemma haben. --Proofreader (Diskussion) 22:33, 15. Jan. 2023 (CET)
@Proofreader: Sollte der Fall des Klammerlemmas mal eintreten, stellt sich die Frage: Hello! (Zeitschrift) oder Hello! (Magazin), da es anscheinend dafür keine einheitliche Regel gibt. --158.181.68.122 00:30, 16. Jan. 2023 (CET)
Ausnahmen von dieser Regel können in solchen Fällen gemacht werden, wo eine Anpassung verwirren würde oder wenn die unkonventionelle Schreibung eindeutig die üblichere ist und Wortverbindungen nicht stört. In diesem Fall würde ein "Hello" ohne Ausrufezeichen eher verwirren, weil es dann eine Verwechselungsmöglichkeit mit dem englischen Wort geben würde. Also Hello!. Sollte im unwahrscheinlichen Fall ein Klammerlemma notwendig sein, würde ich tendenziell Hello (Magazin) oder Hello! (Magazin) bevorzugen, da das ähnlicher zu deren offiziellen Webseite 'hellomagazine.com' wäre, durch den Klammerzusatz eliminiert man dann auch die Mehrdeutigkeit, sodass ich das Ausrufezeichen dann für nicht erforderlich halten würde. -- Jonathan 10:01, 16. Jan. 2023 (CET)
In dem Fall ist ein Klammerlemma völlig unnötig, da es keine zwei Beiträge mit dem Lemma Hello! gibt. Man könnte natürlich in der Begriffsklärung Hello vorher die zwei Klammerlemmas Hello! (Magazin) und Hello! (Album) aufführen, dann hätte Hello! (Magazin) zumindest eine Daseinsberechtigung. --158.181.68.122 12:18, 16. Jan. 2023 (CET)

Hinweis: Für Fragen speziell zur Wikipedia gibt es ein eigenes Diskussionsforum: Fragen zur Wikipedia.

One Drop Rule

 
One Drop. Schwarz?

Wurde die in amerikanischen Rassengesetzen des 20. Jahrhunderts enthaltene One-Drop-Rule wirklich konsequent verfolgt und eingehalten, galt also jeder mit einem einzigen nachgewiesenen schwarzen Vorfahren als schwarz und wurde so behandelt, selbst wenn es wirklich nur "ein Tropfen" schwarzen Blutes war? Als Beispiel habe ich hier eine Frau aufgeführt, die wirklich im wörtlichsten Sinne "one drop" ist und wahrscheinlich kein einziges nachweisbares afrikanisches Gen haben dürfte.

--2A0A:A541:FDDC:0:1460:455:18BE:BDF5 05:54, 15. Jan. 2023 (CET)

Die en:WP sagt:
"20. Jahrhundert und heutige Zeit
Im Amerika des 20. Jahrhunderts wurde das Konzept der One-Drop-Regel vor allem von weißen Amerikanern auf Menschen schwarzafrikanischer Abstammung aus Ländern südlich der Sahara angewandt, als einige Weiße versuchten, ein gewisses Maß an offener oder verdeckter weißer Vorherrschaft aufrechtzuerhalten.
Diese Regel bedeutete, dass viele Menschen mit gemischter Hautfarbe und unterschiedlicher Abstammung einfach als Afroamerikaner angesehen wurden und ihre vielfältigeren Vorfahren vergessen und ausgelöscht wurden, was eine genaue Rückverfolgung der Abstammung in der heutigen Zeit erschwert.
Viele Nachkommen derer, die von Europäern und Amerikanern versklavt und verschleppt wurden, gehen davon aus, dass sie von den amerikanischen Ureinwohnern abstammen. Die PBS-Dokumentation von Henry Louis Gates Jr. aus dem Jahr 2006 über die genetische Ausstattung von Afroamerikanern, African American Lives, konzentrierte sich auf diese Geschichten über das Erbe der amerikanischen Ureinwohner in afroamerikanischen Gemeinschaften. Die Ergebnisse von DNA-Tests zeigten, dass alle bis auf zwei der interviewten Prominenten nach afrikanischen vor allem europäische Vorfahren hatten.[14] Viele Kritiker weisen jedoch auf die Grenzen von DNA-Tests zur Bestimmung der Abstammung hin, insbesondere bei Minderheiten.[15][16][17]
Während des Zweiten Weltkriegs erklärte Oberst Karl Bendetsen, dass jeder, der auch nur einen Tropfen japanisches Blut" in sich trage, zwangsweise in Lagern interniert werden müsse[18].
Heute gibt es in den USA keine durchsetzbaren Gesetze, die die Ein-Tropfen-Regel anwenden. Soziologisch gesehen ist das Konzept zwar in den letzten Jahren in der schwarzen Gemeinschaft weniger akzeptabel geworden, da sich immer mehr Menschen als gemischtrassig bezeichnen, doch haben Untersuchungen ergeben, dass es in der weißen Gesellschaft immer noch üblich ist, gemischtrassige Kinder in erster Linie mit der nicht-weißen Abstammung der Person in Verbindung zu bringen.[1][19]"
Personen, die in der Evolution des H.s. bewandert sind, wissen, dass diese Regel ein mehr als willkürliches Konzept ist.
Was die gezeigte Dame betrifft: Vielleicht hat sie ihren Vater (Man in Black) nur falsch verstanden ? Nga Ahorangi (Diskussion) 09:23, 15. Jan. 2023 (CET)
Ihre Mutter Vivian Cash hat ein Schwarze Ur-Urgroßmutter: "Vivian Cash’s maternal great-great grandmother was indeed an enslaved Black woman, Sarah Shields, whose White father in 1848 had granted her and her eight siblings their freedom and their passage into Whiteness, too. Shields married a White man — albeit illegally — and by the time Jim Crow arrived in the 1930s all of her children and their descendants were listed as White", laut diesem Washington-Post-Artikel --Digamma (Diskussion) 21:18, 15. Jan. 2023 (CET)
Also, in en:WP steht, Sarah Shields' Hochzeit sei (während des Sezessionskrieges!) ganz offiziell beurkundet und das alles von ihrem Vater bezahlt worden. Wie viel Prozent weiße Vorfahren die hatte, wird übrigens nirgends erwähnt (wahrscheinlich unbekannt?), aber es wird wohl mindestens 50 Prozent, eher mehr gewesen sein. Sarah Shields stellt ein 1/32 von Rosanne Cashs Urururgroßelterngeneration dar, wenn es also nicht noch andere schwarze Vorfahren gibt, hat Rosanne Cash allerhöchstens 1/64 afrikanische Abstammung und sieht ja auch vollkommen weiß aus. Aber nach One-Drop-Rule müsste sie ja als schwarz oder farbig gelten, oder? Und die Frage hier ist, ob das auch in solchen Fällen konsequent umgesetzt wurde, als diese Regel galt. --Universal-InteressierterDisk.Arbeit 00:26, 16. Jan. 2023 (CET)
Die Regel wurde ja offensichtlich schon bei Sarah Shields und ihren Nachkommen nicht konsequent umgesetzt. Vivian Cash, die Mutter von Rosanne Cash wurde aber laut dem verlinkten Artikel teilweise durchaus als schwarz wahrgenommen, was ja die ganze Diskussion erst ausgelöst hat. Wenn Rosanne Cash zu 1/64 afrikanischer Abstammung ist, dann hat sie jede Menge "afrikanischer Gene", auch wenn sie ihr Aussehen vor allem von ihrem Vater geerbt hat. --Digamma (Diskussion) 09:11, 16. Jan. 2023 (CET)
Die (nie formal so genannte) one-drop-rule wurde formal bundesweit eingeführt mit dem 14.Census im Jahr 1920 ("The term "white" as used in the census report refers to persons understood to be pure-blooded whites. A person of mixed blood is classified according to the nonwhite racial strain .... thus a person of mixed white ... and Negro ... is classified as ... a Negro ... regardless of the amount of white blood."). Bei der Debatte um "mulattoes" ging es darum, Kinder aus gemischten Beziehungen (in der Regel mit weißem Vater und schwarzer Mutter) gezielt zu benachteiligen (Misch-Ehen waren sowieso verboten). Die Gesetze in den Sklavenhalterstaaten (Etwa Virginia und North Carolina) gingen zurück bis zu den Ururgroßeltern, also einem Sechzehntel des "Blutes", um "mulattoes" von "whites" zu unterscheiden. Später wurden dann "mulattoes" und "blacks" zusammengeworfen. Auf drops auch jenseits des Sechzehntels dürfte das auch angewendet worden sein, wenn es denn bekannt war, dann aber wohl eher sozial als juristisch.--Meloe (Diskussion) 10:44, 16. Jan. 2023 (CET)
Alle heute lebenden Menschen stammen von Afrikanern ab. Hier in Europa kam die weiße Hautfarbe erst späte, vermutlich mit der Einwanderung von Steppenvölkern aus Zentralasien, bei denen sich das Merkmal irgendwann entwickelt hat. Um da irgendwas bestimmen zu können, müßte man erst mal festlegen, ab wann die Europäer (bzw. die Eroberer Amerikas) nicht mehr als Nachkommen der eingewanderten Afrikaner gelten sollen, sondern als eigene Rasse. --178.4.186.178 15:38, 16. Jan. 2023 (CET)
Nein müsste man nicht. Man müsste einfach alle vorhandenen Gene des Menschen einem bestimmten Phänotyp zuordnen und daran scheitern. Das ist bereits passiert und damit ist die ganze schöne Rassenlehre der vergangenen Jahhunderte dahin und nicht wissenschaftlich fundierbar. Die Rassenzuordnung des US-Zensus ist damit Verschwörungstheorie. --2003:F7:DF07:DE00:357A:A0DC:E357:3315 23:04, 16. Jan. 2023 (CET)
Was ist da Verschwörung? --Sabrieleauftistik (Diskussion) 10:29, 17. Jan. 2023 (CET)

E-Scooter ohne Versicherungskennzeichen, mit ausgeschaltetem Motor bewegen

Bin seit gestern Besitzer eines E-Scooters mit ABE, der noch kein Versicherungskennzeichen hat. Da ich momentan nicht gut zu Fuß bin, wollte ich damit zu einer Versicherungsagentur fahren, um mir ein Versicherungskennzeichen zu besorgen. Da ich dazu nichts gefunden habe, nun die Fragen:

  • Darf ich den E-Scooter auf öffentlichen Straßenland mit ausgeschaltetem Motor per Fuß bewegen (wie z. B. einen Tretroller), wenn er kein Versicherungskennzeichen hat? Zur Sicherheit würde ich ein Handtuch um das Bedienelement wickeln und es mit Paketband mehrfach fixieren, um ein An- oder Ausschalten zu unterbinden.
  • Darf ich auf dem Bürgersteig ohne Motorantrieb mit ausgeschaltetem Bediendisplay fahren, da ich wohl kaum diese „magischen“ 6 km/h erreiche?

Falls das mit den genannten Maßnahmen nicht möglich ist, was muss ich machen, um es zu ermöglichen (z. B. Akku abklemmen/ausbauen)?

Anmerkung: Meine Mutter hatte zu ihren Lebzeiten einen elektrischen Rollstuhl ohne Versicherungskennzeichen, der keine 6 km/h erreichte. Mit dem durfte sie überall fahren (außer auf Kraftfahrstraßen und Autobahnen). --158.181.68.122 18:30, 15. Jan. 2023 (CET)

Für Krankenfahrstühle gelten besondere Regeln. --Digamma (Diskussion) 21:24, 15. Jan. 2023 (CET)
Und Wikipedia hat auch einen wunderbaren Artikel dazu: Krankenfahrstuhl (Kraftfahrzeug). --158.181.68.122 21:38, 15. Jan. 2023 (CET)

Kein Fachmann on Board? --158.181.68.122 20:38, 16. Jan. 2023 (CET)

Wenn Du nicht gut zu Fuß bist, solltest Du darauf verzichten, auch noch einen schweren E-Scooter durch die Gegend zu schieben. Nimm besser einen Mietscooter. Den darfst Du fahren, aber natürlich nicht auf dem Bürgersteig. --2003:F7:DF07:DE00:357A:A0DC:E357:3315 23:50, 16. Jan. 2023 (CET)
Nicht schieben, sondern rollern, wie mit einem Tretroller. Mietroller kommt nicht infrage (kein Tarif mit Datennutzung, App, Kontoverbindung, Adresse, Registrierung, eventuelle Schäden), deshalb habe ich mir zwei E-Scooter gekauft, einen leichten mit 25 Kilometer Reichweite und einen größeren mit 50 Kilometer Reichweite. --158.181.68.122 14:35, 17. Jan. 2023 (CET)

Vorteil von Tempo 130

Hallo! Nein, keine Cafediskussion geplant, sondern ein Verständnisproblem für mich. Warum ist Tempo 130 so wesentlich für den Klimaschutz?

Beispiel die Strecke Berlin-München, ca. 650km. Mit Tempo 100 = 6,5h , mit Tempo 130 = 5h, mit Tempo 150 = knapp 4h 20'.

Ich kenne nur Vergleiche zum direkten Verbrauch auf 100km, aber ist die Technik wirklich so, das wesentlich mehr CO2 bei diesem Weg ausgestoßen wird? Also ist der CO2 Ausstoß von 40 Minuten mit Tempo 130 geringer als der vom Unterschied der 260 Minuten mit Tempo 150?Oliver S.Y. (Diskussion) 19:12, 14. Jan. 2023 (CET)

Bei einem Tempolimit ist der Verkehrsfluss für alle deutlich gleichmäßiger, es kommt zu weniger Abbrems- und Beschleunigungsmanövern. Und die Beschleunigungsphasen sind halt Spritfresser und damit CO2 Produzenten. Gibt eventuell auch noch andere Faktoren, aber das ist soweit ich weiß ein wichtiger. Kritzolina (Diskussion) 19:17, 14. Jan. 2023 (CET)
Stimmt; aber ob schnelles Fahren häufige Abbrems- und Beschleunigungsmanöver erfordert, hängt ja auch sehr davon ab, wie viele Autos sonst noch so auf dem fraglichen Streckenabschnitt unterwegs sind. In Innenstädten muss man trotz Tempo 50 und selbst bei grüner Welle mitunter ständig bremsen und beschleunigen. Aber wahrscheinlich ist das ein Grund, warum Stadtverkehr im Ruf steht, viel mehr Energie zu verbrauchen als Überlandverkehr. --Sabrieleauftistik (Diskussion) 19:51, 14. Jan. 2023 (CET)
Keine Antwort, eher ein Einwurf: Da der Luftwiderstand wohl nichtlinear mit der Geschwindigkeit zunimmt, gilt das auch für den Energieaufwand (dafür, die Geschwindigkeit zu halten) und beim Verbrenner dementsprechend für den CO2-Ausstoß. Ich gehe nicht davon aus, dass speziell Tempo 130 da die optimale Geschwindigkeit ist – es gibt ja auch andere Forderungen –, vielmehr dürfte das irgendeine Art Kompromiss sein. Das Ganze ist sowieso eine eher politische Überlegung, denn für E-Autos (auf die der deutsche Individualverkehr ja möglichst schnell umgestellt werden soll) gilt obige Argumentation nicht (zumindest nicht unmittelbar; es kommt darauf an, woher der Strom kommt, mit dem man sie betankt). Spätestens nach erfolgter Elektrifizierung und Umstellung der deutschen Energieversorgung auf 100 % Ökostrom wäre ein Tempolimit also überhaupt nicht wesentlich für den Klimaschutz (dann allerdings wahrscheinlich de facto gegeben, weil E-Autos im für Normalbürger bezahlbaren Preissegment nicht schneller als ca. 140 fahren). Ob bis dahin ein Tempolimit aus Klimaschutzsicht sinnig wäre, ist eine andere Frage. --Sabrieleauftistik (Diskussion) 19:24, 14. Jan. 2023 (CET)
Gerade beim Elektromotor, dessen Wirkungsgrad wenig von der Drehzahl abhängt, ist der Energieverbrauch pro km bei konstanter Geschwindigkeit und ebener Straße direkt proportional zur Summe aus Reibungskraft und Luftwiderstand. Der Anteil, der auf den Luftwiderstand zurückzuführen ist, nimmt aber mit der Geschwindigkeit zu. --Digamma (Diskussion) 21:35, 14. Jan. 2023 (CET)
Aber gerade bei E-Autos kann man das ja vom CO2-Ausstoß völlig entkoppeln. Ist die Relation denn bei Verbrennern wesentlich anders? --Sabrieleauftistik (Diskussion) 22:50, 14. Jan. 2023 (CET)
Kurzer Einwurf zu "weil E-Autos im für Normalbürger bezahlbaren Preissegment nicht schneller als ca. 140 fahren": Von den zehn 2022 meistverkauften E-Auto-Modellen in Deutschland fährt eines (Dacia Spring) 125 und eines (e-Up) 130 km/h, zwei (Corsa-e und Fiat 500) 150 km/h und alle anderen mindestens 160 km/h, teilweise deutlich mehr. --Karotte Zwo (Diskussion) 10:30, 18. Jan. 2023 (CET)
Es ist nicht wesentlich für den Klimaschutz. Wer sagt sowas? Die 130-Diskussion ist doch aufgrund der Verkehrssicherheit und nicht wegen dem Klima. Die ARD führt hier aber noch auf, dass es doch indirekt für das Klima gut sein könnte, da, wenn die Autos langsamer sind, diese auch unattraktiver werden und mehr Leute die Bahn nehmen würden. --Christian140 (Diskussion) 19:59, 14. Jan. 2023 (CET)
Das würde ich für den ländlichen Raum abseits der großen ICE-Strecken aus persönlicher Erfahrung eher verneinen, da müsste man die Autos weit mehr als auf 130 runterregeln, damit die Bahn attraktiver wird (Umsteigezeiten!). Und innerhalb von Städten dürften es auf den Autobahnen an den meisten Orten eh schon Tempolimits geben (aber innerhalb von Städten ist der ÖPNV auch schon wieder ganz anders aufgestellt). Die Verkehrssicherheit will ich nicht vertiefen, das ist ein weites Feld (fehlende Blitzer…). --Sabrieleauftistik (Diskussion) 20:05, 14. Jan. 2023 (CET)
Für mein persönliches Empfinden ist im Zug zu sitzen und zu reisen weitaus bequemer (komfortabler) als im Auto zu sitzen und zu reisen. --2A0A:A541:FDDC:0:61AA:613A:151A:1ABA 02:29, 15. Jan. 2023 (CET)
Verkehrssicherheit bzgl. 130 scheint mir nicht sehr plausibel: die Autobahnen gelten als die sichersten Straßen Deutschlands; Landstraße und Innerorts sind viel riskanter für Leib und Leben. Für innerorts wird zumindest gelegentlich 30 diskutiert, von einer 80km/h-Disk für Landstraße habe ich noch nie was gehört!? Und wenn auf auf Autobahnabschnitten wegen Geschwindigkeiten zu mehr Unfällen kommt werden ja schon jetzt Tempolimits eingeführt.
wenn man die prognostizierten CO2-Einsparungen in % zum deutschen Gesamt-CO2 sieht; da hat der Tropfen auf den heißen Stein mehr Wirkung!? Quarks sagte 1,9 Mio t CO2. Deutschlands Gesamt-CO2 sind 678 Mio t --> 0,28% ...
und ich befürchte das geht jetzt zu sehr Richtung Cafe; wird die Disk. ums Tempolimit ggf. eher gefühlt (bewusst oder unbewusst) um sich nicht mit wirklichen Einschnitten beschäftigen zu müssen? 🤔 ...Sicherlich Post 21:08, 14. Jan. 2023 (CET)
Na ja, es ist immer eine Frage der Art der Aussage. Die Tempolimit-Befürworter haben ja durchaus Recht, dass dadurch CO2 eingespart wird. Das ist aber erst mal eine rein qualitative Aussage. Die Einsparung kann so gering sein, dass man mit der Maßnahme zwar den Störwillen der Letzten Generation stillen könnte, den Einfluss Deutschlands auf das Weltklima aber praktisch überhaupt nicht. (Es ist ja nicht einmal die nominelle Einsparung, die man durch die geringere Geschwindigkeit rechnerisch erhält. Weiß man überhaupt, wie viele Menschen wie oft schneller als 130, 140, 150 km/h fahren? Letztere dürften eher selten sein, und dann wäre ja auch noch die Frage, wie viele von denen sich an ein Tempolimit überhaupt halten würden.) --Sabrieleauftistik (Diskussion) 21:31, 14. Jan. 2023 (CET)
PS. Die Grafiken vom Umweltbundesamt zeigen ja recht deutlich, wo die eigentlichen Einsparpotenziale liegen: Energiewirtschaft (also schnell für mehr Ökostrom-Produktion sorgen!), verarbeitendes Gewerbe (kenne ich mich nicht mit aus), Verkehr (also mehr E-Autos, mehr Züge, mehr Fahrräder…), Haushalte und Kleinverbraucher (Wärmepumpen installieren!). Die Frage ist natürlich auch immer, wie leicht man diese Potenziale heben kann… --Sabrieleauftistik (Diskussion) 21:38, 14. Jan. 2023 (CET)
Zu 80 km/h auf Landstraßen: Das gilt z.B. in der Schweiz. Dort dürfen auf Landstraßen Pkw und Lkw gleichermaßen 80 km/h fahren. Das führt zu wesentlich weniger Überholungen und verbessert damit enorm die Verkehrssicherheit. --Digamma (Diskussion) 21:38, 14. Jan. 2023 (CET)

Der Spritverbrauch pro km steigt mit der Höhe der Geschwindigkeit exponentiell an. Ein Auto, das bei Tempo 100 z.B. 8l/100km verbraucht, verbraucht bei Tempo 200 nicht 16l/100km, sondern z.B. 32 l/100km. --Doc Schneyder Disk. 20:51, 14. Jan. 2023 (CET)

Das ist doch Quark! Nach der Rechnung hätte mein Auto 50 Liter verbrauchen müssen als ich mal 100 Kilometer mit Tempo 250 gefahren bin, womit mein Tank hätte leer sein müssen. Stattdessen konnte ich die weiteren 300 Kilometer mit deutlich niedriger Geschwindigkeit problemlos ohne Tankstopp zurücklegen. Natürlich verbrauchen Autos bei hohen Geschwindigkeiten deutlich mehr Sprit, aber mit solchen Märchen bringen wir die Diskussion nicht weiter. --Carlos-X 21:04, 14. Jan. 2023 (CET)
Hallo Carlos ,schau mal hier ins Kapitel Verbrauchskurve. dort kannst Du ein Diagramm sehen, dass für ein Fahrzeug den Verbrauch über die Geschwindigkeit aufzeigt. Es werden dazu die Verbräuche der unterschiedlichen Gänge gezeigt. Wie es zu diesen Werten kommt, kannst Du im Kapitel „Rollwiderstand“ und „Luftwiderstand“ sehen. in letzterem siehst Du, dass die Geschwindigkeit im Quadrat eingeht (v2) --Elrond (Diskussion) 00:57, 15. Jan. 2023 (CET)
Das Diagramm sieht definitiv auch nicht so aus, als ob sich der Verbrauch bei doppelter Geschwindigkeit vervierfacht. Wenn ich mir meinen 7. Gang in das Diagramm reindenke, dann wäre der Verbrauch bei Tempo 200 vielleicht um 50 % höher als bei Tempo 100, aber definitiv nicht um 300 %. Und das deckt sich dann auch eher mit der Alltagserfahrung. --Carlos-X 01:29, 15. Jan. 2023 (CET)
Das mit exponenziell ist wohl nicht richtig. Mir scheint so einen Rechen/Denkfehler gabs auch bei Quarks; "„Sorry, Kritik völlig berechtigt. Wie haben in der Gleichung Luftwiderstand mit Verbrauch gleichgesetzt“." Quelle: Focis (ich bekomme da sehr agressive Telekom-werbung ...Sicherlich Post 21:16, 14. Jan. 2023 (CET)
Luftwiderstand mit Verbrauch gleichzusetzen ist nicht so falsch. Aber der Luftwiderstand wächst eben nicht exponentill, sondern "nur" quadratisch. --Digamma (Diskussion) 21:41, 14. Jan. 2023 (CET)
Ist das denn kein exponentielles Wachstum? ² ist auch ein Exponent. Und selbst wenn es rein formal (mathematisch) gesehen keines sein sollte, umgangssprachlich meint man ein ansteigendes/größer werdendes nicht-lineares Wachstum, wenn man von exponentiellem Wachstum spricht. --2A0A:A541:FDDC:0:28D0:E784:ADE4:D0E0 22:57, 14. Jan. 2023 (CET)
Es wäre mir neu, dass des den Begriff "exponentielles Wachstum" in der Umgangssprache gibt. Hier werden schlicht Begriffe falsch gebraucht. --Digamma (Diskussion) 23:00, 14. Jan. 2023 (CET)
Hallo Digamma, da geht es Dir hier und jetzt so, wie es mir mit Dehydratisiert und Dehydriert geht. Da werden fachlich klar definierte Begriffe fleißig durcheinander gewürfelt. Einmal stört es Dich, ein anderes Mal nicht. Mich stört es in beiden Fällen ;-) --Elrond (Diskussion) 01:01, 15. Jan. 2023 (CET)
Exponentiell kann ich mir auch nicht vorstellen. Bei Tempo 100 Verbrauch 8 Liter, bei Tempo 200 Verbrauch 32 Liter, dementsprechend bei Tempo 300 dann 128 Liter und bei Tempo 1000 schon 2 Mio. Liter? Gilt das auch für Flugzeuge? Durch die Pandemie sollte das Bewusstsein darüber, was exponentielles Wachstum bedeutet, eigentlich etwas weiter ins Allgemeinwissen gerutscht sein… --Sabrieleauftistik (Diskussion) 21:23, 14. Jan. 2023 (CET)
(BK)Mit dem Flugzeug fährst du aber auch nicht auf der Autobahn rum und hast ausserdem andere Widerstandswerte als die Blechkiste auf der Strasse (die so kalibriert sind das das Ding sogar fliegt ;)). Aber ernsthaft. Ja, der Spritverbauch steigt an, ob nun exponentiell ist nur eine Frage des Faktors. Die Allianz gibt ein offenbar gängiges Beispiel: 6 l/100 km für 100 km/h, 10 l100 km bei 160 km/h. Hängt natürlich vom Fahrzeug ab und was man draufschraubt. Hinzu kommt wie oben schon erwähnt die Tendenz öfter mal Bremsen zu müssen wenn viele verschiedene Geschwindigkeiten parallel gefahren werden. Und natürlich spielt auch die Fahrzeugtechnik eine Rolle: Wenn man ausserhalb von Nurburgring (und vergleichbare) nicht mehr mehr als 130 km/h in der Welt fahren kann, dann lohnt es sich auch fürm die herstelelr die Maschinerie (also das Getriebe) anders zu optimieren, womit dann eben Einsparpotentiale bei niederen Geschwindigkeiten erzielt werden könnten (eben niedere Drehzahlen). Hängt aber natürlich vom Motor ab, und bei den E-Autos werden Reibung und Luftwiderstand (dank Automatik) Hauptprobleme bleiben.--Maphry (Diskussion) 21:43, 14. Jan. 2023 (CET)
Schon klar, größenordnungsmäßig Millionen Liter Kerosinverbrauch pro 100 km bei einem Flugzeug würden die Flugbranche aber ganz schnell unrentabel machen, selbst wenn es jetzt nicht genau zwei Millionen sind. Der Unterschied kann so groß nicht sein, wenn es primär um Luftwiderstand geht (Flugzeuge sind wahrscheinlich etwas aerodynamischer, aber auf Aerodynamik wird bei Autos auch geachtet). --Sabrieleauftistik (Diskussion) 23:02, 14. Jan. 2023 (CET)
Ein Fligzeug hat eben wenn es höhere Geschwindigkeiten erreicht auch keine Bodenreibung mehr zu überwinden. Und entsprechend ist der cw-Wert beim Auto ja auch deshalb nicht zu niedrig, weil eben das Auto auf den Boden gedrückt werden muss damit es nicht abfliegt. Beim cw-Wert liegen selbst beste Autos über 1, Flugzeuge eher so bei 0,08. Hinzu kommt, dass ein Flugzeug idealerweise mit dem Wind fliegt und in der Höhe der Luftdruck und damit die Dichte absinkt. All das sorgt eben für vollkommene andere Bedingugnen bei diesem Vergleich.--Maphry (Diskussion) 23:18, 14. Jan. 2023 (CET)
"selbst beste Autos über 1"? Hmm lt. Strömungswiderstandskoeffizient#Luftwiderstandsbeiwerte von Kraftfahrzeugen irrst Du dich und auch bei der Süddeutschen erfahre ich Werte deutlich unter 1?! ...Sicherlich Post 23:36, 14. Jan. 2023 (CET)
Sorry, sollte 0,1 heissen. Entnommen aus [18].--Maphry (Diskussion) 12:48, 15. Jan. 2023 (CET)
Entscheidend für den Luftwiderstand ist weniger der  -Wert als das Produkt aus  -Wert und Querschnittsfläche. Ein kleiner  -Wert nützt wenig, wenn die Querschnittsfläche groß ist. --Digamma (Diskussion) 20:52, 15. Jan. 2023 (CET)
2 ist nur eines der möglichen Exponenten bei exponentiellem Wachstum. Es gibt auch Exponenten kleiner 2. Möglicherweise wie der Begriff exponentiell nicht verstanden. --Doc Schneyder Disk. 22:09, 14. Jan. 2023 (CET)
Wenn man durch deine beiden Beispielwerte eine Exponentialfunktion legt (diese lautet übrigens  ), dann kommen die Werte raus, die ich geschrieben habe. Wo ist da ein Exponent zwei? --Sabrieleauftistik (Diskussion) 22:57, 14. Jan. 2023 (CET)
Nein, der Spritverbrauch steigt nicht exponentiell mit der Geschwindigkeit, sondern quadratisch, weil der Luftwiderstand proportional zum Quadrat der Geschwindigkeit ist. Nicht alles, was stärker als linear wächst, wächst auch exponentiell. --Digamma (Diskussion) 21:40, 14. Jan. 2023 (CET)
auch das quadratisch ist wohl nicht korrekt; siehe link bissl weiter oben auf den Focus. ...Sicherlich Post 22:12, 14. Jan. 2023 (CET)
Ein quadratischer Anstieg ist ein exponentieller Anstieg mit dem Exponent zwei, also natürlich ebenfalls ein exponentieller Anstieg. --Doc Schneyder Disk. 22:13, 14. Jan. 2023 (CET)
Nein. Du verwechselst Exponentialfunktionen mit Potenzfunktionen. Quadratische Funktionen haben die Gestalt  , Exponentialfunktionen die Gestalt  . --Digamma (Diskussion) 22:44, 14. Jan. 2023 (CET)
…und ich habe oben die Exponentialfunktion angegeben, die sich aus den beiden Beispielwerten ergibt. --Sabrieleauftistik (Diskussion) 22:59, 14. Jan. 2023 (CET)
Wird aus einer Exponentialfunktion mit dem festen Exponenten x=2 nicht eine Quadratfunktion? Was sagen die Mathematiker hier? Ich dachte, eine Quadratfunktion sei eine Teilmenge möglicher Exponentialfunktionen, halt eine Exponentialfunktion mit dem Exponenten 2. Vielleicht kann ich ja noch was lernen... --Doc Schneyder Disk. 01:56, 15. Jan. 2023 (CET)
Eine Funktion mit einem festen Exponenten ist eine Potenzfunktion. Bei einer Exponentialfunktion ist die Basis fest und der Exponent variabel. Im Prinzip steht das schon in meinem Beitrag, auf den du antwortest.   mit festem   ist eine Potenzfunktion.   mit festem   ist eine Exponentialfunktion. --Digamma (Diskussion) 14:52, 15. Jan. 2023 (CET)
Das sind zwar korrekte Definition, die aber für die Frage nach exponentiellem Wachstum einigermaßen irrelevant sind, da letzteres selbstverständlich bei einer Potenz von 2 ebenso vorliegt wie z.B. bei einer Potenz von 1,0085 (mein großzügiger Zinssatz beim Tagesgeld).-- Seelefant (disk.) 19:28, 18. Jan. 2023 (CET)
Sorry, nein, das ist Unsinn. Exponentielles Wachstum liegt vor, wenn die unabhängige Variable (hier die Geschwindigkeit) im Exponent steht, also der Exponent variabel ist, aber die Basis fest. Das ist bei deinem Tagesgeld der Fall: Die 1,0085 sind die Basis, der Exponent ist die Zeit in Jahren. Bei der Abhängigkeit des Luftwiderstands von der Geschwindigkeit ist aber die Basis variabel (nämlich die Geschwindigkeit) und die Basis fest (nämlich = 2). --Digamma (Diskussion) 20:01, 18. Jan. 2023 (CET)
Zitat: „Das Umweltbundesamt (2020) hat berechnet, wie sich generelle Tempolimits auf Autobahnen auf die Treibhausgas-Emissionen auswirken würden. Aufgrund der besprochenen Limitationen der Messdaten von Löhe (2016) insbesondere bei Tempolimit 130 konzentrierte sich das Umweltbundesamt (2020) auf ein Tempolimit von 120 km/h. Basierend auf den Geschwindigkeitsdaten von Löhe (2016) und projizierten Kraftstoffverbrauchswerten in Abhängigkeit von der Geschwindigkeit (Hausberger 2019) ergibt sich, dass die CO2 -Emissionen um 10,8 Prozent zurückgehen würden. Bezieht man diesen Prozentsatz auf die gesamten von Pkw und leichten Nutzfahrzeugen auf Streckenabschnitten ohne Tempolimit verursachten Emissionen, ergibt sich ein Rückgang von 2,8 Millionen Tonnen, der mit einem Tempolimit 120 erzielt werden könnte. Bei einem Tempolimit 130 würden die Einsparungen auf Basis ähnlicher, aufgrund der Datenlimitationen jedoch weniger detaillierter Berechnungen bei 8,4 Prozent oder 2,2 Millionen Tonnen CO2 -Äquivalenten liegen. All diese Berechnungen unterliegen wieder der Annahme, dass der deskriptive Querschnittsvergleich von Löhe (2016) eine valide Approximation der Effekte eines Tempolimits auf die Geschwindigkeitsverteilung darstellt. Falls ein Tempolimit 130, wie in Abschnitt 2.2 erörtert, zu einem stärkeren Rückgang der Geschwindigkeiten führte, dann wäre auch mit größeren Emissionsrückgängen zu rechnen. Um die Einsparmöglichkeit von 2,2 Millionen Tonnen CO2 weiter einordnen zu können, verweist das Umweltbundesamt (2020) auf Agora Verkehrswende (2018), wo berechnet wird, dass für eine Treibhausgas-Minderung von 1 Millionen Tonnen im Jahr 2030 der Pkw-Verkehr in Städten um 6 Prozent sinken müsste. Unter Linearität würde dies bedeuten, dass ein generelles Tempolimit 130 auf deutschen Autobahnen so viel Treibhausgase einspart wie eine Reduktion des Pkw-Verkehrs in Städten um 13 Prozent.“ (Stefan Bauernschuster, Christian Traxler: Tempolimit 130 auf Autobahnen: Eine evidenzbasierte Diskussion der Auswirkungen. S. 12 f., Pre-print Version – wird veröffentlicht in den „Perspektiven der Wirtschaftspolitik“). Siehe auch ergänzend: 7 Zusammenfassende Bewertung und Ausblick S. 22 f. --88.72.111.240 22:14, 14. Jan. 2023 (CET) Ergänzend zu den sehr geringen Investitionskosten eines Tempolimits, die ja auch in ein Verhältnis zu dem Effekt gesetzt werden können: #Faktenfuchs: Genug Schilder für temporäres Tempolimit. von Karen Bauer, Bayerischer Rundfunk 20. April 2022 --88.72.111.240 22:44, 14. Jan. 2023 (CET)
Die erste Frage, die mir dazu in den Sinn kommt: Worauf beziehen sich die 10,8 Prozent Rückgang? Auf die vom Verkehr verursachten Emissionen? Dann könnten die Zahlen von Sicherlich oben ja stimmen, denn der Verkehr wiederum macht den Bundesumweltamt-Grafiken nach nicht mehr als 20 % des deutschen CO2-Ausstoßes aus. (Der Kontext im verlinkten Papier legt nahe, dass die Grundgesamtheit aber sogar nur die Emissionen von PKW und leichten Nutzfahrzeugen auf Autobahnen sind, das wäre noch mal ein Bruchteil davon.) Zweite Frage: Von wie vielen Fahrtkilometern, die momentan schneller zurückgelegt werden als die jeweils anvisierten Limits, gehen die Modellrechnungen denn aus? Ich kann mir nicht vorstellen (das liegt aber wahrscheinlich nur an meiner mangelnden Fantasie), dass tagtäglich ein großer Teil der autofahrenden Bevölkerung bei Tempo > 130 Hunderte von Kilometern durch die Republik brettert. Tatsächlich würde ich (habe ich, glaube ich, oben schon mal geschrieben) davon ausgehen, dass ein Großteil der PKW-Emissionen in Deutschland in Innenstädten (inkl. Ruhrgebiet) entsteht. --Sabrieleauftistik (Diskussion) 23:09, 14. Jan. 2023 (CET)
Nein, nicht auf den gesamten Verkehr sondern nur einen Teil davon lt. dem Zitat: "Pkw und leichten Nutzfahrzeugen": also ohne LKW (und sowieso ohne Flieger, ohne Schiffe usw.) - die Prozentrechnung ist reine Augenwischerei ...Sicherlich Post 23:31, 14. Jan. 2023 (CET)
Hab mal die Zahl von Statista.com. Für 2019 (2020 macht keinen Sinn wegen Corona): 163 Mio. t CO2 durch den Verkehr (ich vermute/hoffe komplett). Das bei den o.g. 2,2 Mio. t Einsparung: 1,35% für den Verkehr. Auf das Gesamt-CO2 bezogen ists dann wie oben schon erwähnten 0,3% ...Sicherlich Post 23:50, 14. Jan. 2023 (CET)
Tja, so ein Tempolimit würde also ziemlich wenig bringen. Ich schätze, im politischen Diskurs hält diese Idee sich deswegen so gut, weil sie relativ einfach und schnell umzusetzen wäre (anders als die Transition zur E-Mobilität oder das Hochfahren einer substanziellen Produktion von E-Fuels). Und natürlich, weil der Reduktionseffekt, so niedrig er auch sein mag, nicht von der Hand zu weisen ist. --Sabrieleauftistik (Diskussion) 12:40, 15. Jan. 2023 (CET)
Das Argument "relativ einfach und schnell umzusetzen wäre" amüsiert mich immer etwas. Denn die Realität ist: im der politischen Realität zeigt sich seit vielen Jahren, dass es eben nicht so ist sondern ein theoretischer Wunschgedanke. ...Sicherlich Post 14:35, 15. Jan. 2023 (CET)
Okay, formulieren wir’s so: einfacher und schneller umzusetzen als eine echte Umstellung hin zur Nachhaltigkeit (wozu die Transition zur E-Mobilität oder das Hochfahren einer substanziellen Produktion von E-Fuels zählen könnten). Dass die politische Realität langsam ist, stimmt wohl, aber trotzdem wären manche Vorhaben im Prinzip schneller und einfacher umsetzbar als andere. --Sabrieleauftistik (Diskussion) 15:29, 15. Jan. 2023 (CET)
das geht jetzt bissl Richtung Cafe, aber noch kurz reagiert; es hält IMO aber auch davon ab sich mit relevanten Änderung zu beschäftigen und sie anzugehen. Und dazu fühlt ich jeder gut, denn "ich fahre ja fast nie schneller als 130" - ja zum Einkaufen, für die Brötchen, die Kinder in die Schule und zum Fußballclub usw.: aber das ist ja okay denn ich fahre nie >130 ;) - solls von mir gewesen sein. Müßig Diskussion ...Sicherlich Post 16:24, 15. Jan. 2023 (CET)
Vor allem geht es ein bisschen an dem vorbei, was ich ausdrücken wollte…? Allerdings kann auch das ein Grund sein. Seit Pandemiebeginn bin ich nicht mehr schneller als 80 gefahren, das hat aber nichts mit dem Klima zu tun und eine allgemeine Geschwindigkeitsbegrenzung fände ich mehr Gängelung als sinnvollen Politikakt – „es hält IMO aber auch davon ab sich mit relevanten Änderung zu beschäftigen und sie anzugehen“, da stimme ich doch zu! Auch das ist ein Grund für Schwerpunktsetzung im politischen Diskurs, fällt bei mir übrigens mit unter „einfacher“… --Sabrieleauftistik (Diskussion) 18:03, 15. Jan. 2023 (CET)
Stellt sich halt die Frage, was mehr ins Gewicht fällt: die vielen (unnötigen) Kurzstrecken oder die Langstrecken. --Digamma (Diskussion) 21:03, 15. Jan. 2023 (CET)
Ich denke auch, dass die CO2-Reduzierung zwar ein Argument, aber nicht das wichtigste Argument für ein Tempolimit ist. Eine geringere Zahl schwerer Unfälle und reduzierte Stauhäufigkeit durch gleichmäßigeren Verkehrsfluss erscheinen mir wesentlicher. Ich bin seit Jahrzehnten Fan des Tempolimits – seit ich zum ersten Mal in Dänemark und den Niederlanden gefahren bin und erlebt habe, wie wunderbar entspannt es sich fährt, wenn alle ungefähr gleich schnell fahren, ohne jederzeit damit rechnen zu müssen, dass von hinten irgendwelche PS-Boliden angeschossen kommen und sich durchdrängeln wollen. Das funktioniert aber eben nur, wenn nicht nur einzelne Leute aus eigenem Entschluss langsamer fahren, sondern die Regel für alle gilt. --Jossi (Diskussion) 22:16, 15. Jan. 2023 (CET)
Was du beschreibst, habe ich in meiner Kindheit bei einer Familienreise nach Frankreich erlebt (die haben sogar 130, oder?). Beziehungsweise erinnere ich mich an Papas begeisterte Worte am Steuer – Tempomat an und entspannen… Das geht aber auch auf deutschen Autobahnen (also ohne Tempolimit), wenn es genug freie Fahrspuren gibt, auf die der PS-Bolide ausweichen kann, was in erster Linie eine Frage der Streckenauslastung ist (und bei entsprechend ausgelasteten Strecken ist ein angemessenes Tempolimit eh sinnvoll: Verkehrsfluss, Unfälle… du sagst es). Diese PS-Boliden gibt’s aber auch im Stadtverkehr. Die überholen mich bei vorschriftsmäßigen 50 km/h dann gern mal mit 70 oder so. Beim Blitzer überhole ich sie, da fahren sie 40… --Sabrieleauftistik (Diskussion) 22:30, 15. Jan. 2023 (CET)
Ach so, und ich glaube einfach nicht, dass man die vermeintlich unnötigen Kurzstrecken vermeiden kann. Kinder in die Schule ist so eine Sache (ich musste laufen, später Bahn fahren…), aber Einkaufen kann je nach Familiengröße, Entfernung zum Supermarkt und persönlicher Fitness (zum Thema Lastenrad) schon herausfordernd werden ohne Auto. Und gerade am Stadtrand oder noch mehr in Richtung ländlicher Raum ist das dann auch nicht unbedingt nur Bequemlichkeit, ab einer gewissen Zeitersparnis. Umweltfreundliche fahren ja nicht mit dem Auto in die Stadt, sondern mit dem Auto zum nächsten Bahnhof und dann mit der Bahn in die Stadt (mit dem Bus zum Bahnhof würde vielleicht noch eine halbe Stunde länger dauern… ist die Frage, ob das noch Bequemlichkeit ist). Das ist eine Kurzstrecke. Aber um die Kurzstrecken mache ich mir in Sachen CO2 auch für die Zukunft überhaupt keine Sorgen, weil die sich für E-Autos ja nun wirklich perfekt eignen. --Sabrieleauftistik (Diskussion) 22:38, 15. Jan. 2023 (CET)
Ich habe jetzt nicht alle Beiträge dieses Stranges gelesen, aber
Beispiel die Strecke Berlin-München, ca. 650km. Mit Tempo 100 = 6,5h , mit Tempo 130 = 5h, mit Tempo 150 = knapp 4h 20'.
ist ein höchst theoretisches Konstrukt und so nicht realistisch, zumindest nicht bei einem durchschnittlichen Verkehrsaufkommen. Wenn man mit einer höheren Marschgeschwindigkeit unterwegs sein möchte, kommt es unweigerlich, durch andere Verkehrsteilnehmern und anderen Umständen, zum einer Vielzahl von Brems- und Beschleunigungsvorgängen. Dass das Energieverschwendung ist, sollte klar sein. Das unabhängig vom sowieso höheren Verbrauch. Dass ein Tempolimit bzw. eine engere Geschwindigkeitsverteilung dazu zu einem durchschnittlich sichereren Verkehr führt, wurde sicher schon angemerkt. Wenn ich in anderen Ländern Auto fahre, finde ich das in den meisten Fällen entspannter als in D. Seltsamerweise können so gut wie alle Autofahrer außerhalb D trotz eines Tempolimits fahren, ohne sich vergewaltigt zu fühlen, nur die Deutschen konstruieren einen trotzigen Widerstand. --Elrond (Diskussion) 10:52, 18. Jan. 2023 (CET)
+1 Um mal zur Ausgangsfrage zurück zu kommen. Ich gehe immer noch davon aus, dass viele Verbrennertypen auf eine bestimmte Geschwindigkeit optimiert sind, oberhalb dieser die Fahrzeuge dann unwirtschaftlich viel verbrauchen. Hoher Verbrauch= hoher CO2-Ausstoß. Ob es dann immer genaue Berechnungen braucht, sei mal dahingestellt, die Message ist jedenfalls nicht falsch. Wie dann weiterhin anklang, hätte ein Tempolimit natürlich weitere positive Nebeneffekte, das Hollandbeispiel klang an, selber erlebt, super Sache. Grundlegende Überlegung muss doch aber Folgende sein: wir gehen auf 50 Mio Fahrzeuge zu, bei ca. 82 Mio Einwohnern !! Dazu bekommt man in den Hauptverkehrszeiten auf den Hauptautobahnen (einstellige Zahl) und auch nicht wenigen Zubringern sowieso ein Tempolimit aufgezwungen, weil schlicht zuviel Fahrzeuge unterwegs sind. Entweder wir bauen dann alles mit nem Dreifachwumms dreispurig aus oder man befleißigt sich ernsthaft mal zu anderen Verkehrsmitteln. Und selbst wenn wir 30 Mio E-Autos haben, der Strom muß erstmal bereitgestellt werden und dann steht noch die Frage der Stromerzeugung. Von der Ökobilanz der Akkuherstellung ganz zu schweigen. Über kurz oder lang wird das Wort Verzicht wieder salonfähig werden.--scif (Diskussion) 11:58, 18. Jan. 2023 (CET)
Schön gesagt, aber ich glaube beim besten Willen nicht, dass von 80 Millionen Deutschen genügend viele irgendeine Art von (subjektivem) Verzicht üben werden. Da muss entweder der Leidensdruck sehr hoch sein (hier zum Beispiel: Stau) oder es eine von sich aus attraktive Alternative geben (also: einen richtig guten ÖPNV). Bei der Sache mit dem Strom für die E-Autos hingegen mache ich mir irgendwie gar keine Sorgen. Die Energie ist ja da (Sonne, Wind), muss nur genutzt werden. Mit der Umweltbilanz von Akkus kenne ich mich zu wenig aus, um pessimistisch zu sein, aber da wird sich etwas tun müssen, sonst wird das nichts mit der Energiewende. --Sabrieleauftistik (Diskussion) 13:36, 18. Jan. 2023 (CET)
Auch auf Deutschlands Autobahnen gibt es ja an diversen Stellen Tempolimits und kaum jemand beschwert sich (manche ignorieren sie auch), nur eben kein allgemeines. Ich gehe davon aus, dass solche punktuellen Höchstgeschwindigkeiten in den meisten Fällen wegen der Verkehrssicherheit erlassen werden, also wegen konkreter Unfallgefahr aufgrund des Verkehrsaufkommens oder baulicher Bedingungen (Kurvenradius, kurze Auf-/Ausfahrt) und so weiter. Nicht auf Autobahnen, aber im Stadtverkehr haben meines Wissens auch einige Städte stellenweise durchaus Geschwindigkeitsbegrenzungen unter Tempo 50 eingerichtet, um die Schadstoffbelastung zu reduzieren (Feinstaub, ließe sich mit CO2 aber sicher genauso begründen). Im Prinzip sollte so was auf Autobahnen auch möglich sein, hoffentlich schon bei jetziger Rechtslage. In meiner laienhaften Vorstellung gibt es aber auch unzählige Autobahnstrecken, auf denen einfach so wenig Verkehr ist, dass Brems- und Beschleunigungsmanöver bei Tempo 200 nicht signifikant häufiger nötig wären als bei Tempo 130 (umgekehrt kenne ich Autobahnstrecken, wo man selbst bei Tempo 80 ständig bremsen und beschleunigen muss, weil der Verkehr so dick ist). --Sabrieleauftistik (Diskussion) 13:28, 18. Jan. 2023 (CET)
Mein POV dazu: solang man es für notwendig erachtet, 200 fahren zu müssen.... Aber das ist vielleicht auch eine Altersfrage und ressourcenschonend mit Sicherheit nicht.--scif (Diskussion) 15:30, 18. Jan. 2023 (CET)
Als das baden-württembergische Verkehrsministerium auf dem südlichen Teil der A81 ein Tempolimit von 130 verordnet hat, wurde dagegen geklagt, und zwar teilweise mit Erfolg. Auf Teilen der Strecke musste das Tempolimit wieder aufgehoben werden. Es ist bei der jetzigen Rechtslage also nicht so einfach möglich, ein Tempolimit zu erlassen. Auch in Städten gab es erfolgreiche Klagen gegen Tempo 30. Da hat sich die Rechtslage aber in den letzten Jahren verbessert. --Digamma (Diskussion) 20:08, 18. Jan. 2023 (CET)

Erdung Bohrinseln

Ich habe auf norwegischen Bohrinseln gesehen, dass diese das übliche Schuko-Stecksystem verwenden, also Nullleiter, Erde und Phase. Nur frage ich mich, sind Bohrinseln? geerdet? Wenn ja, wie sieht diese Erdung aus? Lässt man da ein Kabel bis zum Meeresgrund hängen? Ich war auf einer Bohrinsel, die nur mit Stahltrossen festgesetzt war und schwamm - also keinen Ground-Kontakt hatte. --2A02:810D:4ABF:DC60:31CA:F989:22E9:18E4 18:12, 15. Jan. 2023 (CET)

Ein Bekannter ist Inbetriebsetzer/Inbetriebnehmer von Anlagentechnik auf Bohrinseln. Leider ist er gerade in seinem Job unterwegs, sonst hätte ich ihn fragen können. Da Du bedauerlicherweise nicht angemeldet bist, kann ich Dir später schlecht eine Nachricht zukommen lassen, falls Deine Frage hier nicht beantwortet werden sollte. --158.181.68.122 18:22, 15. Jan. 2023 (CET)
Off-topic-Diskussion:
Warum willst Du Deine Antwort nicht hierher schreiben? Würde möglicherweise auch andere Mitlesende interessieren. --TheRunnerUp 18:39, 15. Jan. 2023 (CET)
Wenn er zurück ist, ist der Beitrag sicher schon archiviert  Vorlage:Smiley/Wartung/;-) . --158.181.68.122 18:43, 15. Jan. 2023 (CET)
Dann machst Du hier einen neuen Abschnitt mit Verweis auf den archivierten auf. --TheRunnerUp 18:45, 15. Jan. 2023 (CET)
Mein Bekannter arbeitet für eine niederländische Firma, wann er wieder nach Berlin kommt ist ungewiss. Man kann ja nun nicht erwarten, dass der Fragesteller alle naselang hier vorbeischaut. --158.181.68.122 18:57, 15. Jan. 2023 (CET)
Du bist ja auch nicht angemeldet. Wie soll das dann gehen mit der Nachricht, wenn ich einen Account hätte, du aber nicht? Naja.. Egal, danke für deine Meldung! Freute mich sehr! --2A02:810D:4ABF:DC60:31CA:F989:22E9:18E4 18:32, 15. Jan. 2023 (CET)
Gehört hier nicht her, und vllt. bist Du auch schon wieder weg, aber: Wenn Du angemeldet bist, hast Du eine eigene Kontaktseite, die sog. Benutzerdiskussion, auf der jede IP (und alle anderen auch) gezielt mit Dir Kontakt aufnehmen können. So funktioniert das mit der Nachricht. --G-41614 (Diskussion) 18:38, 15. Jan. 2023 (CET)
Auch als IP hat man eine Diskussionsseite... -Ani--46.114.153.141 18:45, 15. Jan. 2023 (CET)
Die a) nach einer gewissen Zeit gelöscht wird, b) keine Benachrichtigungsfunktion hat, und c) an andere dynamische IP-Adressen vergeben wird, die mit der Antwort nichts anzufangen wissen. --158.181.68.122 19:03, 15. Jan. 2023 (CET)
Nicht wirklich falsch... Aber auch nicht exakt. Benachrichtigungen z.B. werden auf Funktionsseiten als Kopfteil angezeigt, solange man noch aktiv über die IP verfügt, Auf dem Weg bin grad informiert worden dass Benutzer:G-41614 mich auf meiner Disk angesprochen hat. Und ob die Seiten automatisch gelöscht werden, da bin mir nicht sicher. Einzig dass es praktischere Wege gibt, darauf lasse ich mich gern ein. -Ani--46.114.153.141 19:33, 15. Jan. 2023 (CET)
Sie werden nach einer gewissen Zeit geleert, kannst Dich darauf verlassen. Ansonsten muss man bei einer laufenden Diskussion, dessen Antwort sich hinzieht, um längeres behalten bitten bzw. um Wiederherstellung beim löschenden Admin. Und wenn sich jemand morgen oder noch später mit einer neuen IP auf Wikipedia bewegt, bekommt er auch keine Nachricht. Ich sagte doch, es wird einige Zeit dauern, bis ich eine verlässliche Antwort aus erster Hand bekomme. --158.181.68.122 19:45, 15. Jan. 2023 (CET)
"...solang man noch aktiv über die IP verfügt..." Also, wir müssen jetzz nicht alles dreifach durchkauen, denke ich. Ich hab diesbezügich jedenfals keine Mission. Aber, wo ein Wille ist ist auch Weg. Es gibt sicher ein Tag mit man verhindern kann dass die Disk gelöscht wird, man kann die IP mit der die Frage gestellt wurde bookmarken etc. Gruß, -Ani--46.114.153.141 20:11, 15. Jan. 2023 (CET)
Es ist doch gehopst wie gesprungen, ob die IP hier oder auf ihrer ehemaligen Diskussionsseite u. U. Wochenlang nach einer Antwort suchen muss. Und bookmarken muss ich mir den Benutzernamen ohnehin, sonst habe ich irgendwann eine Antwort und weiß nicht mehr wohin damit. Die Suchfunktion mit der Trefferliste nach archivierten Artikeln ist echt grauenhaft. --158.181.68.122 21:05, 15. Jan. 2023 (CET)
Letzteres ist mir auch schon mehrfach aufgestoßen. Wäre es wenigstens chronologisch geordnet... Noch ein Beitrag OT: Wenn ihr beide die IP zur Ausgangsfrage bookmarked, ist eigentlich egal ob es irgendwann gelöscht wird falls du da antwortest. Die Antwort wird ja auf jeden Fall in der Historie angezeigt. Andere Alternative wäre hier eine Neuauflage mit Hinweis auf diesen jetzigen Thread. Allerdings sieht es für mich so aus als könne die zu erwartende Antwort auch nichts Wesentliches mehr ergänzen. So sei es denm... jetzt ist das OT-Konto aber auch schon reichlich überzogen. :) Mit Verlaub, ein lezzter Gruß^^, -Ani--46.114.153.141 21:59, 15. Jan. 2023 (CET)
Meerwasser hat eine ausreichend gute elektrische Leitfähigkeit. Habe gerade keine Daten zur Hand, aber die dürfte besser sein als die Leitfähigkeit von "normaler" Erde. --Blutgretchen (Diskussion) 18:42, 15. Jan. 2023 (CET)
Ergänzung: siehe Erdungswiderstand. Ergo: Je feuchter der Untergrund, desto besser. Das Meer dürfte mit seinem hohen Salzgehalt eine bessere Erde sei, als jedes Festland. Aber auch Flusswasser dürfte genügend Ionen enthalten, dass eine Erdung unnötig wird. --Blutgretchen (Diskussion) 18:56, 15. Jan. 2023 (CET)
Geerdet wird hier einfach, in dem man die Erdung mit der Stahlstruktur verbindet. Genau wie bei einem Schiff. --91.114.184.118 19:32, 15. Jan. 2023 (CET)
Richtig! Alles andere wäre auch sinnfrei. Selbst, wenn die Verbindung zur "richtigen" Erde schlecht wäre (was sie nicht ist), steht man doch auf dem Blech des Schwimmkörpers. Damit wird der Berührungsschutz gegenüber Betriebsmitteln mit Fehlern gewährleistet. --31.212.244.31 21:23, 15. Jan. 2023 (CET)
....wobei die Frage ist, welche Netzform man dort hat... eine Bohrinsel und erst recht ein Schiff wird / muß ja nicht am öffentlichen Stromnetz hängen, sondern erzeugt den Strom mit eigenen Generatoren. Ich bin kein Elektriker, aber z.B. im Baubereich bei den Stromaggregaten hat man einerseits die meist älteren, die mit einem Erdungsstab geerdet werden und die neueren arbeiten inzwischen mit sog. Isolationsüberwachung, wodurch mal sich das Erden spart... was auch wohl häufig unterlassen / "vergessen" wurde. Und in wiefern auf Ölbohrplattformen der Arbeitsschutz mit reinspielt, nach welchem man in Räumen, Silos, Behältern mit metallenen Wänden / Auskleidungen oder in beengten Situationen / Arbeitssituationen mit Trenntrafos (DGUV Regel 113-004 Kapitel 4.10) arbeitet weiß ich nicht. --Btr 00:14, 16. Jan. 2023 (CET)
Das geht in Richtung Schutzleitungssystem mit Isolationsüberwachung bzw Erdschlussüberwachung. --Rôtkæppchen₆₈ 00:28, 16. Jan. 2023 (CET)
Das Erden ist bei leistungsschwachen (bis wenige kW bzw. kVA) Generatoren so eine Sache... Die (konventionelle) Schutzmaßnahme ist ja eigentlich nicht der Potentialausgleich zwischen (Geräte-)Körper und "Erde", also das "Erden" des Geräts über den "SchuKo" bzw. den PE, sondern "Schutz durch Abschalten": die Erdung ist eigentlich dazu da, daß im Fehlerfall eine ausreichend kleine Schleifenimpedanz vorliegt, die zu einem genügend hohen Kurzschlußstrom führt, der das Überstromschutzorgan veranlaßt, den Fehlerstrom in Schnellzeit abzuschalten, u. a. auch aus Gründen des Brandschutzes. (Ein funktionierender niederohmiger Potentialausgleich wäre als Schutz bei indirektem Berühren ausreichend, weil der Körper dann keine unzulässig hohen Berührspannungen annehmen könnte, aber man will in der Anlage ja keine länger anstehenden Kurzschlußströme herumtoben haben, weil sonst nämlich bald die Feuerwehr kommt.) Und diese ganzen Notstromaggregate aus dem Baumarkt bringen einfach nicht die erforderliche Kurzschlußleistung dafür, weswegen die eigentlich nur von "unterwiesenen Personen" und nur mit einem RCD als Minimalschutz benutzt werden sollten. Diese ganzen neu angeschafften "Blackout-Notstromaggregate" sind im Zweifelsfall aus verschiedenen Gründen (u. a. auch Kraftstoff, Lärm und Abgase) keine gute Idee und wären eigentlich besser nicht vorhanden als vorhanden. Ob da nun noch jemand irgendwo einen Erdspieß in den Sand- oder Felsboden hämmert, oder nicht - je nun, etwas nützen tut der hauptsächlich nur psychologisch. --77.3.5.30 12:18, 19. Jan. 2023 (CET)
Daß "man" auf dem (gut leitfähigen) Blech des Baukörpers steht, ist eher die Ursache als die Lösung des Problems. Was passiert denn, wenn man in einer "normalen" Installation (TN-S-System) mit einer Hand einen (spannungsführenden) Außenleiter (mit 230 V gegen Erde) anfaßt? Antwort: nichts, wenn man auf einem isolierenden Fußboden steht (was mal eine zulässige Schutzmaßnahme war). Spannung ist nämlich ziemlich belanglos, entscheidend ist der Strom, und der fließt in dem Fall lediglich über die Körperkapazität von ca. 0,1 nF, was über den Daumen gepeilt einer Impedanz von 30 MOhm entspricht und rund 10 Mikroampère Ableitstrom bewirkt - das ist so wenig, das merkt man überhaupt nicht. Unfälle passieren erst dann, wenn man sich direkt oder indirekt zwischen zwei Außenleiter oder einen Außenleiter und Erde einschleift, und das erfordert zwei leitfähige Berührpunkte auf verschiedenen Potentialen. Gummihandschuhe und isolierende Schuhsohlen schützen also schon ganz gut vor Niederspannungs- Elektrounfällen. --77.3.5.30 11:53, 19. Jan. 2023 (CET)

Sonntagsbrötchen

Im Grunde genommen eine mehrteilige Frage. Ich gebe zu, die letzte Frage ist dann doch ein Politikum:

a) Wann durften Bäckereien in der BRD mit der Sonntagsöffnung beginnen? (Ende Achtziger? Es gab ja erst die Tricksereien, wo man für 50 Pfennig Mitglied im Brötchenklub wurde)

b) Gab es vor diesem Termin schon die Idee des Sonntagsbrötchens? Falls es ihn schon vorher gab, war das doch merkwürdig wo man doch zuvor nie solche kaufen konnte?

c) Ist das nicht alles absurd? Zuerst die Begeisterung für die Sonntagsöffnung und nun schließt bei uns eine Bäckerei nach der anderen und man muss weiter und weiter fahren um überhaupt einen Bäcker zu finden!

--Rolz Reus (Diskussion) 21:36, 14. Jan. 2023 (CET)

Ad a: 1. November 1996[19]. Unser Artikel Gesetz über den Ladenschluss schweigt sich darüber aus. Ad b: Mindestens seit 1995, bestimmt aber früher. Ad c: Aufgebackende Fabrikbrötchen, egal aus dem Supermarkt, der Tankstelle oder dem heimischen Backofen zeigen dem Bäckerhandwerk, dass Lebensmittelversorgung auch so funktioniert. Zudem klagen die Handwerksbäckereien über Personalmangel – wenn die Handwerksbäckereien leistungsgerechte Löhne zahlen wollten, wären die handwerklich gefertigten Backwaren noch teurer. Außerdem führen die Aufwendungen für Energie, Kraftstoffe und Rohstoffe dafür, dass viele Handwerksbäckereien keinen Gewinn mehr erwirtschaften können, im Gegensatz zu Backwarenfabriken, bei denen Skaleneffekte zu erträglicheren Endverbraucherpreisen führen. --Rôtkæppchen₆₈ 22:33, 14. Jan. 2023 (CET)
Die Vorschrift steht auch nicht im Ladenschlussgesetz, sondern in der dadurch erlassenen Verordnung über den Verkauf bestimmter Waren an Sonn- und Feiertagen: https://www.gesetze-im-internet.de/sonntverkv/BJNR018810957.html. Diese Verordnung gibt es bereits seit 1957, damals aber anscheinend noch ohne Bäckereien. Inzwischen öffnen einige Bäckereien im Übrigen auf Grundlage des Gaststättengesetzes: https://www.walhalla.de/news/gaststaettengesetz-erlaubt-backwarenverkauf-am-sonntag. --BlackEyedLion (Diskussion) 22:39, 14. Jan. 2023 (CET)
In der Tat, in der Fassung von 1957 gab es eine Ausnahme nur für Konditoreien, für zwei Stunden. Die Sahnetorte am Sonntagnachmittag war wohl unabdingbar ;). Grüße Dumbox (Diskussion) 08:48, 15. Jan. 2023 (CET)
Handwerklich sehr gut gemachte Brötchen sollen ja laut Meinung der Handwerksbäcker auch am nächsten Tag noch wie frisch schmecken... Was waren das noch Zeiten, als die Läden samstags spätestens 14.00 schlossen... Wie haben wir das damals nur überlebt.--scif (Diskussion) 15:38, 18. Jan. 2023 (CET)
Kann ich dir sagen - es gab weniger Paare mit zwei Berufstätigen. --Magnus (Diskussion) 15:39, 18. Jan. 2023 (CET)
Weniger? In der DDR haben nahezu alle gearbeitet. --Ralf Roletschek (Diskussion) 15:47, 18. Jan. 2023 (CET)
Das typische Sonntagsfrühstück bestand bei uns nicht aus Brötchen, sondern aus Hefezopf. Der kann durchaus zwei Tage alt sein. --Digamma (Diskussion) 20:09, 18. Jan. 2023 (CET)
In der DDR waren Bäckereien Montags geschlossen. Ich kann mich aber beim besten Willen nicht mehr entsinnen, ob sie Sonntags geöffnet waren. @Oliver S.Y.: kannst du dich erinnern? --Ralf Roletschek (Diskussion) 21:02, 18. Jan. 2023 (CET)
Nein, daran kann ich mich nicht erinnern, das gabs in der Regel in der DDR nicgt. Es waren auch nicht alle Mo geschlossen. Ein Teil hatte von Di-Sa auf, ein anderer Teil von Mo-Fr. --Global Fish (Diskussion) 21:06, 18. Jan. 2023 (CET)

Hallo! Ich bin mir auch sicher, dass in der DDR die Bäcker, welche meist private Einzelunternehmer waren die Öffnungszeiten von Dienstag bis Samstag hatten. Was dazu führte dass Samstags wesentlich mehr Brötchen gebacken wurden. ABER - mehrere Einschränkungen, es gab in vielen Landkreisen keine Großbäckereien, also mussten auch diese Bäcker die Versorung von Krankenhäusern, Schichtbetrieben, Gastronomie und Armee übernehmen. Es wurde also durchaus gebacken, aber nicht verkauft. Anderseits waren viele Konditoreien (in der DDR bis zum Ende eigenständiger Beruf) Sonntags geöffnet, und es wurde frischer Kuchen dafür zubereitet - jedoch kein Brot oder Brötchen. Eine Koordination entsprechend Global Fish will ich aber nicht ausschließen. Oliver S.Y. (Diskussion) 22:48, 18. Jan. 2023 (CET)

Also es ist ja wie so oft, es mag auch in der DDR regionale Unterschiede gegeben haben. Meine Wahrnehmung: Montags hatten normal die privaten Bäcker zu, da mussten die Konsumbrötchen herhalten. Samstags war dann meistens jedoch schon ab 6.00 Uhr auf, nicht selten gingen da aber auch schon um elf wieder die Rollos runter. Was das Konditorgedöns betrifft, würde ich mich dazu versteigen dass das sonntags vor allem eher in Cafes angeboten wurde, Ausnahmen bestätigen sicher die Regel. Es mag vielleicht sogar Unterschiede zwischen Bäckerei und Bäckerei & Konditorei gegeben haben. Denkbar ist zusätzlich, das pragmatischerweise in Urlaubsgegenden saisonal und z.B. in Bezirkshauptstädten auch sonntags der ein oder auf hatte. Davon abgekoppelt sehe ich die Werksversorgung. Das lief ja über die bezirksgeleitete Industrie und da hatte zum Schluß jeder Bezirk mind. ein Backwarenkombinat. Speziell bei der NVA kann ich mir je nach Truppenteilgröße vorstellen, dass die selber gebacken haben.--scif (Diskussion) 07:38, 19. Jan. 2023 (CET)
Bei uns waren die Bäckereien in den sowjetischen Kasernen interessant, das Schwarzbrot war Spitze. Die haben alle Tage gebacken. --Ralf Roletschek (Diskussion) 19:54, 19. Jan. 2023 (CET)
Ich bin mir in meinem Fall sehr sicher (Rostocker Wohngebiet mit diversen kleinen Läden, darunter 2 Bäckern, eine Kaufhalle gab es in der Gegend weit und breit nicht). Einer hatte Mo-Fr, der anderen Di-Sa. Wenn ich die freie Wahl hätte, würde ich als privater Bäcker sicherlich auch Di-Sa aufhaben wollen, wenn ich mich entscheiden müsste. Mehr Umsatz und kürzere Öffnungszeit, also weniger Arbeit. Aber in dem mir gut bekannten Fall war es sicher eine klare Absprache zwischen den beiden. --Global Fish (Diskussion) 21:36, 19. Jan. 2023 (CET)--Global Fish (Diskussion) 21:36, 19. Jan. 2023 (CET)
Das will doch auch gar keiner bestreiten. Das nannte sich Versorgungssicherheit. Da war man schon hinterher.--scif (Diskussion) 07:44, 20. Jan. 2023 (CET)
An Brötchen in meiner Kindheit kann ich mich nicht erinnern, sonntags gab es Rosinenstuten. Andere Brotsorten waren Weißbrot, Graubrot, Pumpernickel und Kasseler Brot, mehr gab es nicht. --2003:E7:BF2A:F792:58A3:9FE3:DB5E:D78E 07:55, 20. Jan. 2023 (CET)

vergangenen Donnerstag

Am vergangenen Donnerstag ist Karl Klühspies im Alter von 94 Jahren in München gestorben, Meldung von Freitag 13. Januar 2023, 18:52 Uhr. Ist der Mensch eher am 12.01 oder am 05.01 gestorben? "Vergangenen Donnerstag" hört sich irgendwie komisch an, wenn man von gestern redet. --Carlos-X 17:42, 14. Jan. 2023 (CET)

Falls hier nichts kommt, dann schau mal am Montag bei den Traueranzeigen: https://trauer.sueddeutsche.de/traueranzeigen-suche/ --88.64.175.189 17:50, 14. Jan. 2023 (CET)
Da gibt es anscheinend zwei Sichtweisen. Ich habe schon beide Sichtweisen angetroffen: Ein „vergangener Donnerstag“ wäre bei der ersten Sichtweise der Donnerstag der vergangenen Woche (nach meiner Beobachtung häufiger so verwendet) oder ein „vergangener Donnerstag“ könnte bei der zweiten Sichtweise auch in derselben Woche liegen, wenn jemand diese Formulierung freitags, samstags oder sonntags anwendet (nach meiner Beobachtung seltener in dieser Weise verwendet).
Ich vermute, dass der Beitrag in der Woche nach dem Versterben entworfen worden ist und sich die Formulierung:
wahrscheinlicher auf den 5. Januar bezieht. Wäre der 12. Januar gemeint, hätte man am 13. Januar vielleicht eher „gestern“ geschrieben.
MfG --Dirk123456 (Diskussion) 10:22, 16. Jan. 2023 (CET)
Wobei andererseits solche Meldungen oft so formuliert werden, dass sie auch noch richtig sind, wenn sie erst ein paar Tage später veröffentlicht werden. Der Schreiber weiß in der Regel nicht, ob die Meldung gleich in der nächsten Ausgabe der Zeitung erscheint. --Digamma (Diskussion) 18:39, 16. Jan. 2023 (CET)
@Digamma, das (18:39, 16. Jan.) denke ich eigentlich auch. Genau deshalb versuche ich immer noch zu verstehen, warum nicht „Am Donnerstag, dem [X]. Januar 2023, ist ...“ o. ä. geschrieben wurde. Geht es um Stilistik, Pietät oder war dem Autoren bzw. der Autorin beim Schreiben nichts weiter bekannt als „am vergangenen Donnerstag“?
MfG --Dirk123456 (Diskussion) 15:24, 19. Jan. 2023 (CET)
Auflösung: Er ist am 12.01 verstorben. Quelle --Carlos-X 14:33, 21. Jan. 2023 (CET)
Ähnliche Unklarheiten gibt es beim Ausdruck "nächster"+ [Wochentag], z.B. Mittwoch. Vgl. hier und hier. Ich persönlich verstehe darunter den Mittwoch, der jetzt kommt, andere den der nächsten Woche. Bei "vergangenen Donnerstag" wird wohl genauso gedacht. Das kann durchaus zu Missverständnissen führen, aber "richtig" scheint beides zu sein.--Altaripensis (Diskussion) 09:58, 22. Jan. 2023 (CET)
Ja, es scheint wohl leider beides „richtig“ zu sein (Bezugnahme auf 09:58, 22. Jan./ @Altaripensis2). Einer deiner beiden Links (der unter →atlas-alltagssprache.de) gibt dazu folgendes bekannt:
  • „Welcher Tag gemeint ist, wenn man sich für nächsten Mittwoch verabredet, ist offenbar fast überall im deutschen Sprachgebiet nicht ganz klar ...“,
wobei ich die Passagen, die das meiste Kopfschütteln verursachen, fett markiert habe. Wenn also zweie am 23. Januar 2023 telefonieren und einer sagt: „Wir treffen uns nächsten Mittwoch!“, dann kann einer am 25. Januar 2023 kommen und der andere am 1. Februar 2023 und jeder der beiden hat alles „richtig“ gemacht. Das ist ja auch schön.
MfG --Dirk123456 (Diskussion) 00:39, 23. Jan. 2023 (CET)
Das kenne ich auch. Manche Menschen verwenden diesen Mittwoch und nächsten Mittwoch synonym, bei anderen liegt eine Woche dazwischen. Bei manchen scheint auch der Wochenwechsel eine Rolle zu spielen, so dass in der Nacht auf Montag der nächste Mittwoch zu diesem Mittwoch wird. --Carlos-X 17:58, 23. Jan. 2023 (CET)
Und es gibt auch immer noch Menschen, bei denen die Woche am Sonntag anfängt. --Digamma (Diskussion) 21:05, 23. Jan. 2023 (CET)
Milliarden --Carlos-X 13:34, 24. Jan. 2023 (CET)

curl ausprobieren auf Win 10

{nicht archivieren}

Und wieso sollte das sinnvoll sein? --95.119.143.151 00:09, 10. Jan. 2023 (CET)
Da keine Begründung kam: Baustein entschärft. --77.8.98.92 01:45, 14. Jan. 2023 (CET)
Weil unten eine Frage noch nicht beantwortet ist.--Sanandros (Diskussion) 06:24, 14. Jan. 2023 (CET)
Und das wird sich weswegen doch gleich bis wann ändern? (Mal abgesehen davon, daß die Anordnung der Frage unten nicht erkennen läßt, worauf sich "dieses Verhalten" eigentlich bezieht.) --77.8.121.249 00:41, 15. Jan. 2023 (CET)
Wenn ich eine Atwort erhalten nehmen ich den Baustein auch wieder raus.--Sanandros (Diskussion) 08:03, 15. Jan. 2023 (CET)
Und bis wann wird das geschehen? Schon am St.-Nimmerleinstag, oder erst, wenn das fragliche Betriebssystem abgekündigt wurde? Es kam keine Antwort, die Dich zufriedenstellt, es wird auch keine kommen: was Du hier machst, ist Mißbrauch der Auskunft. --95.112.36.252 15:15, 15. Jan. 2023 (CET)

Ich verseteh nicht was du nicht verstehst.--Sanandros (Diskussion) 17:05, 15. Jan. 2023 (CET)

gudn tach! ich hab jetzt mal den baustein entschaerft. sollte unten die antwort immer noch nicht genuegen, waere vermutlich ein windows-forum die bessere anlaufstelle. -- seth 23:52, 19. Jan. 2023 (CET)

Hallo ich versuche gerade curl auf Win 10 Pwrshell mit Adminrechten auszuprobieren und versuche das Beispiel hier mit curl https://reqbin.com/echo aus zu führen. Jedoch bekomme ich ein 403 Fehler. Was muss ich vorher ändern damit das funktioniert?--Sanandros (Diskussion) 13:24, 1. Jan. 2023 (CET)

403 bedeutet, dass der Server dir den Abruf verbietet. Hier wird offensichtlich die von curl verwendete User-Agent-Kennung blockiert. Mit einer anderen geht's jedenfalls: curl -UserAgent "-" https://reqbin.com/echo. -- Gruß, aka 13:36, 1. Jan. 2023 (CET)
OK danke hat funktioniert, aber warum muss man das hier nicht machen?--Sanandros (Diskussion) 13:50, 1. Jan. 2023 (CET)
Das "curl"-Kommando in der Powershell ruft vermutlich nicht einfach das bekannte cURL auf, zumindest sind die Kommandozeilenparameter schon einmal andere. Deshalb kann dort auch ein beliebig anderer User-Agent hinterlegt sein. Wenn ich unter Win10 mit Cygwin eine Bash starte und darin curl aufrufe, geht es auch ohne veränderten User-Agent. -- Gruß, aka 14:48, 1. Jan. 2023 (CET)
Die besagte Webapp sendet gar keinen UA, siehste an den Response-Headern. Entspricht -A "" -Zero Thrust (Diskussion) 15:12, 1. Jan. 2023 (CET)
Für eine diesbezügliche Aussage bräuchtest du die Headers aus der Anfrage, nicht aus der Antwort. Tatsächlich sendet die Webapp Mozilla/5.0 (Windows NT 10.0; Win64; x64) AppleWebKit/537.36 (KHTML, like Gecko) Chrome/108.0.0.0 Safari/537.36 X-Middleton/1. -- Gruß, aka 09:49, 2. Jan. 2023 (CET)

OK Danke. Und wie bringt an Windoof dazu dies automatisch zu unterlassen?--Sanandros (Diskussion) 12:29, 4. Jan. 2023 (CET)

gudn tach!
eigentlich wollte ich nach WP:FZW, hab mich jedoch vertan. aber wenn ich schon mal da bin ...
da gibt's mehrere moeglichkeiten, die ich in absteigender persoenlicher priorisierung nenne:
  • windows loeschen und linux installieren oder
  • linux parallel zu windows installieren oder
  • linux-subsystem nutzen
wenn du unbedingt weiter microsoft nutzen moechtest:
  • beachte, dass "curl.exe" und "curl" unter windoof10 unterschiedliche dinge zu sein scheinen.[20][21][22]
  • nutze beim aufruf von curl.exe den parameter -A 'was auch immer du lieber senden moechtest' oder
  • lege dir eine datei %APPDATA%\_curlrc an, in der du den user agent manuell und pauschal setzt, z.b. mit user-agent = "microsoft ist was fuer karl klammer!"[23]
-- seth 23:15, 18. Jan. 2023 (CET)
OK Windows kann das effektiv nicht.--Sanandros (Diskussion) 06:53, 22. Jan. 2023 (CET)
gudn tach @Sanandros!
windows soll was effektiv nicht koennen?
ich habe dir doch die anleitungen fuer windows verlinkt. -- seth 15:33, 22. Jan. 2023 (CET)
HMM ich habe nur das für Linux gelesen. Ich werde mal dein Ding unten ausprobieren. Thx.--Sanandros (Diskussion) 08:08, 25. Jan. 2023 (CET)

Von welcher Kirche oder welchem Ort spricht der alte Herr hier (englisch)?

Ich wäre mal wieder dankbar für Englischhörundtranskribierhilfe. YouTube Transkript lässt die Stelle leider aus. In diesem Video, etwa ab 4:20, fragt Kavanagh den alten Herrn erst, ob er ein Vollzeitorganist ist oder war, was dieser bejaht, und dann, wo er gespielt hat/spielt. Wer versteht die Antwort ganz? ("Bren..."?) --Xocolatl (Diskussion) 14:52, 10. Jan. 2023 (CET)

Schwer zu verstehen, aber ich hab die Szene mal mit verlangsamter Geschwindigkeit abgespielt und da höre ich recht deutlich ein "Brighton" heraus. --Proofreader (Diskussion) 16:37, 10. Jan. 2023 (CET)
Ich habe in en:List of churches in London und en:List of towns in England, in der im Übrigen Städte wie zum Beispiel Brighton nicht enthalten sind, nichts Passendes gefunden. Brendan Kavanagh scheint den Ort im Übrigen auch nicht zu verstehen. --BlackEyedLion (Diskussion) 16:59, 10. Jan. 2023 (CET)
Ich versuch's gerade mit "organist + Andrew + Brighton" - an sich möchte ich herausfinden, wer dieser reizende Mensch ist. Bislang allerdings vergebens. Eigentlich kann ich mir auch eher vorstellen, dass er einen Kirchen- als einen Ortsnamen nennt, aber das ist vielleicht nur mein Hirngespinst. Vielen Dank jedenfalls schon mal für eure Bemühungen... --Xocolatl (Diskussion) 17:04, 10. Jan. 2023 (CET)
Auf en:List of places of worship in Brighton and Hove springt mir auf Anhieb auch kein passender Kirchenname ins Auge. -- 79.91.113.116 22:21, 10. Jan. 2023 (CET)
Poste die Frage doch mal in den Kommentaren zum Video; vielleicht haben andere ein besseres Gehör. --Proofreader (Diskussion) 17:12, 10. Jan. 2023 (CET)
Ich bin einer der "strangers", die ohne YouTube-Account leben. Aber vielleicht macht's ja jemand anderes oder vielleicht ist Kavanagh ihm auch mit seiner Karte nachgerannt und er meldet sich selbst mal. --Xocolatl (Diskussion) 17:20, 10. Jan. 2023 (CET)
Das ist total schlecht zu hören. Vielleicht kann das jemand beantworten, der Lippenlesen kann? --2001:9E8:49FA:EB00:2CAA:10CC:788D:5E52 18:33, 10. Jan. 2023 (CET)
Es braucht seit 15 Jahren keine dedizierten YouTube-Accounts mehr um bei YouTube zu kommentieren. Das macht inzwischen der Google-Account, der ja auch z.B. von Android gepusht, oder von anderen Google-Diensten wie Gmail, Google Drive, etc. benötigt wird. Wenn du also so einen hast, brauchst du keinen neuen Account wenn du den Video-Kommentar-Weg gehen willst. --Karotte Zwo (Diskussion) 10:16, 11. Jan. 2023 (CET)
Jo. Rate mal, was ich in den letzten 15 Jahren nicht gebraucht habe, Karotte Zwo;-) --Xocolatl (Diskussion) 17:52, 11. Jan. 2023 (CET)
Hab zwischenzeitlich selbst gepostet und warte mal ab, ob wer antwortet, aber ich fürchte, das wird auch für Muttersprachler nicht viel leichter zu verstehen sein. --Proofreader (Diskussion) 13:53, 11. Jan. 2023 (CET)
Danke, hoffen kann man ja mal. Das Video ist übrigens auch auf Facebook, aber dort pflegen die Kommentare ja ziemlich dämlich zu sein... --Xocolatl (Diskussion) 17:52, 11. Jan. 2023 (CET)
Hast du schon beim Pendant der Auskunft bei der en:wp nachgefragt? Da findet sich möglicherweise eher jemand der versteht, was da gesprochen wird. --95.116.66.241 19:02, 11. Jan. 2023 (CET)
Gute Idee, das hab ich jetzt mal probiert. ([24]) --Xocolatl (Diskussion) 19:44, 11. Jan. 2023 (CET)
Dort kam jetzt für das "Bre..." die Erklärung, dass der gute Herr mit einer Silbe angesetzt und sich dann korrigiert hat, indem er so etwas sagt wie "Bre... Brighton"; das ist ziemlich genau das, was ich mit Geschwindigkeit 0,75 auch raushöre. Dort wurde dann angemerkt, dass es in Brighton eine Reihe von Kirchen gebe, wo er als Organist tätig sein könnte - an welcher, wird sich wohl kaum feststellen lassen. --Proofreader (Diskussion) 21:42, 12. Jan. 2023 (CET)
Wenn es tatsächlich Brighton ist, vielleicht kennt man den Herrn ja dort. --Proofreader (Diskussion) 21:45, 12. Jan. 2023 (CET)
Oh ja, sehr wahrscheinlich. Wenn ich auf die Galerie gehe, dann ist er wohl auf einigen Bildern zu sehen. Man suche insbesondere nach einem Bild mit der Unterschrift "Andrew and Alastair played the Frobenius", Deeplink: [25] . (Das sind Bilder von Exkursionen/Treffen, geben also noch keinen Hinweis, an welcher Kirche er wirkt. Ich bin noch nicht durch die ganze Galerie durchgegangen, vielleicht gibt es irgendwo noch weitere Indizien).-- 79.91.113.116 09:34, 13. Jan. 2023 (CET)
Nicht jeder Organist hat eine feste Anstellung bei einer Kirche. Vielleicht handelt es sich bei dem Gesuchten um Andrew Beaizley, der bei der Brighton & District Organists’ Association unter Officers and Council Members aufgeführt ist? --pretobras (Diskussion) 17:43, 13. Jan. 2023 (CET)
Ist er das auf dem Foto? [26]. Es könnte sein, ganz sicher bin ich mir nicht. -- 79.91.113.116 18:04, 13. Jan. 2023 (CET)
Da steht aber "Paul Shelley" daneben und dass der Gesuchte sich Andrew nennt, ist in dem Livestream ziemlich am Schluss, denke ich, deutlich zu hören. Aber Proofreaders Galeriebilder finde ich einen ziemlich heißen Tipp! Den Andrew Beaizley hatte ich neulich wegen irgendeines Bilderfundes ausgeschlossen, aber vielleicht war das auch ein Irrtum. Ich kann gerade nicht mehr rekonstruieren, was mir da in die Finger gefallen war... --Xocolatl (Diskussion) 19:56, 13. Jan. 2023 (CET)
Ich denke schon eher, dass das Bild den Andrew zeigen soll, da der Name ihm direkt auf die Brust geheftet ist. Paul Shelley könnte eher der auf dem Bild ganz oben sein, was eher zum "accompanied" (durch einen professionellen Sänger) passen könnte, während das Bild unten links ohne weiteres Material gut auf den Chorleiter passen könnte (und Jessica Leschnikoff ist auch erwähnt, ohne dass ein Bild daneben wäre). Eine interessante Vokalgruppe in "Beaizley", untypisch fürs Englische. -- 79.91.113.116 21:39, 13. Jan. 2023 (CET)
Du hast so recht. Facebook hatte mir mit der Anmeldeaufforderung den unteren Teil des Bildes abgeschnitten... --Xocolatl (Diskussion) 13:24, 14. Jan. 2023 (CET)
Ich war jetzt mal mutig und hab die Brighton & District Organists' Association angemailt; wenn es irgendjemand weiß, dann die. Sobald ich Antwort bekomme, gebe ich natürlich hier Bescheid. --Proofreader (Diskussion) 22:23, 13. Jan. 2023 (CET)

So, Andrew Beaizley ist offenbar ein ganz heißer Tipp. Schaut mal hier [27] bei 3:24. Der Mann war vier Jahre lang bis 2018 Orgelbegleitung des Chors (Brighton Male Voice Choir) und dessen Managing Director, außerdem war er President und dann Secretary der Brighton Organists' Association und er hat in verschiedenen Kirchen und bei einer Reihe von Orgelaufführungen gespielt (sagt diese Quelle: [28]). Übrigens steht die Orgel, an der er in dem Bild aus der Gallery zu sehen ist, hier: en:Lancing College Chapel, eine prachtvolle Kirche nordwestlich von Brighton und die Orgel stammt von der dänischen Orgelbauerfirma Frobenius, die in den letzten hundert Jahren über 1.000 Orgeln gebaut hat, die im Netz als die "Rolls-Royce unter den Orgeln" bezeichnet werden. Wir haben ein Bild der Orgel in den Commons: [29], auch bei flickr gibt es ein Bild davon: [30]. Der Chor war 2015 in dieser Kirche aufgetreten und Andrew wird auch bei dieser Gelegenheit die Orgelbegleitung übernommen haben. Also, für mich spricht einiges dafür, dass das der Gesuchte ist. Ich vermute, dass die Brighton Organists' Association, an die ich wie gesagt gemailt habe und deren Präsident er war, mir das bestätigen wird. --Proofreader (Diskussion) 23:55, 13. Jan. 2023 (CET)

Oja, und vom Alter her würde Beaizley auch passen, der übrigens auch als Komponist gewirkt hat und offenbar in Hove wohnt(e) - s. z. B. diese Quelle aus dem Jahr 2014. Ich habe wirklich keine Ahnung mehr, was mich neulich von dieser Spur abgebracht hat. Cool. --Xocolatl (Diskussion) 13:24, 14. Jan. 2023 (CET)

Also, Alistair MacFadyen von der Brighton & District Organists' Association (sicher der Alistair, der im Foto aus der gallery erwähnt wurde) hat mir jetzt geantwortet; dort hat man inzwischen mitbekommen, dass wir die Person identifiziert haben und jetzt fragt er, ob diese Rechercheaktion, von der man dort ziemlich begeistert ist, im nächsten Newsletter erwähnen dürfe, der momentan zusammengestellt wird. Von meiner Seite dazu natürlich keine Einwände; so kann man auch in die Zeitung kommen (auch wenn's nur ne kleine Mitgliederzeitung der Brightoner Orgelfreunde ist) :-). Xocolatl, Einwände deinerseits? Sonst gebe ich entsprechendes Feedback nach Brighton zurück. --Proofreader (Diskussion) 20:45, 15. Jan. 2023 (CET)

Hi Proofreader, das ist ja klasse, herzlichen Dank dir und allen anderen, die mitrecherchiert haben! Nein, natürlich habe ich keine Einwände. Weißt du zufällig, ob dieser Newsletter auch für uns bzw. überhaupt für die Öffentlichkeit einsehbar ist? --Xocolatl (Diskussion) 15:41, 16. Jan. 2023 (CET)
Ich frage mal nach. --Proofreader (Diskussion) 17:08, 16. Jan. 2023 (CET)
Antwort kam prompt: Der Newsletter geht nur an die Mitglieder der Association. Man könnte aber nach der Veröffentlichung ein pdf davon erhalten. --Proofreader (Diskussion) 17:36, 16. Jan. 2023 (CET)
Schade, ich hatte schon gehofft, man könne diesen Newsletter dann vielleicht auch als Beleg für unseren Kavanagh-Artikel nutzen (sofern dann überhaupt etwas über Dr K drinsteht, vielleicht beschränken sie sich ja auch auf Beaizley). Wer ist "man", du? --Xocolatl (Diskussion) 18:11, 16. Jan. 2023 (CET)
In der Mail hieß es, ich könnte ein pdf zugesandt bekommen, wobei ich annehme, dass ich das dann auch individuell weitergeben darf. Aber als Quelle für einen Wikipedia-Artikel wäre das natürlich sehr grenzwertig, da das dann nicht wirklich öffentlich zugänglich und jederzeit für jeden nachprüfbar wäre. --Proofreader (Diskussion) 18:50, 16. Jan. 2023 (CET)
Nein, dafür ist es dann nicht zu verwenden, aber ich bin trotzdem gespannt und hoffe, dass du zumindest berichtest, wenn du das PDF kriegst. --Xocolatl (Diskussion) 14:57, 17. Jan. 2023 (CET)
Das Youtube-Video hat übrigens in den 6 Tagen bis jetzt 51.000 Views und fast 2.000 Likes bekommen. --Proofreader (Diskussion) 21:00, 15. Jan. 2023 (CET)
Das ist leider nicht mal soooo viel für diesen Kanal... ich meine mich zu erinnern, dass vor ein paar Monaten ein Video mit der Sängerin Jessica Norton innerhalb weniger Tage im Millionenklickbereich war. Naja, egal. Beaizley taucht, das wollte ich für Interessierte (Proofreaders Hinweis auf YouTube hat offenbar bislang niemanden interessiert) hier noch hinterlassen, in vielen Bänden des Royal National College for the Blind Annual Report auf, und man findet etwas mehr, wenn man nicht nach "Andrew Beaizley", sondern nach "J. A. Beaizley" googelt, wobei jetzt die Anschlussfrage wäre, für welchen Vornamen das "J." steht. Und 1970/71 scheint er in Australien gewesen zu sein, s. z. B. dies hier (mit einem Bild auf S. 3). --Xocolatl (Diskussion) 16:22, 17. Jan. 2023 (CET) Nachtrag: Sooo, jetzt hab ich seinen Eintrag im International Who's who in Music and Musician's Directory gefunden:-) --Xocolatl (Diskussion) 19:23, 17. Jan. 2023 (CET)
Mir war Kavanagh bisher unbekannt und nachdem ich ein paar Videos angeschaut habe, muss ich sagen, dass sein YouTube-Kanal musikalisch ziemlich enttäuschend ist, da fehlt meist das Gefühl für die Musik, aber es geht wohl auch eher um Show und Selbstvermarktung. --pretobras (Diskussion) 18:51, 20. Jan. 2023 (CET)
Mhm, Beaizley ist eine Perle zwischen ziemlich viel... anderem. Die Erfolgsgeschichte dieses Kanals hat sicher nicht nur musikalische Wurzeln. Wobei ich z. B. die ganz alten Videos aus der Zeit der Tutorials schon interessant finde und Kavanagh tatsächlich eine Menge Leute dazu gebracht hat, mit dem Klavierspielen anzufangen. Ich weiß nur nicht, wieviele es dann wie weit gebracht haben. --Xocolatl (Diskussion) 12:29, 21. Jan. 2023 (CET) - Nachtrag: Ich habe heute eine jüngere Auflage (die 15.) dieses Who's Who in die Hände bekommen. Darin gibt es keinen Abschnitt mehr über Beaizley :-( --Xocolatl (Diskussion) 20:58, 23. Jan. 2023 (CET)
Dann wäre es schön, wenn wir einen Artikel über den Organisten lesen könnten. MfG --Regiomontanus (Fragen und Antworten) 19:08, 24. Jan. 2023 (CET)
Ich würde mit Vergnügen einen schreiben... wie kommt denn die Redaktion eines Who's who dazu, einen lebenden Menschen, der schon einmal einen Eintrag hatte, wieder zu entfernen? --Xocolatl (Diskussion) 20:42, 25. Jan. 2023 (CET) - Ok, kleiner Stub, wer helfen kann, der helfe: John Andrew Beaizley. --Xocolatl (Diskussion) 14:47, 26. Jan. 2023 (CET)